X



トップページ物理
1002コメント416KB
■ちょっとした物理の質問はここに書いてね267■
レス数が1000を超えています。これ以上書き込みはできません。
0001ご冗談でしょう?名無しさん
垢版 |
2021/07/29(木) 08:57:30.23ID:oyBq7zFL
★荒らし厳禁、煽りは黙殺
★書き込む前に   >>2   の注意事項を読んでね
★数式の書き方(参考)はこちら   >>3-5   (予備リンク:   >>2-10   )

===質問者へ===

重要 【 丸 投 げ 禁 止 】

・質問する前に
1. 教科書や参考書をよく読む
2. http://www.google.com/ などの検索サイトを利用し、各自で調べる
3. 学生は自分の学年、物理科目の履修具合を書く
4. 宿題を聞くときは、どこまでやってみてどこが分からないのかを書く
・質問に対する回答には返答してね、感謝だけでなく「分からん」とかダメ出しでもOK
・質問するときはage&ID表示推奨
・高度すぎる質問には住人は回答できないかもしれないけれど、了承の上での質問なら大歓迎

===回答者へ===

・丸投げは専用スレに誘導
・不快な質問は無視、構った方が負け
・質問者の理解度に応じた適切な回答をよろしく
・単発質問スレを発見したらこのスレッドへの誘導をよろしくね
・逆に議論が深まりそうなら新スレ立てて移動するのもあり
・板違いの質問は適切な板に誘導を
・不適切な回答は適宜訂正、名回答は素直に賞賛

※前スレ
■ちょっとした物理の質問はここに書いてね262■
https://rio2016.5ch.net/test/read.cgi/sci/1614996014/
■ちょっとした物理の質問はここに書いてね263■
https://rio2016.5ch.net/test/read.cgi/sci/1618480057/
■ちょっとした物理の質問はここに書いてね264■
https://rio2016.5ch.net/test/read.cgi/sci/1621079610/
■ちょっとした物理の質問はここに書いてね265■
https://rio2016.5ch.net/test/read.cgi/sci/1624111571/
■ちょっとした物理の質問はここに書いてね266■
https://rio2016.5ch.net/test/read.cgi/sci/1625578988/
0002ご冗談でしょう?名無しさん
垢版 |
2021/07/29(木) 08:59:20.76ID:oyBq7zFL
書き込む際の注意

1.) 以下のような質問に物理板住人はうんざりしていますので荒れます。
しないでください。
「相対性理論は間違っています」「宇宙論は間違っています」
「量子力学は間違っています」「確率論はまちがっています」
「シュレディンガーの猫は変です」
「永久機関を作りました」「タイムマシンについて教えて」

「どうして〜?・なぜ〜?」:物理で答えられる問題とは限りません。
質問によっては哲学板・雑談系板へ誘導されるかも。
意図的になされた物理と関係ない質問:スルーの方向で

参考サイト
Wikipedia http://ja.wikipedia.org/
物理のかぎしっぽ http://hooktail.sub.jp/
EMANの物理学 http://homepage2.nifty.com/eman/
ときわ台学 http://www.f-denshi.com/
以上のサイトの説明はすべてが正確なわけではありません。
このスレでの受け答えもそうですが。相互に補完しつつ精度を高めましょう。

Wolfram Alpha 計算知能 https://www.wolframalpha.com/
簡単な計算ならWolfram先生が答えてくれます。
0003ご冗談でしょう?名無しさん
垢版 |
2021/07/29(木) 08:59:42.28ID:oyBq7zFL
数式の書き方 ※適切にスペースを入れると読みやすくなります

●括弧: (), [], {}を適切に入れ子にして分かりやすく書く
●スカラー: a,b,...,z, A,...,Z, α,β,...,ω, Α,Β,...,Ω,...(「ぎりしゃ」「あるふぁ〜おめが」で変換)
●ベクトル: V=(v1,v2,...), |V>,V↑, (混同しないならスカラーの記号でいい。通常は縦ベクトル)
●テンソル: T^[i,j,k...]_[p,q,r,...], T[i,j,k,...; p,q,r,...]  (上下付き1成分表示)
●行列: M[i,j], I[i,j]=δ_[i,j] M = [[M[1,1],M[2,1],...], [M[1,2],M[2,2],...],...], I = [[1,0,0,...],[0,1,0,...],...]
 (右は全成分表示。行または列ごとに表示する。例:M=[[1,-1],[3,2]])
●対角行列: diag(a,b) = [[a,0],[0,b]]
●転置行列・随伴行列:M^T, M†("†"は「だがー」で変換可) ●行列式・トレース:|A|=det(A), tr(A)
●複号: a±b("±"は「きごう」で変換可)
●内積・外積: a・b, a×b
●関数・汎関数・数列: f(x), F[x(t)] {a_n}
●平方根: √(a+b) = (a+b)^(1/2) = sqrt(a+b) ("√"は「るーと」で変換可)
●指数関数・対数関数: exp(x+y)=e^(x+y) ln(x)=log_e(x) (底を省略して単にlogと書いたとき多くは自然対数)
 括弧を省略しても意味が容易に分かるときは省略可: sin(x) = sin x
●三角関数、逆三角関数、双曲線関数: sin(a), cos(x+y), tan(x/2), asin(x)=sin^[-1](x), cosh(x)=[e^x+e^(-x)]/2
●絶対値:|x| ●ノルム:||x|| ●共役複素数:z^* = conj(z)
●階乗:n!=n*(n-1)*(n-2)*...*2*1, n!!=n*(n-2)*(n-4)*…
0004ご冗談でしょう?名無しさん
垢版 |
2021/07/29(木) 09:00:02.75ID:oyBq7zFL
a:加速度、昇降演算子 A:振幅、ベクトルポテンシャル B:磁束密度 c:光速 C:定数、熱・電気容量
d:次元、深さ D:領域、電束密度 e:自然対数の底、素電荷 E:エネルギー、電場
f:周波数 f,F:力 F:Helmholtzエネルギー g:重力加速度、伝導度
G:万有引力定数、Gibbsエネルギー、重心 h:高さ、プランク定数 H:エンタルピー、Hamiltonian、磁場
i:虚数単位 i,j,k,l,m:整数のインデックス I:電流、慣性モーメント j:電流密度・流束密度
J:グランドポテンシャル、一般の角運動量 k:バネ定数、波数、Boltzmann定数 K:運動エネルギー
l,L:長さ L:Lagrangian、角運動量、インダクタンス m,M:質量 n:物質量 N:個数、トルク
M:磁化 O:原点 p:双極子モーメント p,P:運動量、圧力 P:分極 q:波数
q,Q:一般化座標、電荷 Q:熱 r:距離 R:抵抗、気体定数 S:エントロピー、面積 t:時間 T:温度
U:ポテンシャル、内部エネルギー v:速度 V:体積、ポテンシャル、電位
W:仕事、状態数 x,y,z:変数、位置 z:複素変数 Z:分配関数
0005ご冗談でしょう?名無しさん
垢版 |
2021/07/29(木) 09:01:54.41ID:oyBq7zFL
前スレ
>>995の人
何から何までありがとうございます。
数学をもう少しちゃんとやらないといけないと実感しました。ありがとうございました。
0007太上天君
垢版 |
2021/07/29(木) 16:09:46.06ID:tSlUAf7Q
>>1
Wolfram先生に「キムヨナの太腿の周囲は何pですか?」という質問をしたけど、答えてくれなかった・・・・
0010ご冗談でしょう?名無しさん
垢版 |
2021/07/29(木) 20:21:20.45ID:ABtX3DJG
>>5
いえいえどう鼬まして!❤
こんなのお安い誤用ですぜ、ダンナ🤣
ついでに>>1乙🙄🤡🤢(´ω`🔴)
0012P○ΘM
垢版 |
2021/07/29(木) 20:47:25.91ID:???
冷度と凍で冷凍ァー(「ヒーター」のイントネーション)作ってくれ

壁に水の細管張り巡らせる壁水冷も実用化してないんだよね

暑ぃ
0015ご冗談でしょう?名無しさん
垢版 |
2021/07/29(木) 23:50:07.40ID:tHNFF8Pj
>>12
エアコン買え。寒くなるぞ、マジで。🤓🤡🤢(´Д`❄)
0017ご冗談でしょう?名無しさん
垢版 |
2021/07/30(金) 02:03:28.06ID:QFI1q/+a
>>12
家にガスコンロはあるか?🙄🤡🤢(´Д`⛄)
ガス冷蔵庫なんてのがあるくらいだから、
ガスで冷やすエアコン作れ。
0020ご冗談でしょう?名無しさん
垢版 |
2021/07/30(金) 14:13:08.21ID:???
昔、レーザー光の色を変換する結晶があると聞いたことがある
KDPだかKTPだか
どういう原理なんだろう
0021ご冗談でしょう?名無しさん
垢版 |
2021/07/30(金) 15:57:24.14ID:???
光学応答が非線形だと波形が歪む
歪んだ波には高調波が含まれるから
それを増幅すれば2倍や3倍周波数の光が作れる
KDPはリン酸2水素カリウムで結晶内のイオン配置が
水素とカリウムで偏ってるため非線形光学素子に使われる
0022P○ΘM
垢版 |
2021/07/30(金) 19:27:11.20ID:???
今回は荒らしじゃなくちゃんとした質問なんだけど
あと、地球科学かもしれないけど
今日雷すごかったじゃん
で、なんとなく積乱雲とか金床雲とか調べてたんだけど、
ついでに地震の前兆として報告されていること調べて、地震雲(実際には実在しないかもしれないが)が発生するなら、断層からの電磁波、地磁気、空電気、と(何度も見てるけど)書いてあって
金床雲思い出して
積乱雲発生すると(成層圏に届くと)金床雲発生するけど
その金床雲って迫り出すように大きいけど、迫り出さない小さい金床雲もあるかな
ってそこまで思ったとき

もしかして!
金床雲が雷の発生原因じゃないか!…って

金床雲が小さい範囲なら、積乱雲の中心付近しか雷ならないし
金床雲がズレてたり、歪んでたりすれば雷の範囲も変わるし
今回みたいに(予想ね)金床雲が色んなところに点在していたんじゃないか…って

金床雲の必要もないけれど
つまり要点は

"対流圏と成層圏の境に雲があると雷が発生するんじゃないか?"

対流圏と成層圏って電離度違うだろうし、
対流圏の中、雲のなかで氷が摩擦して雷が溜まるってより
対流圏と成層圏の中、2つの異相にある氷が電離度の違いで、氷が行き来すると、静電分極し、雲(の氷)に静電気が溜まる

ってことは考えられる?

それなら、積乱雲の範囲外の降雨がまだない先端で発雷するとか
海上の特に発雷してない(積乱雲でないただの雲しかない)何もない場所で1回だけ発雷するとか

全て対流圏と成層圏の境に雲があるかってことが発雷の条件なんじゃないかって

これどう考える?
これって考えられる?

タイムリーネタ
0028ご冗談でしょう?名無しさん
垢版 |
2021/07/31(土) 06:23:14.44ID:???
>>22

>今日雷すごかったじゃん

いや知らんし。世界がおまえの住んでる町だけで閉じているとか思ってんじゃねぇ。
天文・気象板ってのがあるんだから、そっちで聞け。
0029P○ΘM
垢版 |
2021/07/31(土) 11:57:03.51ID:???
まともに質問した方が当たり酷いって意味分からんな
まともに質問しないときの方が当たり弱いぞ
0030P○ΘM
垢版 |
2021/07/31(土) 12:10:13.27ID:???
上レーザーの話だけど
重力波の観測ってL字のトンネル作って光の到達時間で観測してるけど
重力波もしくは重力を光の大規模なトンネル作らず、小さな箱の中の光子で直接観測できる方法ってないかな
量子論にそんな効果ってない?
重力波が空間と時間の歪みなら、光子などの量子の不確定性と相互作用しない?そんな効果ない?
0032P○ΘM
垢版 |
2021/07/31(土) 12:15:48.72ID:???
あとついでに

先生!雷ってどうやって発生するの…?(小学生の質問並感)
0034P○ΘM
垢版 |
2021/07/31(土) 13:49:51.31ID:???
なるほど
0035ご冗談でしょう?名無しさん
垢版 |
2021/07/31(土) 17:55:25.64ID:k6vYr0ND
>>32
ふぁいんまんぶちゅりがくにかいてあるお。🙆🤓🤡🤢(´Д`🔴)イマダケタダデヨメマス!
0036ご冗談でしょう?名無しさん
垢版 |
2021/07/31(土) 18:21:23.67ID:???
米国LIGOとイタリアVirgoが同時に中性子星とブラックホールの連星からの重力波を何度も観測に成功してる
日本のKAGRAは干渉計の長さが3kmで他と大差ないが、検出器が未だに完成せず、仲間外れ。
日本の科学技術力が衰退してる証拠だろうな、底辺の物理スレも酷すぎるし。
0040ご冗談でしょう?名無しさん
垢版 |
2021/07/31(土) 19:27:03.28ID:k6vYr0ND
>>31
良かったら、あなたのおツラいこと、
打ち明けて見ませんか?🤓🤡🤢(´ω`⛄)
0043ご冗談でしょう?名無しさん
垢版 |
2021/07/31(土) 21:46:17.06ID:k6vYr0ND
>>41
あなた、気持ち悪いんだって!🤣
0047ご冗談でしょう?名無しさん
垢版 |
2021/08/01(日) 16:55:35.44ID:???
物理学の常識を覆すような発見、その他とんでもなく有意義だと思われる指摘を書き込む際は、読み逃しを避けるため是非とも顔の絵文字を付けて目立たせて下さい。
0049テスト
垢版 |
2021/08/01(日) 20:32:41.37ID:???
(・∀・)0の発見
0050テスト
垢版 |
2021/08/01(日) 20:34:00.45ID:???
(・∀・)地動説の発見
0051テスト
垢版 |
2021/08/01(日) 20:34:46.46ID:???
(・∀・)どう?飛ばせる?
0052ご冗談でしょう?名無しさん
垢版 |
2021/08/02(月) 02:29:24.11ID:???
微分と変分って何がどう違うの?
変分の計算で微分の規則を
なんの断りもなく使ったりして
さっぱり意味がわからない。
0055ご冗談でしょう?名無しさん
垢版 |
2021/08/02(月) 04:35:49.72ID:Fj0ujCcZ
原子間間隔dの一次元結晶格子においてバンドギャップを生じる電子のエネルギーってわかりますか?
0057ご冗談でしょう?名無しさん
垢版 |
2021/08/02(月) 08:41:37.89ID:???
>>52は俺も知りたい
汎関数を微分するようなもんだからって言葉で納得してるけど
連鎖律が成り立つことも展開して2次以上を無視する操作をして変分に問題がないことも全く理解してない(微分だったら2次以上の項は→0で消えるってすぐ示せるけど)
0058ご冗談でしょう?名無しさん
垢版 |
2021/08/02(月) 09:13:53.01ID:b2kh9V80
水力発電で思ったんですが、水の落差から発電エネルギーを取り出すということは水のエネルギーが何か失われることになるはずですが、それは何なのでしょうか?
例えば河川の上流から下流に水は流れていて、途中に発電機があろうとなかろうと水は本来の水路に戻るだけのように思えますが、流速の減少とか起きているんでしょうか?
0059ご冗談でしょう?名無しさん
垢版 |
2021/08/02(月) 09:56:07.15ID:???
タービンを回す分のエネルギーを失ってるよ
ただ具体的にどの程度流速に影響があるのかは知らない
0060ご冗談でしょう?名無しさん
垢版 |
2021/08/02(月) 10:02:14.20ID:???
>>57
だから一緒よ
変数自体が関数になってるけど
関数自体をちょっと動かすだけだで

2次が消えるのも微分と完全に同じだよ
0061ご冗談でしょう?名無しさん
垢版 |
2021/08/02(月) 10:12:49.81ID:???
>>58
発電機がなければ粘性抵抗などで熱に変わっていたであろうエネルギーかな
発電機から充分下流では、最終的な流速はほとんど変わらない気がする
0063ご冗談でしょう?名無しさん
垢版 |
2021/08/02(月) 12:43:00.18ID:???
>>57
汎関数はベクトルの関数と同じ
汎関数の独立変数は関数で、ベクトル関数の独立変数はベクトルだが
ベクトル自体が添字を独立変数とする関数でもある
v = (v_1, v_2, v_3) なら独立変数は 1, 2, 3 で v_1, v_2, v_3 が関数値
v_1, v_2, v_3 の代わりに v(1), v(2), v(3) と書けばまんまだろ
0064ご冗談でしょう?名無しさん
垢版 |
2021/08/02(月) 12:48:11.96ID:???
>>58
元の河川の流速と比べてるんだろうが
河川も土地の抵抗で流速を失ってるよ
それでエネルギーを無駄してるのを無駄なく取り出してるだけ
0065ご冗談でしょう?名無しさん
垢版 |
2021/08/02(月) 12:52:24.66ID:???
>>55
それだけで分かるはずねーよ
結晶格子は周期ポテンシャルで「ポテンシャルの深さ=束縛力」が影響するんだから
0067ご冗談でしょう?名無しさん
垢版 |
2021/08/03(火) 10:49:35.44ID:???
>>63
>v = (v_1, v_2, v_3) なら独立変数は 1, 2, 3 で v_1, v_2, v_3 が関数値
ここがまだ理解できない。
v_1は[v_1」で一つの記号という認識でいたんだけど
0071ご冗談でしょう?名無しさん
垢版 |
2021/08/03(火) 13:38:54.45ID:???
ずらすってのが、足し算で微小関数を足す、っていう変化しか考慮していないが、他のずらし方は考えなくても良いの?
0076ご冗談でしょう?名無しさん
垢版 |
2021/08/03(火) 14:39:28.64ID:???
重力の弱い場所でスポーツをすると記録は良くなりそうですが、
変わらない競技もありえますか
たとえば重力1/6での水泳、軽くなるのは有利だが、酸素が取りにくくなるかも知れない
0079ご冗談でしょう?名無しさん
垢版 |
2021/08/03(火) 16:56:30.24ID:ORrCJKGV
>>58
あのさ、水力発電所って、ある構造体のオマケみたいに🤓🤡🤢(´Д`🔵)
建設されてるよね?ニンゲンがそれ作らなきゃどーなると思ってる?
あー、石臼つく水車とかの話してんの?
0083ご冗談でしょう?名無しさん
垢版 |
2021/08/03(火) 20:04:51.40ID:ORrCJKGV
パシュート競技見てて思たんですが〜、☺🤓🤡🤢(´ω`😹)🚲
自転車の輪っかって、穴空いてない方が速いの?
0085ご冗談でしょう?名無しさん
垢版 |
2021/08/03(火) 20:34:47.53ID:ORrCJKGV
>>58
あのさ、今は中学の理科で🤓🤡🤢(´Д`🙆)
《位置エネルギー》って
教わらひんのん?🤣
0088ご冗談でしょう?名無しさん
垢版 |
2021/08/03(火) 23:47:28.57ID:1chbgzWa
>>86
だってぇ〜、間違いなくそゆことやなぃん?🤣🤓🤡🤢(´Д`🌀)
0089P○ΘM
垢版 |
2021/08/03(火) 23:57:10.19ID:???
>>83
多分市販の自転車は軽量化
競技用は一枚板は重いのか軽いのか
どうなんだろ
0090P○ΘM
垢版 |
2021/08/04(水) 00:30:24.58ID:???
スケール問題(ガリバー旅行記の小人の国と巨人の国を物理的に検討する話)って説明するまでもなく知ってると思うけど、一応
地球で巨人が立って歩くためには巨人が我々と同じ姿でなく、骨と筋肉がものすごい太くなければならない。同じ姿なら動けず地を這うことになる
体の質量の増加に筋肉と骨が耐えられない
もしガリバー旅行記の巨人の国の住人が我々と同じ骨格・筋肉の太さなら、そこは我々の地球とは法則の違う別世界だ
って話
スケールが違うと姿形が異ならなければならない
転じて、マクロでは重力が支配的だが、ミクロでは原子表面の電子殻の力が、もっとミクロならまあここの住人が詳しい量子の性質が支配的、という風にスケールによって支配する法則が変わり、同じ姿ではいられないっていう、教養的な話ね

例えば木星ほどの重力(地球と同環境)ならそれ相応の骨格と筋肉になるから、それがどう働くかが問題
強い重力の環境で骨格と筋肉が過度に発達するけど、過度に発達したら木星上で木星人は地球上の地球人の記録より高記録出せるのか(高記録=筋肉至上主義か)
それとも水星ほどの重力なら筋肉と骨格は退化するけど、重力のくびきは弱くなるから弱い筋力で高記録出せるのか(弱負荷至上主義)
それと
木星と水星で体の大きさも変わってくるだろうし、体格が大きいほど記録が出やすいのは外人の高記録で(タイムリー)でよく見てるんじゃないかなって思うけど、
その上で木星の重力のくびきを筋肉が凌駕するかがミソ
わからんぜよ
荒らしじゃないでしょ
0091ご冗談でしょう?名無しさん
垢版 |
2021/08/04(水) 00:51:01.54ID:JOyQBKVF
>>89
やは、ポイムちゃん!あれ、五枚くらいを張り合われてる🤓🤡🤢(´ω`🌀)
みたいだけど、使われてるのはアロンアルファかな?
0093ご冗談でしょう?名無しさん
垢版 |
2021/08/04(水) 01:15:58.63ID:???
>>91
めっちゃ生活感だけど、アロンアルフアってめっちゃ弱い
いや脆いのか
アロンアルフアで付けたものって(面積ちっちゃいからかもだけど)簡単にぽろっといく

って金属ってアロンアルファでつくっけ
0094P○ΘM
垢版 |
2021/08/04(水) 01:18:14.40ID:???
あ、カーボン強化プラスチックかな

もしかして:重くない…?
0095ご冗談でしょう?名無しさん
垢版 |
2021/08/04(水) 02:21:56.91ID:JOyQBKVF
>>58
何度読み返しても、ナニ言ってるのかよぉワカラン😭💦💦🤓🤡🤢(´Д`🔴)
も一度詳しく分かりやすぅ教えてくりや!?
0096P○ΘM
垢版 |
2021/08/04(水) 03:33:13.16ID:???
>>91
競技自転車のリムが軽いなら…って、重くても軽い素材はあるから
自転車のリム、軽量化って嘘かも。材料節約かな
安い素材ないのかな
子供の脚挟む事故よくあるから、変えた方がいいけど
円盤のリム出ないよね
0097P○ΘM
垢版 |
2021/08/04(水) 03:36:42.72ID:???
って
もしかして本当にアロンアルファで貼り合わされてるってことないよね
0098P○ΘM
垢版 |
2021/08/04(水) 03:55:03.41ID:???
グーグル検索したら
軽量化と緩衝材兼ねてるんだって
あと材料節約もかな(たぶん)

最小材料での効率設計か…
もうちょっと緩衝力強化する作り方あるだろうに
それと脚挟むの危ないから板くらい付けて欲しいけど

競技用は何で緩衝してるんだろ
スポンジ材…いやスポンジ材はありえない。板の中に骨組みがあるのかな
0104ご冗談でしょう?名無しさん
垢版 |
2021/08/04(水) 17:48:19.40ID:Z5iI4MVz
>>103
ああなるほど!あの棒がシュっワン!と空気の中をかき分ける時の
うにゅうにゅ感が、一枚の板なら減りますよね!(=゚ω゚=)
0108ご冗談でしょう?名無しさん
垢版 |
2021/08/04(水) 22:29:30.17ID:DiUavOJu
反光子凝縮、ブラックホール爆弾って、プレデター神様が惑星を消滅させる時に使うんだろう。
人間には扱えない。
0113ご冗談でしょう?名無しさん
垢版 |
2021/08/05(木) 02:56:11.42ID:S+zis/Bw
波動が全然わからない...
θが小さい時の変形わけわかめ
y方向成分の張力S(x、t)=Stanθ
???????
0114ご冗談でしょう?名無しさん
垢版 |
2021/08/05(木) 03:13:00.80ID:AuP6Qrhm
なんとなくわかってきたじょおおお
0115ご冗談でしょう?名無しさん
垢版 |
2021/08/05(木) 09:02:42.41ID:wp5wm/LM
量子力学の1次元点粒子のヒルベルト空間Hはよくいろんなところに解説が書かれてて、
無限次元で、一つの直交基底が位置に対応し、
それらが均等に混ざってるベクトルたちが作る別の直交基底が運動量に対応する
って感じでなんとなくわかるんですけど
3次元点粒子に対応するヒルベルト空間ってなかなか解説が見当たらないんですが、
どういうものですか?
0116ご冗談でしょう?名無しさん
垢版 |
2021/08/05(木) 10:42:03.77ID:???
>>115
1次元空間の座標のベクトルが解るのに、3次元空間の座標のベクトルが解らないのか
普通の知能レベルなら理解できるはずだから解説書では省略してるだけだろ。
0117ご冗談でしょう?名無しさん
垢版 |
2021/08/05(木) 10:48:52.30ID:???
どうでもいいけど内積が保存するベクトル空間程度しか要請されないのに(というかユニタリ性が定義できればいい程度)わざわざヒルベルト空間って呼ぶのって実際どうよ
0120ご冗談でしょう?名無しさん
垢版 |
2021/08/05(木) 13:42:12.85ID:???
先日NHKで物理?の教授がニュートリノとかの話をしてたんだけど、宇宙が「無」からスタートした前提だったんだけど確定してるんだっけ?
質量保存とかどうなるんだろう
0124ご冗談でしょう?名無しさん
垢版 |
2021/08/05(木) 13:59:08.36ID:???
>>115
量子力学でのヒルベルト空間は個別粒子じゃなく
全部の状態をヒルベルト空間の1つのベクトルで表すもの
粒子が1次元だろうと3次元だろうと何個あろうと1つのベクトルで表す
1つの粒子の位置と別の粒子の運動量をまとめて1つのベクトルで表しても構わん
混ざった基底になるがな
無限次元なのは何個あっても表せるようにするため
0125ご冗談でしょう?名無しさん
垢版 |
2021/08/05(木) 16:36:00.97ID:???
なぜ点と時間と空間しかないのか、すごく不思議
まったく関係のないい点同士が、すれちがったり衝突したりできる
量子性は両者をつないでいる
0127ご冗談でしょう?名無しさん
垢版 |
2021/08/05(木) 17:54:02.20ID:???
点などと思わん方がいいぞ
点の属性など個数しかないからな
場の状態として「何個ある状態」というのがあるだけだ
0128ご冗談でしょう?名無しさん
垢版 |
2021/08/05(木) 18:18:28.75ID:???
結局場ってなんなの?
一点に量子が複数詰まっててそれぞれの量子が無限次元を持ってて
その点自体が連続体を作ってるってこと?
0129ご冗談でしょう?名無しさん
垢版 |
2021/08/05(木) 20:19:28.01ID:???
>>128
ちょっとした質問では、始めから無理ということ

現実の宇宙がホントはどんな状態なのか誰もしらないし解る見込みもない。

人間が解るのは、人間が創った数学とそれで創った物理学の理論だけだから
点、空間、場、量子...の意味は基礎から順に学習すれば理論は理解できるはず。

金払って大学で教えてもらうか、貧乏なら独学でコツコツ努力するか
それをすっ飛ばして、宇宙が点、空間、場、量子...の言葉悪戯は妄想と同じ。
0130ご冗談でしょう?名無しさん
垢版 |
2021/08/05(木) 20:41:20.77ID:???
物理学工学だと数学的形式化以前からディラックのデルタ関数で一点をなんとなく定義してたような経緯もあるけどね。
0132ご冗談でしょう?名無しさん
垢版 |
2021/08/05(木) 21:03:53.83ID:BZDE10Zd
カルノーサイクルの熱効率を向上させるためには高温熱源の温度を上げるのと、
低温熱源の温度を下げるのと、どちらの方が効率が良いのですか?
単純に熱を挙げるほうがエネルギーが必要だから下げるほうが効率が良いと思ったのですが、低温熱源の温度TLは外界の温度で所与なので人為的に変えることは不可能、
という説明もあり、
0133ご冗談でしょう?名無しさん
垢版 |
2021/08/05(木) 21:34:17.63ID:???
>>128
もうちょっと説明を注意深く読んで
いい加減なでっち上げで受け取るのをやめないと
理解など遠ざかるだけだぞ
0136ご冗談でしょう?名無しさん
垢版 |
2021/08/05(木) 21:40:52.29ID:???
>>132
理論的には一定の有限温度差から、高温熱源の温度を上げて効率100%に上げるには
無限大の温度に近づける。必要なエネルギーが無限大になってしまう。

低温熱源の温度を下げて効率100%にするには絶対零度(極近く)にすればよいだけ。

>低温熱源の温度TLは外界の温度で所与なので人為的に変えることは不可能
それは現実話になるからカルノーサイクルー>スターリングエンジンに置き換える
必要がある。
高効率のスターリングエンジンは低温熱源に海水温度を利用するそうりゅう型潜水艦などで
実用化されている。
0137132
垢版 |
2021/08/05(木) 21:44:41.63ID:BZDE10Zd
結論としては、カルノーサイクルの熱効率を向上させるためには、
高温熱源の温度THを挙げるより、低温熱源の温度TLを下げるほうが効率がよいのですか?
0139ご冗談でしょう?名無しさん
垢版 |
2021/08/05(木) 21:56:45.49ID:pSPpnu9O
とにかく暑っちぃんで、愛🐿Big Mountain のスポットクーラー
買ったら、めちゃめちゃ寒いくらいに冷えます。いったいどういう
技術革新があったんですか?あんなの、しょせん吸った分以上の
廃熱出してんだから、結局部屋は暑なる一方や、あんなの買うのは
アホとバカだけやハハハ…( ´∀`)と思ったのが大間違いで驚いたわ👀‼
0140ご冗談でしょう?名無しさん
垢版 |
2021/08/05(木) 22:02:01.37ID:???
お世話になります

現在時計について学んでいるのですが、
振動子の周波数が大きくなるほど誤差が小さくなる理由が数式的に理解できません
感覚的には何となく分かるのですが、
振動数などを用いた場合の式の記述がわかりません

すれちでしたら誘導していただけると幸いです
よろしくお願いします
0142ご冗談でしょう?名無しさん
垢版 |
2021/08/05(木) 22:10:03.44ID:???
>>140
デジタル時計だろうから 周期=1/周波数
振動子の周期が時間を刻む単位になる。
物差しの単位目盛りが小さいほど誤差が小さくなるのと同じ。
0143ご冗談でしょう?名無しさん
垢版 |
2021/08/05(木) 22:47:15.67ID:???
日本で温度という概念が入ってきたのは明治以降ですか?
昭和初期あたりまでは摂氏温度よりも華氏温度の方が使われていたのでしょうか
そうだとすれば温度計がアメリカから輸入されていたからですか
0145ご冗談でしょう?名無しさん
垢版 |
2021/08/06(金) 06:18:58.85ID:???
>>142
ありがとうございます
例えの概念は分かるのですが、それを数式で表したいのです
周期が短いとして、そこから何年で一秒ずれるのか、
の算出までいきたいです

>>144
ありがとうございます
いくつかのソースがありましたが、
最初に見たのは時計の科学という本です
0146ご冗談でしょう?名無しさん
垢版 |
2021/08/06(金) 08:59:27.96ID:???
>>145
>周期が短いとして、そこから何年で一秒ずれるのか

理論的に振動数に比例する時間測定精度(誤差)と、物理的な振動子自体の精度(誤差)は
物理構造・環境で決まる量だから混同しないように
振動子の精度(誤差) = Δf/f
振動数の変動Δfは理論的な物理定数が温度などの外乱で変化する揺らぎ
現実の振動子の精度で比較すれば バネ > 水晶 >> 原子放出光
詳しい変動の値はWebなどで調べればよい。

また、混同しやすいのは
相対性理論の理論的な時間経過の違い、時計同士の遅れは変動Δf=0 の理想振動子の値。
0147ご冗談でしょう?名無しさん
垢版 |
2021/08/06(金) 10:38:32.68ID:???
車や航空機、工場の安全な自動運転は、そのもの自体では不可能である、
外部からの情報が必要であるという定理は成り立ちますか
0148ご冗談でしょう?名無しさん
垢版 |
2021/08/06(金) 11:25:12.46ID:???
>>147
意味不明

現実には車、航空機などは人間が運転してる。それを法律で認めている。
法律が要求する人間の安全運転能力以上の安全運転がAI技術で可能か?

それを現在、多種の実験で試験している。特殊なチューリングテストともいえる。
0151ご冗談でしょう?名無しさん
垢版 |
2021/08/06(金) 15:27:02.11ID:???
>>149
キミが勝手に”定理”などと言ってるんだろ

自動運転と関係なく事故の確率がゼロでないのは誰でも認めるが
「安全」の定義のコンセンサスが無ければ、定理うんぬんなど無意味だ

原発反対派が「原発の安全が保障されない」と主張し
原発賛成派が「原発の安全が保障されている」と反論する。
両派の安全の定義が同じでないから論争が終わらない。または意図的にそうする。
0152ご冗談でしょう?名無しさん
垢版 |
2021/08/06(金) 15:37:01.13ID:???
>>151
確率的に起きうることはいつか起きる自体は定理化出来るんでねえの
測度ぶっこんで無限の時間取ればいいだけだろ
0153ご冗談でしょう?名無しさん
垢版 |
2021/08/06(金) 16:01:11.51ID:???
>>145

いくつかソースがあるというなら、
 クォーツ時計:周波数 32768Hz、誤差 10^(-7)
 セシウム時計:周波数 9192631770Hz、誤差 10^(-15)
というような情報から、君がそう感じたってことかな?
0155ご冗談でしょう?名無しさん
垢版 |
2021/08/06(金) 18:23:55.68ID:???
皆様ありがとうございます

>>146
一言に誤差といっても
数値的に出される理論上の誤差と、
測定機器や環境などの精度に由来する誤差
がある、という解釈で良いでしょうか

>>153
実際に文章として「周波数が大きいほど時間当たりの誤差が小さくなる」という旨の記述があります
0156ご冗談でしょう?名無しさん
垢版 |
2021/08/06(金) 18:23:57.45ID:???
皆様ありがとうございます

>>146
一言に誤差といっても
数値的に出される理論上の誤差と、
測定機器や環境などの精度に由来する誤差
がある、という解釈で良いでしょうか

>>153
実際に文章として「周波数が大きいほど時間当たりの誤差が小さくなる」という旨の記述があります
0157ご冗談でしょう?名無しさん
垢版 |
2021/08/06(金) 18:25:03.62ID:???
皆様ありがとうございます

>>146
一言に誤差といっても
数値的に出される理論上の誤差と、
測定機器や環境などの精度に由来する誤差
がある、という解釈で良いでしょうか

>>153
実際に文章として「周波数が大きいほど時間当たりの誤差が小さくなる」という旨の記述があります
0160ご冗談でしょう?名無しさん
垢版 |
2021/08/06(金) 19:18:42.48ID:???
>>155
>実際に文章として「周波数が大きいほど時間当たりの誤差が小さくなる」という旨の記述があります

数式を使った説明は無いんだね?

おそらく、周波数が大きいほど系統誤差を評価しやすくなり技術的に誤差を抑えられるようになるか、
または周波数が大きいほど偶然誤差が平均化されて小さくなるか、そのどちらか(もしくは両方)だろうね。

後者だけなら、誤差の分布を正規分布と仮定して、その再生性を考えてみれば良いと思うよ。
0163ご冗談でしょう?名無しさん
垢版 |
2021/08/06(金) 20:57:26.14ID:???
>>157
>数値的に出される理論上の誤差と、測定機器や環境などの精度に由来する誤差がある
何が分からないのか?

100m競走の時間計測で、時計の振動子の源振動数が10Hzならば 9.8 9.9 10.0 の様に
0.1秒単位でしか測れない。
真のタイムは 9.9432 だとしても時計タイムは9.9秒誤差は+0.0432秒になる。
時計の源振動数が100Hzになれば、
時計タイムは9.94秒になり誤差は+0.0032秒になる。
つまり、源振動数を10倍にすれば誤差が約1/10になる。(1000Hzでも同様)
また、振動子の精度が10^-6 なら精度誤差は無視できる。
0164ご冗談でしょう?名無しさん
垢版 |
2021/08/06(金) 21:51:22.30ID:???
>>163
源振動数を1Hz として振動子の精度が10^-6 ならば365日の誤差は
秒数に掛けるだけ、誤差の大きさは約31秒で源振動数を10Hzにしても変わらない。
0165ご冗談でしょう?名無しさん
垢版 |
2021/08/07(土) 10:44:57.13ID:???
時計の精度に関する回答ありがとうございます

時計の精度は、最終的には式に当てはめて決めるけど、
式内の変数は測定環境等から決まる値である
なので、机上だけでは求められない(または求めにくい)

とひとまず結論づけました
もう少し自分でも調べてみます
重ねてにはなりますがありがとうございました
0166ご冗談でしょう?名無しさん
垢版 |
2021/08/07(土) 20:32:58.35ID:Sc0hg+6Y
>>162
昔、そーゆー名前のクルマあったな☺
0167ご冗談でしょう?名無しさん
垢版 |
2021/08/08(日) 08:37:14.04ID:???
シュテルン・ゲルラッハの実験を詳しく解説した教科書はありますか?理論的な説明ではなくて、実験のセットアップとデータをちゃんと書いてある本が見たいです。
0169ご冗談でしょう?名無しさん
垢版 |
2021/08/08(日) 16:50:47.39ID:GnmHZ6IN
3次元無限井戸型ポテンシャル(井戸の中のポテンシャルはゼロとします)を考えるとき波動関数はA(sin[(kx)x]sin[(ky)y]sin[(kz)z]となると思いますが、例えばkx=0、ky≠0、kz≠0のときエネルギー固有値とと波動関数はどのようになるのでしょうか。
0170ご冗談でしょう?名無しさん
垢版 |
2021/08/08(日) 17:20:47.83ID:k1dJUB8X
>>169
あれよく考えたら波動関数は普通に0になるか
でもそしたらエネルギー固有値は
E=hbar^2k^2/2m・(kx^2+ky^2+kz^2)=定数(=E0とする)×(nx^2+ny^2+nz^2)ですが
例えばnx=0,ny=1,nz=1としたときのエネルギー
E0・2/3はとらないということですか
0181太上天君
垢版 |
2021/08/09(月) 14:59:45.75ID:nZgHKkzL
キミたちよ、銀河系の中心には巨大ブラックホールがあると言われているだろ?

キミたちよ、銀河系の中心にはブラックホールはないぞ!!

ブラックホールって暗いだろ?見て見ろ!銀河系の中心は明るいぞ!!!!


https://i.imgur.com/InSk8Mn.jpg
0182ご冗談でしょう?名無しさん
垢版 |
2021/08/09(月) 18:02:55.75ID:???
>>178
次の理由により、その解法は大間違いでは?

・伸びているのでσは一定ではない。
・スリンキーは元の状態に戻ったとき、縮みしろがない特殊なバネなので
 波動方程式の一般解は制限を受けるが、それを考慮していない。
0183ご冗談でしょう?名無しさん
垢版 |
2021/08/09(月) 18:09:21.42ID:???
>>178
あと、動画をスロー再生してもそんなふうになっていないように見える。。
特に最後の屋根から長〜く伸ばしてる状態は分かりやすい。
0185178
垢版 |
2021/08/09(月) 19:04:29.57ID:???
>>182
線密度σは確かに一定ではなく、立式途中で近似と断るべきではある。一般解に関しては ct <= L の短時間に限定してるから問題無いと思う。

n 個の質点をバネで繋いだ計算例もあるが、バネの質量は無視してる。
http://www.gakushuin.ac.jp/~881791/mathbook/pdf/slinky20190509.pdf
0187ご冗談でしょう?名無しさん
垢版 |
2021/08/09(月) 19:52:55.26ID:???
>>185
よく考えてみよう。
スリンキーは、バネとしては元の状態から伸びる方向だけの特殊バネ。
これは分かるよね。

元の状態から縮む方向ではバネではなくなり、縮みしろがまったくないので
衝突体になってその反動で階段を下りていくようなよく知られた挙動になる。

手を放して、縮みしろがなくなったスリンキー上部では、重さ有りの固まりになって
落下していき下部に影響を与えるが、それが波動方程式ではまったく示せないので
>>178の解法は完全に大間違い。スリンキーではまったく使えない解法。

元の状態、つまり自然長から伸びるだけでなく、縮むこともできる普通のバネでしか
波動方程式は適用できない。

くっくっく氏の言う通り、間違いばかりがはびこってるよね。
スリンキーが動画のようになるのは難しい解析は不要で単純。
0189ご冗談でしょう?名無しさん
垢版 |
2021/08/09(月) 20:04:46.14ID:???
>>178
単純バネにしか適用できない解法を
スリンキーの特殊性抜きにドヤ顔でさらしてて大笑い。

スリンキーが分かっていない。
0193178
垢版 |
2021/08/09(月) 21:36:08.52ID:???
>>192
いや、違う
0194ご冗談でしょう?名無しさん
垢版 |
2021/08/09(月) 22:44:37.64ID:???
>>188
そうそう。
数式立てられるけど、物理学的意味が分かっていないから間違った適用をしてしまう。

スリンキーは、落下して固まりになった部分はもうバネ扱いできなくなるから
残りのバネ部分の長さがどんどん短くなっていく、つまり元の長さが固定ではなく
短くなっていくバネだから>>178はまったく論外の解法なんだよね。
とても解析学的に解ける問題ではない。
0196ご冗談でしょう?名無しさん
垢版 |
2021/08/09(月) 23:45:33.76ID:???
スリンキーの動画。
最下部に重りをつけなくてもつけても、
どうして最下部がしばらく落下しないのを不思議に感じるのか?

それは、フックの法則f=kXを単純にしか理解していないからなんだよ。
重りをぶら下げると、伸びX全体で重りを支えると誤解しているから。
そして手を放せば、バネ全体が瞬時に縮まないとおかしいと考えるから。

図のように重りをぶら下げて、バネがXだけ伸びているとする。
そのバネを半分切って、代わりにひもか棒でぶら下げるとする。
この時、残りのバネはさらに伸びて、切る前の伸び量Xになるのだろうか?

答えは、さらに伸びることはない。そのままの状態で変わらない。
これはよく考えたら分かるよね?
ごちゃごちゃ書いたら余計に分からなくなるところなので、あえて書かない。

これは余計なことだけど、伸びの状態は変わらないので
バネ全体としての伸びは半分の0.5Xになる。
つまり、バネ定数は2倍の2Kになって、f=2k×0.5X=kXで変わらない。
0197ご冗談でしょう?名無しさん
垢版 |
2021/08/09(月) 23:45:54.69ID:???
話を戻して、これを繰り返していくとバネをどんどん短くすることができる。
重りのつり合いを考えたとき、重りに働く力はあくまで重力+バネだから
この2つが相殺して釣り合う。つまり、バネ全体で重りを支えていたのではなく、
重りとの接続箇所の微小バネが重りを支えているというのが力学的には正解なんだよね。

分かるかな?
フックの法則とは、微小バネのひずみ力なんだよ。あるいは応力と言ってもいい。
重りに着目したときは、バネ全体で重りを支えているのではなく、
重りとの接続箇所の微小バネのひずみ力が重力と相殺しているわけ。要は微小バネのひずみ力。

この微小バネもひもか棒になってしまったら、つまりバネがまったくなくて
すべてがひもか棒ならば、手を放した瞬間に重りは落下する。
それはひもと棒にはほとんど「伸び」がないからだ。

バネには「伸び」があるから、重りはしばらく落下しない。
この全体の伸び量が多いほど落下開始まで時間がかかる。
それはなぜか。あとは簡単、自分で考えてみよう。

>>178は本当に情けない。こんなのばっかりだね。
スリンキーの動画みたいに極めてシンプルなものは
数式なんか使わずに説明できるものなんだよ。
0202ご冗談でしょう?名無しさん
垢版 |
2021/08/10(火) 02:17:14.44ID:???
>>196
あー分かった。
最後のバネのひずみがそのままなら
上側のバネが落下してちぢんでいっても
重りは重力と釣り合って落下しないんだな。

ひずみ復元の伝達が一番下のバネに伝わるまで
時間があるから宙に浮いているように見えるわけか。
なるほどなー
0204ご冗談でしょう?名無しさん
垢版 |
2021/08/10(火) 02:35:19.28ID:???
>>196
フックの法則って、そういう点では確かにみんな誤解してるよな。
バネのすべての微小部分に応力があって、つないだ物体に直接作用するのは
つないだ個所の微小長さのバネの応力ってことか。

この箇所がそのままの伸びなら物体に作用する力は変わらないと。
全体の伸びで考えるのは間違いだったんだな。
全体の伸びに各部の応力が比例する、それがf=kxだったと。
これは100へーだわ。フックの法則すら本当は理解できてなかった・・・

俺だけじゃなく、ほとんどの人間がそうだろ?
正直に言ってみろ。
0205ご冗談でしょう?名無しさん
垢版 |
2021/08/10(火) 02:48:33.91ID:???
動画のように手を放す。
開放された上側微小バネからつり合いが破れて
順に下側微小バネのつり合いも破れていって縮んでいく場合には
単純なフックの法則、全長の伸びf=kxなんて考えてはいけないってことだな。

一番下の微小バネのひずみは最初のままだから
重りは落下せずにいられると。
数式なんかまったくいらないわけだ、こりゃ参ったなあ。
0206ご冗談でしょう?名無しさん
垢版 |
2021/08/10(火) 02:55:30.34ID:bpOKyjS5
>>204
アホか。

ワシは最初から当たり前に知っておったわ。
あんなバネのトリックなんて昔からの定番ネタだぞ。

お前らサルはバネのなんたるかが
まるっきり分かっておらんかったのだ。
まあ、>>195の図は合格だがな。

くっくっく
0208ご冗談でしょう?名無しさん
垢版 |
2021/08/10(火) 06:22:08.74ID:???
>>207
誰かが>>178を引っ張ってくるのを待ってたんだろうな。
確かに長さが変わっていくバネに適用するのはおかしいし、
リンク貼る奴と合わせて2重に滑稽だし。
しかし、間違いを見つけるのは天才的だ。
ひょっとして相対論量子論完全否定も正しいのかもしれんなと思わせられる。
0209ご冗談でしょう?名無しさん
垢版 |
2021/08/10(火) 06:37:41.40ID:???
>>208
こそっと聞くけど
いまどきそんなオカルト2本立て本気で信じてる学者なんていないと思わない?
建前で正しいと言ってるだけで、内心では懐疑的だと思うけど
量子ってつければ予算付きやすいし、量子コンピュータって超電導または光コンピュータだからね
0211ご冗談でしょう?名無しさん
垢版 |
2021/08/10(火) 07:49:38.52ID:???
スリンキーの動画 https://www.youtube.com/watch?v=xlV_DPdu8DI
で垂直に垂らして離すと大部分が縮むまで最下位が落下しないように見える
運動は摩擦力を無視すればダランベールの原理で完全に説明できる。

誘電体コンデンサの仮想変位の原理すら理解できないqqqの類には豚に真珠。
0212ご冗談でしょう?名無しさん
垢版 |
2021/08/10(火) 10:52:22.89ID:???
バネのやつは、
単に上で放された情報の伝達速度が遅いから
下端はそれに気づかずにじっとしてるってだけじゃね?
0213ご冗談でしょう?名無しさん
垢版 |
2021/08/10(火) 12:13:41.82ID:???
>>212
>情報の伝達速度
などという妄想がどうして湧くのか、物理やめたほうがよい

下端にも質量密度があり下向きの重力が常に作用している
0218ご冗談でしょう?名無しさん
垢版 |
2021/08/10(火) 13:55:52.56ID:???
スリンキー(バネ)を垂直に垂らして上端を離すとしばらく下端が自由落下しない

物体は自由落下で同じ加速度を持つ等価原理に反してる様にも見える。
0219ご冗談でしょう?名無しさん
垢版 |
2021/08/10(火) 16:21:03.79ID:???
>>195の通り、
最下端のおもりを支えてるのは
最下端のバネ。もっと言えば近傍の微小な弾性体。

ここが開放されて縮み始めるまでは
重力と釣り合っているので落下しない。

ここに縮みが到達するまで時間がかかる。
フックの法則は、バネの質量なしなら一瞬で伝わる(高校物理でよくある設定)が
バネに質量ありなら各部が運動方程式に従うので一瞬で伝わることはない。

フックの法則の現実を正しく理解できているかどうかだけ。
0221ご冗談でしょう?名無しさん
垢版 |
2021/08/10(火) 16:36:17.33ID:???
ペットボトルを下に向けて水を捨てる場合、もっとも早く終わる角度は60度くらいか
計算はできないが
0223ご冗談でしょう?名無しさん
垢版 |
2021/08/10(火) 16:45:52.64ID:???
>>219
>縮みが到達するまで時間がかかる

最上端から順に縮む理由はなにか?
手を放すまではスリンキー(バネ)のどの部分も重力と釣り合っている

問題は、最下の部分も落下しながら全体のバネが縮まないのか? ということ
動画を見ても最下の部分の質量は十分ある
0224217
垢版 |
2021/08/10(火) 16:48:30.04ID:???
>>218
なるほど、そういう事か
>>219
これで正解じゃん
0225217
垢版 |
2021/08/10(火) 16:57:44.31ID:???
>>223
最上端を最初に離すからだな
離された最上端は上から引く力が消失して下降し始め
下降する事による慣性力は重力と釣り合うから下はそのまま
バネの力が無くなれば下も動く
0226ご冗談でしょう?名無しさん
垢版 |
2021/08/10(火) 17:04:18.25ID:???
>>225
> 下降する事による慣性力は重力と釣り合うから

バネ上端の下降加速度は重力加速度より大きいから重力と釣り合ってなんかいない
0227ご冗談でしょう?名無しさん
垢版 |
2021/08/10(火) 17:05:54.17ID:???
スリンキーを伝わる縦波の伝搬速度はどのくらいだろうか?
上端を放した影響は縦波の伝搬速度を超えることはないと思う
0228ご冗談でしょう?名無しさん
垢版 |
2021/08/10(火) 17:08:41.48ID:???
>>223
動画をよく見れば判るが
最上端を離せばスリンキー(バネ)どの部分も縮んで行くのが見える
しかし、それでも最下端は落下しない!
縮みが上から伝わるのが遅れるからなどという言う説は嘘だと判る。

>219 >バネに質量ありなら各部が運動方程式に従うので一瞬で伝わることはない。
大間違い。
0229ご冗談でしょう?名無しさん
垢版 |
2021/08/10(火) 17:19:03.76ID:???
>>228
くり返しだが
スリンキー(バネ)の最上端を離した力の変化は、直ぐ最下端に伝わり
バネ全体が縮んで行くのに、最下端が落下しない!

qqqの自演劇場は大失敗。
0243ご冗談でしょう?名無しさん
垢版 |
2021/08/10(火) 19:05:38.30ID:MMkKwyST
ベクトル解析すれば、時間差はでるだろう
0244ご冗談でしょう?名無しさん
垢版 |
2021/08/10(火) 19:12:06.40ID:???
>>235
スリンキーが垂直に伸びて静止した下端に働く重力と釣合う力の大きさは
スリンキーの上部がバネの縮む力で下に加速している反作用と等しい。
ということか
上部の加速度には重力加速度も加わってるが、簡単に証明できる人いる?
0245ご冗談でしょう?名無しさん
垢版 |
2021/08/10(火) 19:21:51.42ID:MMkKwyST
それが、E=mC2乗だよ、エネルギー保存則、エネルギーは多少分散するけど、エネルギーはあっている。だから、熱量など、簡単な数式で、エネルギーの数式で計算きるの
0246ご冗談でしょう?名無しさん
垢版 |
2021/08/10(火) 19:23:15.66ID:MMkKwyST
キーボード壊れていて、ごめんなさい。
0247ご冗談でしょう?名無しさん
垢版 |
2021/08/10(火) 19:30:28.63ID:MMkKwyST
エネルギー保存則とか。
0248ご冗談でしょう?名無しさん
垢版 |
2021/08/10(火) 19:36:48.39ID:MMkKwyST
タイムマシンとか、過去にいく単純な方法は、ベクトルの方向にいくスピードと、エネルギー量を、それ以上の
エネルギーで、ベクトルの逆に、ベクトルの方の逆に、逆行すればいいだけ。ベクトルの逆行は危ないから、それを避けて、星づたいに、
それ以上のスピードとエネルギー量で、来ているのが、今のプレアデス星人や天の川超銀河文明などだよ
0253ご冗談でしょう?名無しさん
垢版 |
2021/08/11(水) 02:21:50.59ID:???
やはり、フックの法則も波動も
本当は理解できていなかった人間ばかりだと示された。

バネは垂直方向でも横方向でもよい。
そのときバネの各部分の微小バネは、両隣の微小バネに引っ張られて力が釣り合っている。

伸ばしたバネの片方だけ手を放すと、最初に釣り合いがやぶれるのはその個所の微小バネだけだ。
ここが微小な時間をかけて縮み始めると、その隣の微小バネも引っ張られる力が弱くなり始めるので
釣り合いがやぶれて縮み始める。

この連鎖には必ず時間を要する。
瞬時に伝わるのはバネの質量が零の場合だけである。

弦でも液体でも固体でも波の伝わる速さは密度が小さいほど速い。
密度が零なら無限大の速さだ。
バネも同様、質量があればその縮みという波動が伝わるには時間がかかるのである。

フックの法則の現実版を知らなかった。
ただそれだけのこと。

バネは全長で負荷を支えているのではない。
負荷と接続されている箇所の微小なバネの伸び(ひずみ)によって支えているのである。
この伸びが変わらないなら、他の部分がどう伸縮しようが負荷は釣り合って動かないままなのだ。

f=kXのままで鵜呑みして覚えていると、バネの本質が理解できないままなのである。
0254ご冗談でしょう?名無しさん
垢版 |
2021/08/11(水) 02:38:36.79ID:???
>>253
いつもの花押がないけどくっくっくなんだろ?
やっぱ他とは全然違うな。量子力学と相対論と素粒子論だっけ?
俺も捨てることにするw
0256ご冗談でしょう?名無しさん
垢版 |
2021/08/11(水) 05:47:49.14ID:???
>>253
高校はおろか大学でも、今の物理・数学教育は全然ダメだよね。
結局何も理解できていない人たちばかり。。。
0257ご冗談でしょう?名無しさん
垢版 |
2021/08/11(水) 05:52:12.64ID:???
>>253
バネは全体が同時刻に伸縮するはず、これがフックの法則の誤解なんだよね。
現実はバネの伸縮は波動と同じで伝わるのに時間がかかるってこと。。。
0261ご冗談でしょう?名無しさん
垢版 |
2021/08/11(水) 07:40:11.58ID:???
そもそもスリンキーにフックの法則は適用できるものなのかね?
普通のバネとは違う振る舞いをしてそうな気がするんだが。
0263ご冗談でしょう?名無しさん
垢版 |
2021/08/11(水) 09:01:35.23ID:???
>>261
バネとしては「引張コイルばね(引きバネ)」という種類。初張力は
https://arxiv.org/pdf/1403.6809.pdf
の Fig.2 E を見る限り 0 ではない。傾きは十分直線とみなせるので、原点を適当にとればフックの法則は適用できる。

で、その特性を考慮した finite-collapse time というモデルで >>178 の波動方程式を解いた結果
https://arxiv.org/pdf/1208.4629.pdf
実験とよく合ってる。
0266ご冗談でしょう?名無しさん
垢版 |
2021/08/11(水) 09:53:49.09ID:???
>>246
その相手は、光速を超えるとか言ってる奴か

>>262 はスリンキーの上部が落下中でも最下部まで上部の変化が力学的に伝わってる
という主張だ

波動にも群速度と先端速度があるだろ
0269ご冗談でしょう?名無しさん
垢版 |
2021/08/11(水) 10:50:39.51ID:???
>>263

短いスリンキーならフックの法則が適用できるから、
長いスリンキーは、そのような短いスリンキーを連結したものとして
考えるわけだね。
0270ご冗談でしょう?名無しさん
垢版 |
2021/08/11(水) 11:45:02.66ID:???
ドローンの2Dシミュレータを作ろうと思っています
画像のように推力がT1とT2掛かっているとき、ドローン全体の推力と回転は以下の式であっていますか?
また、左右の長さが異なる場合、推力の計算はどうなるのでしょうか
推力=T1+T2
角加速度=T1の垂直成分による回転モーメント + T2の垂直成分による回転モーメント
https://i.imgur.com/k6kYFOw.png
0271ご冗談でしょう?名無しさん
垢版 |
2021/08/11(水) 12:39:14.63ID:???
>>270
重心の位置ベクトル r、 角速度ベクトル ω、質量 m、 重心周りの慣性モーメント I、重力加速度ベクトル g、ベクトル積 ×、重心から各推力の作用点へのベクトル rᵢ (i = 1, 2)、時間微分「' 」とすると

m r'' = T₁ + T₂ + m g
Iω' + ω×Iω = r₁×T₁ + r₂ ×T₂

2次元なのでベクトル積は行列式を取ってくれ。
0273ご冗談でしょう?名無しさん
垢版 |
2021/08/11(水) 16:58:17.48ID:9v7z7oGV
>>265
△自演上手
○(自分)誉め上手🤣
0275ご冗談でしょう?名無しさん
垢版 |
2021/08/11(水) 20:47:10.91ID:Va+/OKt5
>>258
重心の定義 MR=Σmr (M=Σm)
時間で2回微分すると
MR''=Σmr''=ΣF

バネの内力は微小バネmの間ですべて打ち消し合い
残るのは重力だけなので
MR''=ΣF=Σmg=Mg
よって
R''=g
となってスリンキーの重心は自由落下する。

ワシは国立大電気工学科卒なんだが
お前ら物理学科卒はまったく物理学を理解しておらんな。

ワシの足元にもはるかに及ばなくて
恥ずかしくないのか?

こういう基礎を理解できてない連中が
相対論やら量子論やら素粒子論やらほざいておるのだ。
いかにデタラメの虚構であるのか明らかであろう。

くっくっく
0278ご冗談でしょう?名無しさん
垢版 |
2021/08/11(水) 22:44:29.26ID:9v7z7oGV
>>275
何でデンキ板行かないの?
ハナシ合うでしょ?
0280P○ΘM
垢版 |
2021/08/12(木) 00:37:54.47ID:???
ばね定数kg/s^2
バネといえば
粘度kg/msの話聞きたい。くっく以外の人

(kg/s^2)/(m/s)速度あたりのばね定数って変形もできる粘度って、空気や水の浮力とかの関係とか

kg/s^2ってなんだろ
運動量kgm/sの質量は質量変動せず衝突の物理は相手との質量の比((物理的)負の値のみ(=質量正)、(物理的)正の値の方は一律×1(質量1/2で速度2倍とかはならない))、で今言った質量は変動しないから、
kg/s^2の質量も固定。1/s^2が変動することで、ばねの力が変動するんだろう。(「質量固定」で「質量比」かつ「1/s^2の変動」)
1/sは時間的直線変化だからkg質量は動かないまま時間的に変化(なにが?)
1/s^2は時間的加速変化(なにが?)

って1/sは留まらない力だった
ばねの力との質量比によって、留まらない?1/sはm/sのように慣性、さらにkgは運動を減弱、kg/sで「kgの質量比」と「1/s慣性的に」「kgの運動減弱で運動に抗う?」(=kgm/s:kg/s→○m:△…運動量を入力するとある一定距離で止まる(運動量を消しつつ両者止まる)(…てことはkg/sに弾性はないのか))
kg/s^2はその加速変化で弾性がある(=kgm/s:kg/s^2→○m:△/s…これはどういうこと?弾むんだろうけど、振動を表す?)

粘性kg/msは(kg/s^2)/(m/s)で運動量kgm/sの速度1個を完全無効化して、さらにkg(m/s)^2=J仕事を加えてやっと、振動が起こるがkg^2以上必要って感じなのかな
0281P○ΘM
垢版 |
2021/08/12(木) 00:51:11.44ID:???
kg1個でいいんだ。kgは消さないから

仕事で振動=粘度
運動量で振動=ばね

(粘度は)速度三乗でやっと加速運動的に動かせる
(バネは)仕事で加速運動させられる(あ、手でバネを縮めるとき、力じゃなくて仕事を加えてるのか。人間には腕の力で質量×加速度で加速的に縮めてると思ってるけど、その時の腕の力で腕を空中で動かしても等速運動しかしない((m/s)^2…m:sの比が1:1なので)。人間の認識で等速運動させる腕の運動力でバネを縮めるとき加速運動になるのか。等速運動が加速運動を生む錬金術に見えるけど、速度1階でなく速度2階の「加速度の減速」=等速、つまり釈迦力馬鹿力込めてるってことだね
0282P○ΘM
垢版 |
2021/08/12(木) 00:57:55.54ID:???
なんか自分で考えて納得しちゃった

飛行機とかって粘度とか含めると揚力、複雑なのかな
飛行機はやっぱいいや
虫の羽って空気の粘度で、羽を振動させて、どうなって飛んでる?
羽板の複山振動でどうやって自重を消してるんだろ
振動させると空気の浮力に溶ける?
それとも複山運動で羽が先端方向に直進力を得る?
0283P○ΘM
垢版 |
2021/08/12(木) 00:58:45.92ID:???
↑空気の粘度に溶ける(空気に羽が一体化)
0284ご冗談でしょう?名無しさん
垢版 |
2021/08/12(木) 01:49:34.98ID:nxgpCcdE
>>279
寄せやい、あそこでハナシが合うたことなんか、🤓🤡🤢(´Д`🌀)
ただの一度もないわい!🤣☺
0285ご冗談でしょう?名無しさん
垢版 |
2021/08/12(木) 01:53:27.82ID:nxgpCcdE
>>275
スリンキー、スリンキー、ぐるぐる巻き🌀の秘密わね、
スリンキー、スリンキー、渦巻き状🌀の秘密わね、

…教えてあげないよ、じゃん!
0288ご冗談でしょう?名無しさん
垢版 |
2021/08/12(木) 03:43:11.11ID:???
スリンキーの材料が鋼鉄として縦波の音速(伝搬速度)は約5.9km/s
バネのらせん状に縦波が伝搬するとすればバネの落下速度よりけた違いに速い。
0289ご冗談でしょう?名無しさん
垢版 |
2021/08/12(木) 06:02:39.55ID:???
>>275
その当たり前の自由落下を、お前の馬鹿理論じゃ説明できないから馬鹿にされてんだよ。

「ここが開放されて縮み始めるまでは
重力と釣り合っているので落下しない。」(>>219)


>>288
引張コイルバネは捩じり応力を発生させるバネなので横波。
0290ご冗談でしょう?名無しさん
垢版 |
2021/08/12(木) 06:50:29.32ID:???
>>275
さすがだな
実は、自分もバネはどこも同時に伸縮するものと思い込んでたわw
手をはなしたほうから縮んでいくんだな、確かにバネも波だ
いやーこれは久々に感動したわ
0291ご冗談でしょう?名無しさん
垢版 |
2021/08/12(木) 07:01:15.63ID:???
>>275
電気は実用物理だから、純粋な物理卒より上位カーストなのは否めないw
バネも自分みたいに誤解してた奴ばっかみたいだしww
0292ご冗談でしょう?名無しさん
垢版 |
2021/08/12(木) 07:10:46.33ID:???
10 時間で自演擁護レス 2 つだけ、って相当弱ってるな。
手を離した瞬間から下端 -- 重心間距離が縮む事説明出来なくて困ってんだろ
0293ご冗談でしょう?名無しさん
垢版 |
2021/08/12(木) 07:15:01.69ID:???
結局999の主義主張は何も読み取れなかったな
あいつの主張している独自物理学の実体は俺には全く読み取れなかった
0294ご冗談でしょう?名無しさん
垢版 |
2021/08/12(木) 09:29:40.65ID:???
>>289
>引張コイルバネは捩じり応力を発生させるバネなので横波。
横波だけだと勘違いしてるのか

現実のスリンキーのバネは鋼鉄などの固体物質だから高速で螺旋方向に進む
縦波が有るのは当たり前。
巻き線構造による遅い波動の伝番が、エネルギー移動の大部分とも解釈できる。
0295ご冗談でしょう?名無しさん
垢版 |
2021/08/12(木) 09:46:29.95ID:???
>>294
アホ、5.9km/s なんて体積弾性率と密度から求めた音速。固体の振動は複合的だが、引張コイルばねの伸張モードには関係ない。
0296ご冗談でしょう?名無しさん
垢版 |
2021/08/12(木) 09:55:57.88ID:???
>>222
>垂直。グルグル回して渦を作り、空気が真ん中を通るようにすると連続的に排水される。
へえ、おもしろい
工場などでやっているのだろうな
0299ご冗談でしょう?名無しさん
垢版 |
2021/08/12(木) 10:21:18.64ID:???
バネの伸張モードなどより速く、上端で離した力の変化が下端まで伝搬してる。
スリンキーの動画でも確認できる下部も少しづつ縮んでる理由だ。

上端の変化が下端に伝わってないからなどという説は眉唾ものだと判るが
下端に物を付けても明らかに落下しないように見える、説明は簡単でないだろ。
0305ご冗談でしょう?名無しさん
垢版 |
2021/08/12(木) 11:31:50.37ID:???
>>304
検索しかできないアホ脳は変わらんな
ばね定数しか出てこないモデルだろが

上端の変化が下端に速く伝わってもそれで下端が落下運動するとは限らない。
0307ご冗談でしょう?名無しさん
垢版 |
2021/08/12(木) 11:50:17.44ID:???
>>306
検索しかできないアホ脳の
他人のふんどしで正当化するアホ論理がミエミエ。

そのモデルで実験が再現できたとしても、一つのモデル例にすぎない。
0308ご冗談でしょう?名無しさん
垢版 |
2021/08/12(木) 11:56:20.31ID:???
モデルで実験が説明できるのは、モデルを再現するように実験系をセットしたからだとでもいいたいのか
0310ご冗談でしょう?名無しさん
垢版 |
2021/08/12(木) 12:00:21.98ID:???
>>307
ほう、じゃ、お前の馬鹿理論で立式して説明してみろ。不確定性原理からシュレディンガー方程式の時みたいに逃げんなよ
0311ご冗談でしょう?名無しさん
垢版 |
2021/08/12(木) 12:33:47.97ID:???
>>310
お前の頭では出来ないことを白状したのか

簡単な思考時実験だよ、自由空間で位置の小変化でバネの力が同じとする
 |――――――|  バネが伸びた初期状態
  |――――|   のように縮んでいく
ここまでは解るよな。
0313ご冗談でしょう?名無しさん
垢版 |
2021/08/12(木) 12:45:32.21ID:???
>>312
式が書いてないと理解も出来ないアホだったか

両端|の質量を仮にm、バネの力をf とすれば 縮む加速度は a = f/m
ここまでは解るよな
0318ご冗談でしょう?名無しさん
垢版 |
2021/08/12(木) 13:04:59.32ID:???
>>311,313
から、頭の良いスレ読者には思考実験の推測がつくだろう。
重力加速度の大きさg a = g とすれば
 |――――――| ←g バネが伸びた初期状態
 |――――|   ←g のように縮んでいく
下端は動かない。 めでたしめでたし
0319ご冗談でしょう?名無しさん
垢版 |
2021/08/12(木) 13:36:59.23ID:wHr0yRXy
よろしくお願いいたします。
中学受験の参考書に出てきた問題です。つまり小学生用のものなので、小学生レベルの語彙で教えてください。

ttp://imepic.jp/20210812/489600
この図をごらんください。
これは、10グラムの重さを吊すと1cmのびるバネAとBを、このように重りをつけて天上から吊した図です。
このようにすると、バネAとバネBの、「バネののび」は何センチでしょう?という問題です。
答えは、
バネA:10cm (解説=バネAにかかる力は100g、1×(100÷10)=10)
バネB:10cm (解説=バネBにかかる力は50g、2×(50÷10)=10)

このように書いてあります。バネBの部分がさっぱり分かりません。バネBには50gしか力がかかっていないから5cmしか
のびないと思うんですが、なぜ10cmなのか、そして、解説にある2×という部分がどういう意味なのか。

どなたか、これを詳しく解説していただけないでしょうか?
0321319
垢版 |
2021/08/12(木) 13:55:02.76ID:???
>>320
物理板ははじめてです。
子の夏休み特訓で、すごく焦っています。本当によろしくお願いいたします。
0324ご冗談でしょう?名無しさん
垢版 |
2021/08/12(木) 14:02:31.74ID:???
>>318
まだいたんか。

> a = g とすれば
動画の上端の落下加速度は、自由落下してる重心より遥かに大きい。笑かすなっつうの、朝鮮ハゲ
0325319
垢版 |
2021/08/12(木) 14:06:08.26ID:???
>>322
以下が問題文です。
「自然長10cm、10gで1cmのびるばねAとBがあります。
このばねAとBを直列につなぎ、その間に50gのおもりをはさんで、
ばねBに50gのおもりをつるしました。このときのばねAとBののびは
それぞれ何cmですか。」
答えの解説は、319の通りです。
>>323
別の質問機会のとき算数板で、参考書の図をちょくせつネットにあげるのは
法的に問題だと言われたのでできません。ごめんなさい。
0328ご冗談でしょう?名無しさん
垢版 |
2021/08/12(木) 14:17:02.07ID:???
ばねBの自然長がAの自然長の2倍という条件がどこかにあって、それを見落としてるのなら答えが正しい。
0329ご冗談でしょう?名無しさん
垢版 |
2021/08/12(木) 14:31:23.29ID:???
みなさま、ありがとうございました。
本気で誤植を疑って問い合わせてみることにします。

図を見るかぎりでは、頭のいい人が見ても、
「Aには100gの力がかかっていて、Bには50gの力がかかっている」
としか解釈できないんですね。

その部分だけ太鼓判をおしていただけるだけでもたいへん助かりました。
お騒がせしました。
0330ご冗談でしょう?名無しさん
垢版 |
2021/08/12(木) 19:13:00.72ID:???
>>319
たぶんその2×というのは、Bは両側から引っ張られてるからーとかいうイメージなんじゃないですかね

でもその考えは他の方がいうように間違ってます

Aは片方だけ引っ張ってるように見えますけど、天井がもう片方から引っ張ってるので結局Bのように両側から引っ張ってますからね
0331ご冗談でしょう?名無しさん
垢版 |
2021/08/12(木) 21:02:49.73ID:Gh1hXqVf
恒星 M と惑星 m のみが空間に存在するとする。原点に M があるとし、 m の運動を考える。

運動の第2法則:
F = m * (ax, ay, az)

万有引力の法則:
F = - (G * M * m / r^3) * (rx, ry, rz)

より、

m * (ax, ay, az) = (G * M * m / r^3) * (rx, ry, rz)
(ax, ay, az) = - (G * M / r^3) * (rx, ry, rz)

が成り立つ。

よって、 (ax, ay, az) と (rx, ry, rz) は平行。

d/dt ((rx, ry, rz) × (vx, vy, vz))
=
(d/dt (rx, ry, rz)) × (vx, vy, vz) + (rx, ry, rz) × (d/dt (vx, vy, vz))
=
(vx, vy, vz) × (vx, vy, vz) + (rx, ry, rz) × (ax, ay, az)
=
(0, 0, 0)

よって、 (hx, hy, hz) をコンスタントベクトルとして、

(rx, ry, rz) × (vx, vy, vz) = (hx, hy, hz)

と書ける。

(hx, hy, hz) ≠ (0, 0, 0) であると仮定してよい。

ここで、質問です。 (hx, hy, hz) ≠ (0, 0, 0) であると仮定してよいのは、もし、 (hx, hy, hz) = (0, 0, 0) であるとすると、

(rx(0), ry(0), rz(0)) と (vx(0), vy(0), vz(0)) が平行であることになってしまって、 恒星と惑星が衝突してしまうというつまらない結果になってしまうからですか?
0332ご冗談でしょう?名無しさん
垢版 |
2021/08/12(木) 21:28:08.36ID:???
>>331
「仮定してよい」ってのは仮定してもしなくてもどっちでも好きにしたらよいってことだから、親切心じゃないですかね?
真意は出題者に聞いてください。
0333ご冗談でしょう?名無しさん
垢版 |
2021/08/12(木) 22:04:57.61ID:???
>>331
無限に離れていく場合もあるから衝突とは限らない
0 でないと仮定しても後でパラメータ設定で 0 も含めることができるから問題ない
0337ご冗談でしょう?名無しさん
垢版 |
2021/08/12(木) 23:08:52.17ID:???
相間、量間、電間の自傷電気工学科卒のqqq!

今回単なる連成バネ問題すら解けない馬鹿っぷりを披露。ニュー間の称号を獲得。次は熱力学か流体力学か
0340ご冗談でしょう?名無しさん
垢版 |
2021/08/13(金) 00:03:26.67ID:???
>>252
スロー再生で見たら、確かにばねの底辺付近は縮んでないね。
だからばね張力と重力はつりあったままで底辺付近は落下しないのか。
これは勉強になった。
0341ご冗談でしょう?名無しさん
垢版 |
2021/08/13(金) 00:06:41.73ID:???
>>335
いつもボロクソに負けてるよね。
今回のスリンキーもくっくっく氏一派(?)がいなければ永遠に謎のままだった。
0342ご冗談でしょう?名無しさん
垢版 |
2021/08/13(金) 00:13:20.46ID:???
中学生に聞かれたらどう答えるか、なかなか難しそう。

磁石は、並列に並べても、直列につなげても磁力が強くなる。
バネは、並列に並べれば力が強くなるが、直列につなげると弱くなる。
0345ご冗談でしょう?名無しさん
垢版 |
2021/08/13(金) 01:41:43.42ID:kQi2Fim9
>>341
「負けてる」の意味、根本的に間違えてるのがいるな☺🤓🤡🤢(´Д`●)
0349ご冗談でしょう?名無しさん
垢版 |
2021/08/13(金) 05:59:52.91ID:???
>>344
それはくっくっくが中学生レベルなのを自覚していて中学生にも分かるレベルの教え方で専門家が教えてくれないと荒らしちゃうぞ!って宣言してると読み解くべき
0351ご冗談でしょう?名無しさん
垢版 |
2021/08/13(金) 12:23:00.02ID:???
熱い液体を素早く冷ますにはどちらがよりよいのでしょうか(ラーメンのスープやコーヒーをイメージして頂ければと思います)

(1)かき混ぜて、空気に触れる面が極力熱い状態を保つ
(2)かき混ぜず、空気に触れるている部分をどんどん冷やしてから混ぜる
0353ご冗談でしょう?名無しさん
垢版 |
2021/08/13(金) 18:37:38.24ID:3l2jGzrE
>>351
ちょっと待ちなさいよ!🤓🤡🤢(´ω`🌀)
条件がさっぱりわからないわ!
撹拌の間隔とか速度とか、そもそも容器の大きさとか形状とか、
高効率の放熱板でもつっこんでおくのか、とか扇風機の風でも
当て続けるのか、その風量は…
0354ご冗談でしょう?名無しさん
垢版 |
2021/08/13(金) 19:07:44.58ID:???
>>351
かき混ぜて温度が均一になった後の温度だとして、空気への熱伝達は空気との温度差が大きいほど速いから、(1)
0356ご冗談でしょう?名無しさん
垢版 |
2021/08/13(金) 20:07:13.19ID:3l2jGzrE
>>355
おかしな人違いすなヴォケかす!🤓🤡🤢(´ω`☺)
0358ご冗談でしょう?名無しさん
垢版 |
2021/08/13(金) 20:13:31.52ID:3l2jGzrE
>>351
そもそもそ、効率を最大限にまで高めなければアカンのでっから〜、🤓🤡🤢(´ω`🙆)
とにかくゆ〜っくり作業を行うこってす。風なんか当てるの厳禁!
かき混ぜるのも、ゆ〜〜〜っくりゆ〜〜〜っくり…寒くなるまで
時間かけましょ!
0359ご冗談でしょう?名無しさん
垢版 |
2021/08/13(金) 20:17:46.65ID:3l2jGzrE
>>353
その昔、ヒートパイプっての流行ったな。
あんなの突っ込んどいて扇風機当てとけばぁ?
いくつかパターン作って実験すればいいのに。
0362ご冗談でしょう?名無しさん
垢版 |
2021/08/13(金) 21:52:46.75ID:/CvW0r6s
>>361
バカ野郎!そのそこの浅い銭儲け詐欺師の🤓🤡🤢(´Д`🔴)
ネタなんか転載すんな!
0365ご冗談でしょう?名無しさん
垢版 |
2021/08/14(土) 02:49:40.59ID:???
ところで、みなさんはコロナの遺伝子ワクチン打ちますか?
なんかヤバそうですよね。

イスラエルやアメリカやイギリス見てると皮肉にも
ワクチン打ちまくってる国ほど感染爆発しています。ワクチン後進国ほどそうなっていません。
3回目打つとか、なんだか中毒患者の様相を呈してきました。これから何回も打つのでしょうか。

自分の細胞に異物遺伝子を取り込ませて
細胞内で異物のスパイクたんぱくを作らせ放出させて
免疫系と接触させて免疫を獲得する。

要は細胞がウイルスに感染してウイルスが増殖する仕組みと
そっくりなやり方ですが、人為的にこんなことをやっても果たして大丈夫なんでしょうか。
あとあと、免疫異常や全身発がんや予測不能な疾患になったりしないのでしょうか。

打った直後に死んだ人や重篤化した人も多数いるし、その副作用(副反応)の強さは
インフルエンザワクチンの比ではないそうです。
何より、後遺症が未知数なので怖いです。

みなさんはもう打ちましたか?
それなら、ワクチン反対派の意見は見ないほうがいいでしょう。
恐ろしいことが書かれていますから。
0366ご冗談でしょう?名無しさん
垢版 |
2021/08/14(土) 02:58:41.63ID:???
ちなみに
本当に自然科学を理解している人間ほど
この全世界で絶賛人体実験中の遺伝子ワクチンは打たないようです。
高見の見物だそうです。大勢の学者や医師が反対しています。

相対性理論や量子論や素粒子論にコロっと騙されている人間は
何の疑いもなしに「重症化が防げている」という大本営の念仏を信じて
きっともう打ってしまっているのでしょうね。

インターネッツがあるのに、自分で調べもせずこんなワクチンを打つとは
何のために脳みそがあるのか意味不明です。

では。
0367ご冗談でしょう?名無しさん
垢版 |
2021/08/14(土) 06:40:24.67ID:???
冷静になるとエネルギーを計算に変換できるって言ってるからコンピュータって意味わからんな
しかも量子コンピュータになるとエネルギー無しで計算できますになるからもっと意味わからん、その計算はどっから来たんだよ
0368太上天君
垢版 |
2021/08/14(土) 06:49:29.55ID:Y7wZpeKs
>>365
>人為的にこんなことをやっても果たして大丈夫なんでしょうか。

うまくいけばワクチン効果で超能力者になれるぞwww
0369ご冗談でしょう?名無しさん
垢版 |
2021/08/14(土) 09:23:53.97ID:???
超能力者とは、元の武漢ウィルスに感染してそのままコピーせず
より感染力毒性が強い変異ウィルスを新たに作り出す人のことだ。
0371ご冗談でしょう?名無しさん
垢版 |
2021/08/14(土) 10:14:43.92ID:???
バカ、反ワクチン、ネトウヨ
なぜか全部セットになってることが多いよね
トランプのせいで可視化されちゃった
0375ご冗談でしょう?名無しさん
垢版 |
2021/08/14(土) 13:59:20.92ID:IMEXQlhJ
mr''=mr(φ')^2 (sinθ)^2 -mgcosθ
という運動方程式があったとします。微分は'で表してます。これの両辺をr''で偏微分するとm=0となってしまう気がします(もしくは1=0)。どこが間違ってるのでしょうか。初歩的なことですみません。お願いします。
0376ご冗談でしょう?名無しさん
垢版 |
2021/08/14(土) 14:52:54.25ID:???
ma = F
の両辺をFで偏微分したら 0 = 1 になってしまいますね。
つまり無意味な偏微分をすると無意味な結果が出るということです。
0378ご冗談でしょう?名無しさん
垢版 |
2021/08/14(土) 15:00:07.72ID:peq/0Wmq
>>376
>>377
レスありがとうございます。なるほど。
ラグランジュ方程式をたてるときは連鎖律を考えなくていいのは何故なのでしょうか。
0380ご冗談でしょう?名無しさん
垢版 |
2021/08/14(土) 15:21:54.61ID:???
>>375

どういう条件を課すかによる。
普通は方程式が与えられれば、r'', r, φ',θは独立な変数とはならず、両辺を r'' で偏微分すると、
m = m (∂r/∂r'')(φ')^2 (sinθ)^2 + 2mrφ' (∂φ'/∂r'') (sinθ)^2 + 2mr(φ')^2 (sinθ)(cosθ)(∂θ/dr'') + mg (sinθ)(∂θ/dr'')

それでも r'', r, φ',θが独立だという条件を課すとしたら、m=0 のときにしか解はない。
更に m≠0 という条件まで課したら、方程式にもう解はない。
0383ご冗談でしょう?名無しさん
垢版 |
2021/08/14(土) 15:30:43.56ID:???
解析力学の偏微分でpとqを独立変数として扱って良いのはなぜか?
というFAQと同じだね。
そのFAQには明確な答えは誰も示せていないようだがw
0384ご冗談でしょう?名無しさん
垢版 |
2021/08/14(土) 15:57:43.41ID:peq/0Wmq
>>382
レスありがとうございます。
ラグランジアンが
1/2mx'^2-mgxとしたときに
x'で微分した時に連鎖率を考えると∂x'/∂xを考えてないのはいいのでしょうか。

>>383
レスありがとうございます。
まさにそのことが疑問です

少し調べてみたのですがオイラーラグランジュ方程式の導出のときで
作用の停留点を求めるときの最初にqを変化させます。
最初にqをq+δqにしたときはq'をq'+δq'にするみたいです。(つまりこの時点では独立変数ではないということですか?)
そこから計算をすると最終的にオイラーラグランジュ方程式はq'を独立変数としてみなした微分として書けるということでいいのでしょうか。
つまりオイラーラグランジュ方程式でのみqとq'を独立変数として扱えるということですか。

このサイトのオイラーラグランジュ方程式の導出を参考にしました。
https://dora.bk.tsukuba.ac.jp/~takeuchi/?解析力学%2Fラグランジアン
0386ご冗談でしょう?名無しさん
垢版 |
2021/08/14(土) 16:52:18.01ID:???
>>384
そうやって好き勝手に書き散らかした疑問文を
自分で答えるのが勉強というものです。
疑問すら持たない大多数の人々よりはあなたは賢いので
かならず答えを見つけられるはずです。
0387ご冗談でしょう?名無しさん
垢版 |
2021/08/14(土) 17:17:31.21ID:???
オイラーラグランジュ方程式の導出にqとq'の関係を使ってるじゃん
中間の操作全部で使う意味はない
0389ご冗談でしょう?名無しさん
垢版 |
2021/08/14(土) 17:24:15.47ID:???
ラグランジアンの段階だと経路はどうとってもいいから独立変数

オイラーラグランジュの段階だともう経路が確定しているから独立じゃない
0390ご冗談でしょう?名無しさん
垢版 |
2021/08/14(土) 17:26:00.60ID:???
>>389
まだ俺もはっきりと理解してないんだけど
ラグランジアンの空間とオイラーラグランジュの空間って等価ではないの?
0391ご冗談でしょう?名無しさん
垢版 |
2021/08/14(土) 17:56:50.38ID:peq/0Wmq
>>389
そういうことかー
なんとなくですがわかった気がします
ありがとうございます!!
0393ご冗談でしょう?名無しさん
垢版 |
2021/08/14(土) 18:16:41.74ID:???
>>390
ラグランジアンの空間では、(q,q’)という空間内の点は独立で自由にとれますよね

でオイラーラグランジュ方程式があったとしても、まだ(q,q’)という点は独立にとれます

(q,q’)がオイラーラグランジュ方程式を満たすという条件を課して始めて独立ではなくなります
0394ご冗談でしょう?名無しさん
垢版 |
2021/08/14(土) 18:17:22.78ID:???
>>390
ラグランジアンを書いてる時点では座標と速度はバラバラにして良い

真の効果を発揮するのは量子力学にいくときで実際経路積分で量子力学をつくるときはこのバラバラなものを全て足し合わせる

オイラーラグランジュはいろんな経路のうち古典的な経路を見つける式でこれによって座標と速度が真の意味でつながる
0396ご冗談でしょう?名無しさん
垢版 |
2021/08/14(土) 18:43:04.29ID:???
>>394
ラグランジアンが点(の全体)でオイラーラグランジュが線なのは分かったんだけど
ラグランジアンに含まれる点の集合だとみなすとオイラーラグランジュの満たす空間とラグランジアンが入ってる空間って等価になりそうだけど違うっぽいから聞いてる
ラグランジアンで記述される多様体とオイラーラグランジュの入ってる多様体ってイコールではないの?
0399太上天君
垢版 |
2021/08/14(土) 20:04:56.43ID:ERizikc8
ラグランジアンはオイラーオイラー

ラグランジアンはオイラーオイラー

ラグランジアンはオイラーオイラー
0402ご冗談でしょう?名無しさん
垢版 |
2021/08/15(日) 00:18:47.85ID:???
ブラックホールについて質問です
もしブラックホールに人が落下するとその落下中の観測者の時間が無限に遅くなってしまうので最終的にブラックホール表面に張り付いているように見えるそうですが
これって落下中の観測者から宇宙を見るとものすごいスピードで時間が経過してて最終的に宇宙の終わりが見えてしまうということですか?
0403ご冗談でしょう?名無しさん
垢版 |
2021/08/15(日) 00:37:12.67ID:???
>>402
そうはならない。落下の仕方にもよるが、自由落下なら宇宙からの光はむしろ赤方偏移して見える。
つまり宇宙の時間はゆっくり経過していくように見える。
0407ご冗談でしょう?名無しさん
垢版 |
2021/08/15(日) 01:16:47.01ID:???
>>403
と言うことはつまりブラックホールに落下してる観測者から見ると宇宙が急速に膨張してるように見えるということですか?
0410ご冗談でしょう?名無しさん
垢版 |
2021/08/15(日) 04:50:58.61ID:hoWuloBl
>>384

【 アホがよくやるエネルギー偏微分 】

図のように高さhの位置に密度ρの直方体abcがある。
これの位置エネルギーはΦ=ρ・a・b・c・g・hである。

これをhで偏微分するとー∂Φ/∂h=−ρ・a・b・c・gとなって重力となる。
ところがaで偏微分してもー∂Φ/∂a=−ρ・b・c・g・hとなって
意味はないが値は一応出てくる。bとcも同じ。

こういうアホなことをやりまくってるのが理論物理学なので注意しろよ。
存在しない力をあたかも存在するように見せかけてるだけだからな。

入試問題で
そのモデルでは存在しない横方向の力を
エネルギー偏微分で存在するかのように扱った世紀の愚問があったからな。

平行板コンデンサーで誘電体を横方向に引き込む力だ。
そんなものはそのモデルでは最初から存在しておらんのだが
図と同じように適当に偏微分したら値は出てくると言うデタラメ問題だ。

まことに
相対論と量子論と素粒子論を信じておるサルどもは
アホザルしかおらん。物理学科のサルどもは虚学しか知らん。

ワクチン何回も打って
せいぜいセルフ生体実験しとけ。
ワシは絶対に打たんわ。当たり前だ。

くっくっく
0411ご冗談でしょう?名無しさん
垢版 |
2021/08/15(日) 05:03:22.47ID:hoWuloBl
https://detail.chiebukuro.yahoo.co.jp/qa/question_detail/q11177522216
>そもそもどうしてx軸方向に力が働くのかすら分かりません
>電極間の上下の電界の向きなら働きそうだと思いました

この質問者がまったくもって正しい。
平行平板コンデンサーは真電荷も分極電荷も均一に分布して
それぞれが垂直方向にしか電界を作らないものとして
モデル化しているので、誘電体を横方向に引き込む力など
最初から存在せず論外なのである。

実際には
誘電体には不均一に分極電荷が現れ、コンデンサー側の真電荷も
不均一に帯電するので解析学的に解くのはまったく不可能である。

そもそもモデルがまったく別物になるので
近似して出せるという考え方が論理的に大間違いなのである。
近似もクソもないのだ。近似できるという発想が論理的にアウトなのである。

くっくっく
0412ご冗談でしょう?名無しさん
垢版 |
2021/08/15(日) 05:08:26.67ID:hoWuloBl
>>384
そういうことを考えないといけない時点で
ハミルトニアンもラグランジアンも
クソの邪道だと気付け。

じゃあな。
アホザルども

くっくっく
0414ご冗談でしょう?名無しさん
垢版 |
2021/08/15(日) 05:18:28.16ID:hoWuloBl
>>386
ああ、忘れておったわ。

お前みたいなアホザルどもがやっておるのは
ハミルトニアンやラグランジアンを主体とする本末転倒な邪道であって
それはもはや物理学ではないということだ。

物理学から逸脱した数学もどきの辻褄合わせごっこなんだよ。
だから平行平板コンデンサーの件みたいな恥ずかしい大間違いをやらかして
気付きもしないアホザルばっかなんだよバーーーーーカ

くっくっく
0417ご冗談でしょう?名無しさん
垢版 |
2021/08/15(日) 06:23:10.68ID:???
ぶっちゃけ解析力学なら教科書書ける自信あるけどニュートン力学は教科書書けるほどわかってない
0418ご冗談でしょう?名無しさん
垢版 |
2021/08/15(日) 07:03:54.65ID:???
>>410
これ、いい例えですね
どんな変数でもたいてい偏微分できるから
何でもアリってことですか、リロン物理学はw
0420ご冗談でしょう?名無しさん
垢版 |
2021/08/15(日) 07:11:34.63ID:???
>>402
自由落下時、青方偏移するのはシュワルツシルト面を通過するときに水平方向だった領域だけ。
視界は後方の一点に収縮するが、円周部分が無限に青方偏移、内側は無限に赤方偏移する。
https://arxiv.org/pdf/1511.06025.pdf

宇宙の終わりも見えない。John Rennie の回答↓
https://physics.stackexchange.com/questions/82678/
0421ご冗談でしょう?名無しさん
垢版 |
2021/08/15(日) 07:25:49.03ID:???
>>410
ふつうρ=1/abcとおくんだけどね
あなたはバカなので気づかないようですが

エネルギー保存を満たさなくなるのでww
0422ご冗談でしょう?名無しさん
垢版 |
2021/08/15(日) 07:57:51.23ID:T11bsOZv
M1です。量子論の質問です。

当然シュレーディンガー方程式は量子論の方程式ですが、
ヒルベルト空間内での状態ベクトルの時間発展を決定論的に記述している、
という意味では古典的な運動方程式であるとも言えませんか?
(実際、場の理論ではディラック方程式によって、可能なあらゆる経路のうちから古典的な経路を指定されますので)
そこで疑問なのですが、シュレーディンガー方程式を最小作用の原理から出すにあたり、
ヒルベルト空間内の状態ベクトル自体を力学変数とすることはできますか?

psi(t, r) = < r | psi(t) >を場と思って、これを力学変数とするラグランジアンから
シュレーディンガー方程式を導くことはできるのですが、
(ディラック方程式をラグランジアンから導くのと全く同じやり方)
| psi(t) >じたいを力学変数と思って定式化することは可能ですかね?
そもそもここまでの考え方が変ですか?
0424ご冗談でしょう?名無しさん
垢版 |
2021/08/15(日) 08:38:37.65ID:???
>>422
言ってる意味がよくわからんけど時間に依存するユニタリ変換をU(t)と書く時
|x'>=U(t)|x>とシュレディンガー方程式は等価だけどそういうことではなくて?
0425ご冗談でしょう?名無しさん
垢版 |
2021/08/15(日) 09:32:49.20ID:???
>>419
>ワクチンは俺も打ちませんw

重症化しても病院が満杯で入院できず、運よく回復しても味覚・嗅覚障害、倦怠感
の後遺症が治らなくても自己責任だからね。
0426ご冗談でしょう?名無しさん
垢版 |
2021/08/15(日) 09:34:55.06ID:???
理屈の上では非活性ワクチンよりもmRNAワクチンの方が安全なのは理解できるけど
治験が十分ではない緊急承認のmRNAを体内に取り込むのは怖いってのは十分科学的な態度だけどな
0428ご冗談でしょう?名無しさん
垢版 |
2021/08/15(日) 12:24:59.48ID:WW7QxHsj
以下の会話におかしなところはありませんか?


テトラ「不思議です!ボールの運動を調べるとき、x成分とy成分に分けてニュートンの運動方程式を立てましたけれど、あたしたちが決める
座標軸の方向なんて、宇宙は知りませんよね?人間が決めた座標軸なのに、どうして計算がうまくいくんでしょうか……」

僕「テトラちゃんはすごいよ。僕が物理で質点の運動を勉強したとき、そんな疑問なんて思いつかなかったなあ……結論からいうと、
x軸とy軸をどんな向きに定めても、成分ごとにニュートンの運動方程式が成り立つというのは、僕たちの宇宙の性質だね。この性質は実験
で確かめられている。」

テトラ「宇宙がそのような性質を持っているというのは、すごいことじゃありませんか?」

僕「すごいことだよ!成分ごとにニュートンの運動方程式が成り立つから、ベクトルで考えてもかまわないといえる」
0433ご冗談でしょう?名無しさん
垢版 |
2021/08/15(日) 13:00:57.67ID:???
数学ガールの著者の結城浩は何故か専門家扱い受けてるけど学部中退だし、その学部が理系学部だったのかどうかもはっきりしないから
0436ご冗談でしょう?名無しさん
垢版 |
2021/08/15(日) 13:15:38.62ID:???
反ワクチン君は他人とに繋がりが薄いから感染症をリアルに捉えられないし、経験を補う思考力も持ち合わせていないことはこのスレで散見される人を見れば明らか。
0437ご冗談でしょう?名無しさん
垢版 |
2021/08/15(日) 13:16:32.19ID:???
逆に考えると何で光は宇宙の速度の上限で動いてるんだよ
めちゃくちゃ早いけどそれ以上に加速できて、そうやって加速することでようやく宇宙の速度の上限値に到達するでは駄目だったのか?
0438ご冗談でしょう?名無しさん
垢版 |
2021/08/15(日) 13:22:44.21ID:T11bsOZv
>>424
そういうことではありませんね
0440ご冗談でしょう?名無しさん
垢版 |
2021/08/15(日) 13:40:51.00ID:???
>>439
言葉は知ってるけどそれを定式化したものを見たことがないんだよね
同様にパウリの排他原理は相対論を考慮すると導出できるともよく耳にするけど導出できたことがない
0442ご冗談でしょう?名無しさん
垢版 |
2021/08/15(日) 14:06:56.40ID:???
>>440
おまえのいう「定式化する」ってのは、具体的にどういう作業を指す言葉なの?
p = mγvより、
p≠0, m=0 なら γ=∞ つまり v=c 以外には成り立たないのは明らかと思うが。
0445ご冗談でしょう?名無しさん
垢版 |
2021/08/15(日) 14:29:19.65ID:???
・合同会議で、厚労省担当者は、自宅療養での死者数について「把握できていないケースがたくさんあり、網羅的には把握していない」と述べた。自治体側による集計システム「HER―SYS(ハーシス)」への入力作業が遅れている上に、急増する感染者への対応に自治体が追われているため、「さらに遅れるという傾向にある」という。
・田村憲久厚労相が5日の閉会中審査で「肺炎症状のある人は、原則入院」と答弁した点について、方針通りに患者が入院できているのかと問われると、厚労省は「『そうだ』と答えられる材料を持ち合わせていない」とも述べた。
0446ご冗談でしょう?名無しさん
垢版 |
2021/08/15(日) 14:34:23.15ID:???
イスラエルの報道見れば、ワクチン2回打っても普通に重症化している。
だから3回目を打つ。
本当に効果切れだからそうなるのか、そもそも最初から効き目薄いからそうなるのか、
非常に怪しい。

まともな理系、自然科学を理解している人間なら
もうこの時点でこんなものは打たない。
0447ご冗談でしょう?名無しさん
垢版 |
2021/08/15(日) 14:37:54.93ID:???
自宅療養での死者数について、ワクチン接種者がどれだけいるのか発表しない大本営。
というか、カウントできていないのだから発表できない。
そもそも、すべての死者を検査しているのか。

何もかも滅茶苦茶で、完全に世紀末化してきている。
0448ご冗談でしょう?名無しさん
垢版 |
2021/08/15(日) 14:43:10.87ID:???
>>426
細胞に遺伝子取り込ませることの何が安全なのか。
普通にウイルスに感染して細胞内で増殖するのも安全ではない。
その過程で発がんしているかもしれない。

免疫細胞が発がんした細胞を増殖する前に食い殺してくれているから
数年後にがんとして発見されることがないだけ。
0449ご冗談でしょう?名無しさん
垢版 |
2021/08/15(日) 14:47:40.43ID:???
>>436
ワクチン打ってしまった人間は
世界中の記事や報道が見れない本当の情弱。
打つ前に調べなかった残念なやつら。

イスラエルやアメリカやイギリスのざまを見れば
もはやワクチンなど無味だとはっきりしている。
0453ご冗談でしょう?名無しさん
垢版 |
2021/08/15(日) 16:19:32.95ID:???
「アフガニスタン政府軍が自国を守る意志がないのなら、米軍が駐留しても意味がない」
バイデン米大統領 8/15
もし在日米軍が撤退すれば中共軍の餌食’平和憲法’など何の役にも立たない。
0457ご冗談でしょう?名無しさん
垢版 |
2021/08/15(日) 17:08:27.79ID:???
トランプは完全な基地外であり、
国家反逆罪で極刑に処すべきだが、
バイデンも知能では大差ないようだな。
まともなんだが、スピーチが政治慣れしてるだけで
知能は低そうだ。

こんな効き目のないワクチンを
いまだに信じ続けてる時点でバレた。
アメリカのグラフ見てみ。
https://graphics.reuters.com/world-coronavirus-tracker-and-maps/ja/countries-and-territories/united-states/
イスラエルも見とけ、こっちのほうがひどいな。世界一ブザマなグラフだ。
https://graphics.reuters.com/world-coronavirus-tracker-and-maps/ja/countries-and-territories/israel/

どちらもワクチン打ちまくってこのありさまである。
3回目打つとか、4回目も5回目も毎年こんな危険なものを打ち続けるのか。
こんな狂気の世界は見たことないわ。

相対性理論、量子力学、素粒子論を疑いもせず信じてる人間は
このワクチン打ってる人間と本質的にまったく同じだ。
ひとこと、かなり知能が低い。
0460ご冗談でしょう?名無しさん
垢版 |
2021/08/15(日) 17:17:15.92ID:???
打ちたくないなら別に打たんでええわ
いちいち同志を募る必要もないだろ、黙って一人で勝手に逝ってくれたらいい
0461ご冗談でしょう?名無しさん
垢版 |
2021/08/15(日) 17:23:31.85ID:???
アメリカ株は最高値を更新し続けてるいが、これは
カネをばらまき続けてるから、そして今後も大規模インフラ投資でばらまき続けるから
そうなっているだけで、物価は4〜5%も上昇している。
低金利でカネの行先がなく、インフレを見越して株に向かっているだけだ。
何も国内が正常なわけではない。異常そのものである。

白人人口がとうとうマイナスに転じたようで
この先、いずれ内乱が起こるかもしれない。
いずれアメリカ株も安泰ではなくなるので、引き上げどきを見定める必要がある。

コロナ対応を見てると、つくづくアメリカはまったく大したことなく
この国と変わらないブザマな国だと判明した。

まあ、どうあがいても温暖化で人類破滅は確定してるので
世界全体が狂気じみてきているのが分かる。
あと50年持たないだろう。

まず、暑さに弱い昆虫が壊滅して
植物の受粉ができなくなり、動物も終わりだな。

あっという間に終わりそうだ。
好きに生きろ。
0463ご冗談でしょう?名無しさん
垢版 |
2021/08/15(日) 17:42:02.25ID:WW7QxHsj
「成分ごとにニュートンの運動方程式が成り立つから、ベクトルで考えてもかまわないといえる」

これって正しいですか?
ベクトルでニュートンの運動方程式が成り立つから、成分ごとにニュートンの運動方程式を考えてもよいのではないんですか?


テトラ「不思議です!ボールの運動を調べるとき、x成分とy成分に分けてニュートンの運動方程式を立てましたけれど、あたしたちが決める
座標軸の方向なんて、宇宙は知りませんよね?人間が決めた座標軸なのに、どうして計算がうまくいくんでしょうか……」

僕「テトラちゃんはすごいよ。僕が物理で質点の運動を勉強したとき、そんな疑問なんて思いつかなかったなあ……結論からいうと、
x軸とy軸をどんな向きに定めても、成分ごとにニュートンの運動方程式が成り立つというのは、僕たちの宇宙の性質だね。この性質は実験
で確かめられている。」

テトラ「宇宙がそのような性質を持っているというのは、すごいことじゃありませんか?」

僕「すごいことだよ!成分ごとにニュートンの運動方程式が成り立つから、ベクトルで考えてもかまわないといえる」
0464ご冗談でしょう?名無しさん
垢版 |
2021/08/15(日) 17:42:04.26ID:???
結局、相対性理論、量子力学、素粒子論は
一切何の役にも立たず、虚構のままで終わる。

本当に一切何の役も立っていないのに
いまだに信じ続けている人間がいるのには驚愕するが、
コロナで何の疑いもなくワクチンを打つ人間の多さを見ると
いかに世界が阿呆であふれているか、それゆえ虚構理論が存続できてきたのかが
ずいぶんと明らかになってきた。

単純に阿保が大多数だから、ただそれだけのことだったのだ。
物理学ではなく、意味の全くないエネルギー偏微分数学もどきを
やっているだけなのが理論物理学なのである。

この阿保どもは、おそらく真っ先にワクチンを打っているであろう。
阿保ゆえ、疑うことがまったくできないのである。

数年後にどうなるか、高見の見物が正しい選択なのだ。
0469ご冗談でしょう?名無しさん
垢版 |
2021/08/15(日) 18:10:37.95ID:WW7QxHsj
本当にすごいことなんですか?


テトラ「不思議です!ボールの運動を調べるとき、x成分とy成分に分けてニュートンの運動方程式を立てましたけれど、あたしたちが決める
座標軸の方向なんて、宇宙は知りませんよね?人間が決めた座標軸なのに、どうして計算がうまくいくんでしょうか……」

僕「テトラちゃんはすごいよ。僕が物理で質点の運動を勉強したとき、そんな疑問なんて思いつかなかったなあ……結論からいうと、
x軸とy軸をどんな向きに定めても、成分ごとにニュートンの運動方程式が成り立つというのは、僕たちの宇宙の性質だね。この性質は実験
で確かめられている。」

テトラ「宇宙がそのような性質を持っているというのは、すごいことじゃありませんか?」
0471ご冗談でしょう?名無しさん
垢版 |
2021/08/15(日) 18:34:33.55ID:WW7QxHsj
どうして、すごいのでしょうか?
0473ご冗談でしょう?名無しさん
垢版 |
2021/08/15(日) 18:43:21.95ID:0nz5FuwY
>>467
私のただ顔知ってるだけのお爺さん曰く:「脳ミソ使う勉強なんかまったく
しなかった。1日中、ただひたすら計算尺使う練習してた。
0474ご冗談でしょう?名無しさん
垢版 |
2021/08/15(日) 18:49:16.51ID:WW7QxHsj
>>472

ということは、

「成分ごとにニュートンの運動方程式が成り立つから、ベクトルで考えてもかまわないといえる」

の逆が正しいということですか?
0476ご冗談でしょう?名無しさん
垢版 |
2021/08/15(日) 18:51:51.50ID:0nz5FuwY
>>473 は、
>>467 のお姉様❤宛てにあらず。>>466 宛てね。(*^ー゚)ゴメンネ
0479ご冗談でしょう?名無しさん
垢版 |
2021/08/15(日) 19:47:24.42ID:hoWuloBl
>>463
高校でも大学でも
ちゃんと教えてないから仕方ないわな。
物理学の各量をベクトルで考えても良い理由だ。

まず、数学空間でのベクトルが最初にある。
これは各成分を1つに表したものにすぎない。合成則も定義どおりである。
そして速度と加速度であるが、実はこれも数学量である。
これを物理量だと思い込んでおるから、ここから疑問が生じてしまうのだ。
数学空間においては、速度と加速度は最初からベクトルであり
合成則も定義なので、各成分ごとに独立計算するのだ。すべて定義どおりなのである。

次に、物理空間すなわち現実空間での速度と加速度が
数学空間でのベクトルと同一なのかを考える。
承知のとおり、現実空間での速度と加速度は
ガリレイ変換に従うことが物理的に確かめられている。
これは数学空間でのベクトルの定義どおりの計算である。

よって、数学空間と物理空間における
速度と加速度に違いが認められず、同じベクトルとして同じ扱いをしても良いと
いうことにしておるのだ。

さて、ここで問題だ。
力の合成すなわち力の平行四辺形は、はたして定義か法則か。
数学空間と物理空間に分けて答えよ。

くっくっく
0480太上天君
垢版 |
2021/08/15(日) 19:54:48.60ID:biE1Z20E
科学技術はニュートン物理学で成り立っている。
相対性理論や量子力学で成り立っている科学技術はひとつもない。

つまり役に立つのはニュートン物理学だけだってことwwwwwwwwwwwww
0481ご冗談でしょう?名無しさん
垢版 |
2021/08/15(日) 19:59:53.63ID:hoWuloBl
>>480
至極当たり前のことだな。
GPSは相対論、半導体は量子論とかアホもたいがいにしとけよ。

GPSは実補正であり、運用するのにまったく相対論など使っておらんし
半導体は実験で得られた物性を利用しているのであって、量子論による設計で生まれたものではない。

アホザルどもは
とっととワクチン打ちまくって
消え去れ

くっくっく
0483ご冗談でしょう?名無しさん
垢版 |
2021/08/15(日) 20:12:04.05ID:???
999、おじいちゃんなのにワクチン打たないとかすごいな
もうどうせすぐ死ぬからどうでもいいと思ってるんかな
0489ご冗談でしょう?名無しさん
垢版 |
2021/08/15(日) 21:47:26.38ID:ycbngd7Z
現在岩波書店の物理入門コース6量子力学Uで学んでいるのですが、スピン-軌道相互作用と角運動量の合成でつまずいています

ψ(ml,ms)やクレプシュ-ゴルダン係数及びψをスピン関数の積で表す辺りです

いくつかの資料には|1/2〉等の表示を使って説明してあるのですが本書ではこの辺にブラケットを用いてなく割いてあるページも少なく理解が困難です

何か詳しく解りやすい書籍等ありましたら教えて下さい
よろしくお願いいたします
0491ご冗談でしょう?名無しさん
垢版 |
2021/08/15(日) 22:37:01.34ID:/6MKEeXC
>>489
逆に問う。ブラケットじゃないとダメなん?
なおせばええんや。
0492ご冗談でしょう?名無しさん
垢版 |
2021/08/15(日) 23:22:23.72ID:pBgSeauB
「よつべ 量子力学 クレプシュゴルダン係数」でググると、
ホントろくな解説サイトにひっかからんな。ひたすら滑舌悪かったり、
喋りそのものがド下手くそで、まったく耳貸す気にならんかったり…。
英語のサイトなら聞けるのは(-ω- ?)や?
0493ご冗談でしょう?名無しさん
垢版 |
2021/08/16(月) 00:15:30.13ID:e66L2m+g
>>489 youtube
Clebsh-Gordan coefficients Explained
Pretty Much Physics
0494ご冗談でしょう?名無しさん
垢版 |
2021/08/16(月) 01:38:22.53ID:ycZx0zIG
変な質問をしているのかもしれませんが、
よろしければ>>422の質問にどなたか回答してもらえませんか
0495ご冗談でしょう?名無しさん
垢版 |
2021/08/16(月) 01:51:05.04ID:???
>>494
正直良くわからん

ただ場の量子論においてはもはやクラインゴルドン場やディラック場などは状態ではなく演算子であるので
波動関数とは全く異なるものと思ってください
0496ご冗談でしょう?名無しさん
垢版 |
2021/08/16(月) 02:07:31.58ID:???
シュレーディンガー方程式をオイラー・ラグランジュ方程式に当てはめてみりゃいいじゃん
うまく対応取れたら可能、できなきゃダメなだけ
0497ご冗談でしょう?名無しさん
垢版 |
2021/08/16(月) 02:35:24.25ID:ycZx0zIG
>>496
そのような方法でシュレーディンガー方程式を導くことの出来るラグランジアンは実際あります。
http://yuru2physics.blog.fc2.com/blog-entry-265.html
(NGワード回避のため一部全角にしてあります)

しかし、ここでの力学変数はpsi(t, r)になっており、力学変数を| psi(t) >とするゆなフォーマリズムはあるのか?
というのが私の疑問です。
自力ではそのようなラグランジアンを作れませんでしたので
0499ご冗談でしょう?名無しさん
垢版 |
2021/08/16(月) 02:40:48.97ID:ycZx0zIG
ちなみに、昨日からそもそも自分の質問が変なのではないか、と引け目を感じているのは、
そもそも| psi(t) >を考えている時点ですでにハミルトンフォーマリズムなので、
それを力学変数とするようなラグランジアンを考えようとすること自体がナンセンスなような気もするのです。

しかし、シュレーディンガー方程式はヒルベルト空間の元| psi(t) >の時間発展を決定論的に指定する運動方程式なわけですから、
古典論としてシュレーディンガー方程式を導くようなラグランジアンがあったっておかしくないだろう、とも思うわけです。
そこで以上のような質問に至りました。

変な質問なのか初歩的な質問なのかよく分かりませんが、どなたかヒントをお持ちの方がいらっしゃれば教えてください。
0500ご冗談でしょう?名無しさん
垢版 |
2021/08/16(月) 02:42:30.90ID:ycZx0zIG
>>498
ベクトルの指数関数というのは確かになんかよく分かりませんね
しかし、それが定義できなければ、なぜラグランジュフォーマリズムが不可能なのですか?
0501ご冗談でしょう?名無しさん
垢版 |
2021/08/16(月) 02:42:56.59ID:???
それと確率論的決定論であることを以って古典論とは呼ばないので術語は正しくつかってください
0502ご冗談でしょう?名無しさん
垢版 |
2021/08/16(月) 02:46:52.70ID:ycZx0zIG
>>501
まだ学生なので理解が正しくない点もあるかもしれません。
特に本筋に影響を与えるほどの御用であるようには思いませんが、
もし無用の混乱を招いてしまったのであれば失礼しました。
0504ご冗談でしょう?名無しさん
垢版 |
2021/08/16(月) 03:00:46.86ID:???
状態じゃなくて場と思えるからラグランジアンでかけるイメージ

対角化をとらないと
シュレディンガー方程式は無限次元の連立微分方程式でしかないので
0505ご冗談でしょう?名無しさん
垢版 |
2021/08/16(月) 03:20:21.74ID:ycZx0zIG
>>503
なるほど。ご指摘は
「ヒルベルト空間内を決定論的に時間発展しているからといって、その支配方程式が古典論であるわけではない」
ということでよろしいでしょうか。
たしかに仰る通りだと思いますが、であれば、決定論とか古典論とかいう言葉は一旦やめることにして、
ヒルベルト空間内での状態の軌跡が停留値となるような、作用(時刻tの関数| psi(t) >の汎関数)を記述することはできるか?
というのが私の質問です。
変な質問にお付き合いいただきありがとうございます。
0506ご冗談でしょう?名無しさん
垢版 |
2021/08/16(月) 03:24:04.54ID:ycZx0zIG
>>504
無限次元の場合はオイラーラグランジュ方程式が無限個でてくるという考え方でいけそうだと思うのですが変ですかね?
(連続極限については物理なのでいったん棚上げしておきます)
0507ご冗談でしょう?名無しさん
垢版 |
2021/08/16(月) 03:32:27.66ID:???
シュレディンガー方程式はハミルトニアンさえ与えられれば系の時間発展を記述するに必要十分なわけですが、ラグランジアンはそのハミルトニアンからいつでも系統的に得られるわけではありません
量子スピン系の場合は正準交換関係の代わりにSU(2)の代数が課されるわけですが、少し面倒になります
任意の非可換量を含むハミルトニアンに対して一般にラグランジアンを求めることは多分できないと思います
ケットに対するシュレディンガー方程式は、座標表示とは異なり、そういった任意の代数を持つ系に対して成り立つはずですので、そのような一般的なラグランジアンや作用を求めることはできないように思われます
0508ご冗談でしょう?名無しさん
垢版 |
2021/08/16(月) 03:51:01.59ID:ycZx0zIG
>>507
回答ありがとうございます。
たしかにスピンまで含めると、
(位置固有状態で張られるヒルベルト空間) ⊗ (スピン固有状態で張られるヒルベルト空間)
内での時間発展を考える必要があるので仰る通りややこしくなると思います。
したがって、
> 任意の非可換量を含むハミルトニアンに対して一般にラグランジアンを求めることは多分できないと思います
には私も賛成です。

しかし、スピンがない場合については、時間依存するケットを引き数とする汎関数として作用を記述できませんかね?
そのような作用から、最小作用の原理を経由してケットに対するシュレーディンガー方程式を導出してみたいのですが……
0509ご冗談でしょう?名無しさん
垢版 |
2021/08/16(月) 04:00:38.22ID:ycZx0zIG
ちなみに、もし、そのような作用
(ヒルベルト空間内での状態の軌跡 { | psi(t) > | t ∈ R} が停留となる、時刻tの関数| psi(t) >の汎関数)
をつくることができれば、対応するオイラーラグランジュ方程式たちは、
たぶんケットに対するEL方程式たちとブラに対するEL方程式たちとが出てくるんじゃないかと予想してます。
(複素スカラー場psi(t, r)を力学変数とする場合もpsiに対するEL方程式とpsiの複素共役に対するEL方程式とがあるので)
0510ご冗談でしょう?名無しさん
垢版 |
2021/08/16(月) 04:04:42.92ID:???
作用を導出することができたとして、そのために追加で課す条件は座標表示で行う場合と本質的に同等なものになろうかと思います
0511ご冗談でしょう?名無しさん
垢版 |
2021/08/16(月) 06:24:18.11ID:???
どんな量子系でも経路積分ではラグランジアン使うんだから、ラグランジアンがないってことはないんじゃないですか?

状態ベクトルをラグランジアンに埋め込むとかそういうことが数学的にはどうなっているのかということな気がしますけど
0512ご冗談でしょう?名無しさん
垢版 |
2021/08/16(月) 06:34:25.94ID:???
>>422
>psi(t, r) = < r | psi(t) >を場と思って、これを力学変数とするラグランジアンから
>シュレーディンガー方程式を導くことはできるのですが、

そもそもこれがなんか違う気がするんですけど
0513ご冗談でしょう?名無しさん
垢版 |
2021/08/16(月) 06:34:33.08ID:ycZx0zIG
>>510
追加で課す条件というのは正準交換関係のことでしょうか。
正準量子化しなくても古典的運動方程式として直接シュレーディンガー方程式が出てくればいいなあと思っています

>>511
> どんな量子系でも経路積分ではラグランジアン使うんだから、ラグランジアンがないってことはないんじゃないですか?
その通りだと思います。
基本的にはラグランジアンとハミルトニアンとはルジャンドル変換によってメカニックに行き来できますので。

しかし、ここでは量子力学を古典論として導出できるようなラグランジアンが欲しい、という議論をしており、
量子力学においてはスピンは
(位置固有状態で張られるヒルベルト空間) ⊗ (スピン固有状態で張られるヒルベルト空間)
という形でアドホックに扱いますので、スピンまで考慮に入れだすとややこしくなるだろうと考え、
>>508では>>507に賛成しました。
(というか、ラグランジュフォーマリズムでまともにスピンを扱おうとするとスピノルが必要なので、
どのみち「量子力学を古典論として導出できるようなラグランジアンが欲しい」という、
ここでのモチベーションに照らしましても、スピン系を除外しておくのは妥当だと考えました)

> 状態ベクトルをラグランジアンに埋め込むとかそういうことが数学的にはどうなっているのかということな気がしますけど
そうなんです。
>>499でも述べていますが、これがどうなってんのかサッパリ分からないんですよね。
やっぱり問い自体がナンセンスなんですかねえ。
0514ご冗談でしょう?名無しさん
垢版 |
2021/08/16(月) 06:36:52.82ID:ycZx0zIG
>>512
シュレーディンガー方程式をラグランジアンから導く議論は実際あります。
http://yuru2physics.blog.fc2.com/blog-entry-265.html
(NGワード回避のため一部全角にしてあります)

NGワードが何か分からなくて>>422に載せられませんでした。
ちなみにこの手の議論は自由理論のラグランジアンからディラック方程式を導出する議論と全く同じです。
0515ご冗談でしょう?名無しさん
垢版 |
2021/08/16(月) 07:21:31.33ID:???
なんか自分も混乱してきたんですけど、今回は結構よくわからないことしてる気がするんですよね

もし仮に>>514のurlのようにΨを力学変数として取り扱います

普通の場の理論ではどうするかというと、Ψは”演算子”だと解釈します
なので、|Ψ>というようなものはそもそも存在しないです

なので、今回はΨは波動関数のような古典的な場だと考える必要がある
そうすると、Ψはある”状態”を表すものなので|Ψ>という表現に意味が出てくる

で、普通の波動関数を使う量子力学ではどうするかというと、状態に作用する”演算子”をハミルトニアンから作り出す操作をする

なんかここら辺ちょっと整理したら何か見えてくるんですかね
私が混乱してるだけかもしれませんけど
0516ご冗談でしょう?名無しさん
垢版 |
2021/08/16(月) 07:44:02.21ID:ycZx0zIG
>>515
> もし仮に>>514のurlのようにΨを力学変数として取り扱います
リンク先では明示的には書いていませんが力学変数はΨ(t, r)ですね。念のため

> 普通の場の理論ではどうするかというと、Ψは”演算子”だと解釈します
> なので、|Ψ>というようなものはそもそも存在しないです
演算子形式ならそうですね。経路積分形式ならΨは普通に古典場です。
どのみち|Ψ>なるものが意味をなさないというのは仰る通りだと思います。

> なので、今回はΨは波動関数のような古典的な場だと考える必要がある
> そうすると、Ψはある”状態”を表すものなので|Ψ>という表現に意味が出てくる
「そうすると」っていうか、むしろ、ここ逆接じゃないですかね?
今回、力学変数Ψ(t, r)は古典場であるにもかかわらず、
ブラケットノーテーションの量子力学と話を結びつけるには、
こいつを< r | Ψ(t) >と対応づけないといけなくなって???となる。
このあたり整理するともうちょっとクリアになりそうです。

どうも第一量子化と第二量子化の沼にハマってるような気がしてなりません。
0518シュレーディンガー方程式を停留条件とする作用が欲しい人
垢版 |
2021/08/16(月) 10:29:35.46ID:ycZx0zIG
SU(N)ハイゼンベルグ模型ってちょっと存じ上げないのですがググった感じスピン系のことですかね?
>>507-508 >>513を踏まえスピン系はひとまず除外ということでお願いできますでしょうか。

(質問が長引きそうなので一時的にハンドルネームをつけました)
0519ご冗談でしょう?名無しさん
垢版 |
2021/08/16(月) 10:48:50.65ID:???
どんな量子系でもラグランジアンで書けるというのは思い込みです
量子系はハミルトニアンがあれば記述でき、それとルジャンドル変換ができるというのは別の仮定です
0521ご冗談でしょう?名無しさん
垢版 |
2021/08/16(月) 11:19:14.27ID:???
スカラー場の量子論しか考えないが座標表示は使わないという縛りにどれくらい意味があるのか分かりかねます
0522ご冗談でしょう?名無しさん
垢版 |
2021/08/16(月) 12:09:09.31ID:ycZx0zIG
>>520
「作用を導出することができたとして、そのために、
 正準交換関係や|ψ(t)>が|x>で展開できることなどを条件として課す」
という日本語がいまいち理解できません。
いま導出すべきものは作用ではなく、むしろシュレーディンガー方程式が導出すべき方程式です。
シュレーディンガー方程式を運動方程式として導出することができるような作用を見つけたい、というのが、
この議論におけるモチベーションになっております。
そして、作用から、その運動方程式としてシュレーディンガー方程式を導出するにあたり、
必要な仮定は「運動方程式が作用の停留条件になっている」(最小作用の原理)ということだけです。
なので、ここでの議論において正順交換関係や完全性などの条件は特に現れないと思います。
0523ご冗談でしょう?名無しさん
垢版 |
2021/08/16(月) 12:10:02.55ID:ycZx0zIG
>>521
まずスカラー場の理論しか考えないのは、シュレーディンガー方程式に現れるpsiがベクトル添字やスピノル添字を持たないからです。
特にこの点に異論はないと思います。
次に、作用から古典論として導出したい運動方程式が、psi(t, r)の時空間発展を記述するシュレーディンガー方程式ではなく、
| psi(t) >の時間発展を記述するシュレーディンガー方程式の導出にこだわっている点についてですが、
こちらについては説明が必要だと思うので以下に回答します。

- psi(t, r)を力学変数とする議論はお馴染みだが、| psi(t) >を力学変数とする議論はこれまで見たことがない。単純に興味がある。

- 実際にそのような作用をつくることができればOKだし、
 そのような作用をつくることができないのなら、その理由を考えることで一層理解が深まりそう。

- 座標表示という特定の基底に依存した議論よりも、運動量表示のシュレーディンガー方程式すらも
 作用から自動的に導出できてしまう議論の方がイケてる(このところ私の主観)。

- 量子力学は場の理論の言葉で言えば0 + 1次元の場の理論であり、psi(t, r)を力学変数としている時点で、
 実はすでに({ r }の次元) + 1次元の場の理論になっている。
 シュレーディンガー方程式を0 + 1次元の場の理論として導くためには、psi(t, r)からr依存性をファクターアウトする必要があり、
 そのためには1粒子の量子力学におけるpsi(t, r) = < r | psi(t) >の類推から| psi(t) >を力学変数としてみるのが良さそうに思える。

| psi(t) >を力学変数としてみたい理由としては、一番最後に挙げたものが最大の理由です。
回答になりましたでしょうか。
0524ご冗談でしょう?名無しさん
垢版 |
2021/08/16(月) 12:14:27.10ID:???
言葉としては言いたいことが分かるようになってきたけど
全体的に古典的な物理量と量子化された物理量がごっちゃになって認識されている気がする
0525シュレーディンガー方程式を停留条件とする作用が欲しい人
垢版 |
2021/08/16(月) 12:24:22.80ID:ycZx0zIG
>>524
結局のところ、私は自力で以上のような作用をつくることができていないので、
私がこのスレに期待していることとしては「実際にこんな作用があるよ」or
「君の議論そこがおかしいからそんな作用つくれるわけないよ」のどちらかですね。
ですから、私のここまでのレスになにか欠陥があるのであれば指摘が欲しいです。
0526ご冗談でしょう?名無しさん
垢版 |
2021/08/16(月) 13:29:22.24ID:ozHjFjY0
「成分ごとにニュートンの運動方程式が成り立つから、ベクトルで考えてもかまわないといえる」

これって正しいですか?
ベクトルでニュートンの運動方程式が成り立つから、成分ごとにニュートンの運動方程式を考えてもよいのではないんですか?


テトラ「不思議です!ボールの運動を調べるとき、x成分とy成分に分けてニュートンの運動方程式を立てましたけれど、あたしたちが決める
座標軸の方向なんて、宇宙は知りませんよね?人間が決めた座標軸なのに、どうして計算がうまくいくんでしょうか……」

僕「テトラちゃんはすごいよ。僕が物理で質点の運動を勉強したとき、そんな疑問なんて思いつかなかったなあ……結論からいうと、
x軸とy軸をどんな向きに定めても、成分ごとにニュートンの運動方程式が成り立つというのは、僕たちの宇宙の性質だね。この性質は実験
で確かめられている。」

テトラ「宇宙がそのような性質を持っているというのは、すごいことじゃありませんか?」

僕「すごいことだよ!成分ごとにニュートンの運動方程式が成り立つから、ベクトルで考えてもかまわないといえる」
0530ご冗談でしょう?名無しさん
垢版 |
2021/08/16(月) 14:10:13.39ID:ozHjFjY0
>>527

ということは、

>>526

で言っていることは、不思議でもなんでもないということになりますね。
ニュートンが述べている法則に含まれるということですよね?
0531ご冗談でしょう?名無しさん
垢版 |
2021/08/16(月) 14:27:54.21ID:???
>>530
ニュートンの法則が成り立つのが不思議ってこった。
不思議かどうかなんてその人の感覚なんだからおまえが決めるこっちゃない。
0533ご冗談でしょう?名無しさん
垢版 |
2021/08/16(月) 14:31:15.22ID:ozHjFjY0
>>531

だったら、結城浩さんはなぜ、単に、テトラ「ニュートンの法則は不思議ですね。」と書くだけにしなかったのでしょうか?
成分がどうのこうの言う必要などないはずです。
0536ご冗談でしょう?名無しさん
垢版 |
2021/08/16(月) 14:50:50.27ID:???
>>533
マジレスすると結城浩はおそらく
「俺はこんなに数学が得意なんだ!」
ってイキってるだけの大学中退の文系だから
0537ご冗談でしょう?名無しさん
垢版 |
2021/08/16(月) 14:53:48.96ID:???
>>497
|Ψ> と Ψ(t, r) は同じもの
オイラー・ラグランジュ方程式を導くためには状態ベクトル |Ψ> を
関数 Ψ(t, r) で表現する必要があるだけ
0538シュレーディンガー方程式を停留条件とする作用が欲しい人
垢版 |
2021/08/16(月) 15:05:56.61ID:ycZx0zIG
>>537
同じではないと思いますね。
| psi(t) >の| r >成分がpsi(t, r)なので。
0539ご冗談でしょう?名無しさん
垢版 |
2021/08/16(月) 15:10:26.51ID:ycZx0zIG
というか、そもそも両者の違いは多粒子系の量子力学を考えれば明らかで、
多粒子系の波動関数はpsi(t, r1, r2, ..., rN)といった具合になります。
したがって、これは時空間にひろがる場ではなく、こういう形の引数を取る関数です。
一方、>>497に示しているリンク先の議論ではpsi(t, r)は時空間上の場です。

たまたま1粒子の量子力学における波動関数がpsi(t, r1)のような形をしているので場のように見えるだけ、というのが私の理解ですね。
0541ご冗談でしょう?名無しさん
垢版 |
2021/08/16(月) 15:55:09.52ID:???
>>539
やっぱなんか無理な気がしてきました

状態に作用するシュレーディンガー方程式って
i∂/∂t |ψ(t)>=H(p,t)|ψ(t)>ですよね

ラグランジアンがψ(r,t)しか含まれないとして、pという微分演算子でもない抽象的な演算子を導出することができるかと言われたら無理な気がします
0542シュレーディンガー方程式を停留条件とする作用が欲しい人
垢版 |
2021/08/16(月) 16:38:28.69ID:ycZx0zIG
>>541
そうなんですよ、まさにそうなんです。
だから、試しにハミルトニアンの具体形を考えるのはやめて、シュレーディンガー方程式i d/dt | ψ(t) > = H(t, hat{x}, hat{p}) | ψ(t) >
(xが演算子であることを明示的に表わすためにhat{x}としてあります。LaTeXで書けないのつらい……)
がそのまま出てくるように無理やりラグランジアンを
L = < ψ(t) | [ H(t, hat{x}, hat{p}) - i (d/dt)^2 ] | ψ(t) >
としてみますとですね……
(左辺のLはc-数です。右辺のH(...)は全体として演算子。わけわからないけど、ひとまず、こうするとですね)

オイラーラグランジュ方程式は成分< ψ(t) | x >ごとに立てる必要があるので
∂L / ∂( < ψ(t) | x > ) = < x | H(t, hat{x}, hat{p}) | ψ(t) > = H(t, x, ∂/∂x) < x | ψ(t)>,
∂L / ∂( d/dt < ψ(t) | x > ) = i d/dt < x | ψ(t) >
を用いて、EL方程式として i d/dt < x | ψ(t) > = H(t, x, ∂/∂x) < x | ψ(t)>が形式上でてきます。

まあ、この時点で結局、座標表示になっとるやないかい、とツッコミどころ満載なわけですが、
両辺に| x >を作用してxで積分してやることで一応 i d/dt | ψ(t) > = H(t, hat{x}, hat{p}) | ψ(t)>になって、
ひとまず形式的にはなんとかなるんですが、そもそもラグランジアンの定義式の右辺に
ヒルベルト空間に作用するハミルトニアンが入ってるとは一体何事か、というわけで、まあわけわからなくなって質問に来たわけです。

(むりやり数式かくの疲れた……)
0543シュレーディンガー方程式を停留条件とする作用が欲しい人
垢版 |
2021/08/16(月) 16:43:22.06ID:ycZx0zIG
>>541
すみません。凡ミス。
ラグランジアンの定義式を
L = < ψ(t) | H(t, hat{x}, hat{p}) | ψ(t) > + i d/dt < ψ(t) | ψ(t) >
にすると、
∂L / ∂( < ψ(t) | x > ) = < x | H(t, hat{x}, hat{p}) | ψ(t) > = H(t, x, ∂/∂x) < x | ψ(t)>,
d/dt [∂L / ∂( d/dt < ψ(t) | x > )] = d/dt [ i < x | ψ(t) > ]
だからシュレーディンガー方程式が出てくる、という感じです。
まあ、そもそも、この論法の妥当性じたい怪しいのですけれど、一応ミスなので訂正しておきます。
0544シュレーディンガー方程式を停留条件とする作用が欲しい人
垢版 |
2021/08/16(月) 16:50:11.62ID:ycZx0zIG
というか、いま気付いたんですけど、このラグランジアンの定義式、ちょっとだけルジャンドル変換に似てるな……
まあ関係ない……か……?
0545ご冗談でしょう?名無しさん
垢版 |
2021/08/16(月) 17:50:54.10ID:Pl/sAtUf
>>523
キミはシュレーディンガー方程式を駆使して機械を設計してみろ?
0546ご冗談でしょう?名無しさん
垢版 |
2021/08/16(月) 17:52:14.90ID:Pl/sAtUf
>>517
キミはハイゼンベルグ模型をつくったのかな?

どんな動きをする模型なのか動画をアップしてみろ?
0547ご冗談でしょう?名無しさん
垢版 |
2021/08/16(月) 17:53:42.32ID:Pl/sAtUf
>>513
キミはラグランジアンとハミルトニアンがルジャンドル変換によって、行き来する模型をつくってみなさい!!!
0549ご冗談でしょう?名無しさん
垢版 |
2021/08/16(月) 18:10:20.67ID:ycZx0zIG
>>548
モチベーションについては>>523で説明している通りです
0550ご冗談でしょう?名無しさん
垢版 |
2021/08/16(月) 18:37:26.46ID:???
>>549
シュレディンガー場の作用はもはやもともとの一次元の作用じゃないというか
もう第一量子化してるからなぁ

場の作用を(第二)量子化したら1次元の量子論ではないのは当たり前な感じやけど
0551ご冗談でしょう?名無しさん
垢版 |
2021/08/16(月) 18:44:58.05ID:ozHjFjY0
Michael Spivakの『Physics for Mathematicians, Mechanics I』ってどうですか?

読んだ人はいませんか?
0553ご冗談でしょう?名無しさん
垢版 |
2021/08/16(月) 18:53:50.36ID:ycZx0zIG
>>550
その通りです。
ですから当然>>514のリンク先の議論において(空間次元+1)次元の理論が扱われていることに異論はありません。

一方、第一量子化によって得られる量子力学は0 + 1次元の場の理論であるはずなので、
>>514のリンク先の議論をもって粒子の量子力学(シュレーディンガー方程式)が導かれたとは言えないわけです。
(リンク先の議論に現れるpsi(t, r)が粒子の量子力学| psi(t) >の| r >成分であると言うこともできないと思う。なぜなら>>539)

そういうわけで、>>514とは別に、| psi(t) >を力学変数とするようなラグランジアンから出発して、
0 + 1次元の理論としてのシュレーディンガー方程式を導出できないだろうか?というのが私の質問です。
0555ご冗談でしょう?名無しさん
垢版 |
2021/08/16(月) 19:21:00.34ID:ozHjFjY0
>>551

数学者の書いた本なので、物理学者の書いた本よりも分かりやすいはずです。
0556ご冗談でしょう?名無しさん
垢版 |
2021/08/16(月) 19:33:29.98ID:ycZx0zIG
>>554
議論のどの箇所に問題があるか、具体的に指定してもらえれば勉強になるのですけれど
0557ご冗談でしょう?名無しさん
垢版 |
2021/08/16(月) 19:39:27.62ID:???
>>522
言葉が気に入らなければ「導出する」を「導出することができるような作用を見つける」と読み替えてください
最小作用の原理の他に、|ψ(t)>が|x>で張られる空間の上でゼロベクトルでない、あるいはそれと同等な条件・仮定・前提が必要になるだろうという意味です
それが必要でないのなら場の理論に限定する必要もありません

>>523
シュレディンガー方程式に現れる|ψ(t)>は一般に添え字を持ち得ます
QFTが量子論のすべてでは決してありません
0558ご冗談でしょう?名無しさん
垢版 |
2021/08/16(月) 20:03:49.62ID:ycZx0zIG
>>557
>>510の日本語の意味についてはひとまず理解できました。
なぜ「最小作用の原理の他に条件が必要ないのなら場の理論に限定する必要がない」と言えるのですか?
限定されなかったとして、他に、たとえばどんなものがあると考えておられるのですか?
ここを説明していただければ、さかのぼりますが>>521で仰っていることは飲み込めると思います。

>>523
QFTが量子論のすべてであるかどうか、までは私の興味の射程外ですが、
少なくともシュレディンガー方程式に現れる|ψ(t)>が添字を持つ例については、私は見たことがありません。
お手数ですがリンクなどで例示していただければ>>521の指摘が分かると思います。
0559シュレーディンガー方程式を停留条件とする作用が欲しい人
垢版 |
2021/08/16(月) 20:10:22.62ID:ycZx0zIG
というか、仮にシュレーディンガー方程式に現れるケットが一般に添字を持つような例があったとしても、
まずは最も簡単な添字がない場合から議論をはじめてみる、というのは、物理ではよくあるスタイルですし、
特にスカラー場の理論に限定していることには問題がないと考えています。
(もし添字がつくのだとしたら、ますます議論が複雑になるだけなので)
0560ご冗談でしょう?名無しさん
垢版 |
2021/08/16(月) 20:16:55.04ID:???
>>558
いずれに対しても例として>>517を挙げます
|ψ>が|x>で展開できることなどの仮定なしに i(d/dt)|ψ> = H|ψ> が作用原理から得られるならば、
QFTに限らず任意の量子系に対して成り立つことを意味すると思います
そのようなことは恐らくできないでしょう
0563ご冗談でしょう?名無しさん
垢版 |
2021/08/16(月) 23:16:10.13ID:???
せん断耐力について

コンクリートの指圧強度で決まるせん断耐力=長期のせん断耐力+短期のせん断耐力

鋼材で決まるせん断耐力=短期のせん断耐力

みたいな書き方があと施行アンカーの参考書でされてるのですがなぜこうなるのでしょうか?

https://i.imgur.com/JHw3pMK.jpg
0564ご冗談でしょう?名無しさん
垢版 |
2021/08/16(月) 23:24:13.47ID:/RErUuQx
>>552
ホントにここの達人は、いったいどーやって🤓🤡🤢(´ω`🌀)
個人特定できるん?まさか人目で分かる
おかしな特徴でもあるの?
0568ご冗談でしょう?名無しさん
垢版 |
2021/08/17(火) 03:40:19.44ID:AVTgkHIq
>>560
SU(N)ハイゼンベルグ模型ちょっと調べてました。
https://arxiv.org/pdf/0812.3657.pdf
たとえば、この(3)がSU(N)の基本表現基底でインデックスされているため、
ローレンツ変換にしたがわない添字を持つことがある、という主張でよろしいでしょうか。

であれば、仰っていることは正しいと思います。
というか、
> |ψ>が|x>で展開できることなどの仮定
をしているのが、まさに>>513において私が述べた通り、
位置固有状態で張られるヒルベルト空間内でのみ時間発展を考えましょう、というセットアップなわけです。
したがって、仰る通り一般には、スピン固有状態で張られるヒルベルト空間での時間発展もあり得ますね。
(とはいえ、それがQEDから演繹できない、というご意見には賛同しかねますね。
>>517の回答としては、非常に不親切ですがQEDラグランジアンを挙げておきます。
非常に煩雑な計算が必要なはずなのでデモンストレートする気力がわきませんが、
SU(N)ハイゼンベルグ模型はQFTの非相対論極限として演繹されるはずです。
そういう研究を誰もなさってないのなら、計算してみるのも面白いと思います。)
0569ご冗談でしょう?名無しさん
垢版 |
2021/08/17(火) 03:40:40.19ID:AVTgkHIq
>>560
さて、話がだいぶん長くなりましたが、結局、当初の>>521のご指摘に対する私の回答としましては、
>>523の後半についてはご納得いただけいると思うのですが、>>523の前半については捕捉するなら、
やはり「まずシンプルなセットアップで考えてみたいから」ということになります。
スピンまで入れてしまうと、さきほど「煩雑な計算が必要」と註釈しましたとおり、
非相対論極限をとる前のQFTからの類推が容易ではなくなるため、
いったんスピンのことは忘れておいた方がよかろうというわけです。
スピンまで考慮にいれたシュレーディンガー方程式については、ご指摘のとおり、
スピン固有状態で張られるヒルベルト空間における代数を仮定しない限り、作用など構成できないでしょうね。

長文失礼しました。
0570シュレーディンガー方程式を停留条件とする作用が欲しい人
垢版 |
2021/08/17(火) 03:44:47.98ID:AVTgkHIq
>>567
でしたら、>>554のご指摘に忠実に回答しますが、
ここでの議論は、ケットの時間発展を記述するシュレーディンガー方程式を
停留条件とするような、時間依存ケットの汎関数をどうやって定義すればよいか?というものです。
定義に戻る、というより、どうやって作用を定義すべきかが私の質問です。
よろしいでしょうか。
0571ご冗談でしょう?名無しさん
垢版 |
2021/08/17(火) 04:04:38.84ID:???
オイラーラグランジュ方程式がシュレーディンガー方程式になるようなラグランジアンなら
ググれば引っかかるが、そういう話ではなくて?
0572シュレーディンガー方程式を停留条件とする作用が欲しい人
垢版 |
2021/08/17(火) 04:16:06.25ID:AVTgkHIq
>>571
たとえば
http://yuru2physics.blog.fc2.com/blog-entry-265.html
(NGワード回避のため一部全角にしてあります)
などがありますね。
ただし、この議論では力学変数をpsi(t, r)ととってpsi(t, r)の時空間発展を記述するシュレーディンガー方程式を導いています。
私の質問は、| psi(t) >の時間発展を記述するシュレーディンガー方程式を、
| psi(t) >を力学変数とする作用の停留条件から導くにはラグランジアンをどう定義すればよいだろう?というものです。

| psi(t) >を力学変数とするというのは普通しないことでして(私自身このように力学変数を設定してよいものかよく分からない)
そのような議論は見たことがありません。
もし、| psi(t) >を力学変数としてシュレーディンガー方程式を導く議論がググって引っかかったのでしたら、
ぜひ教えて欲しいです。
0573ご冗談でしょう?名無しさん
垢版 |
2021/08/17(火) 05:35:52.63ID:???
>>569
別にシンプルなセットアップで考えることを否定しているのではなく
そうして得られた i(d/dt)|ψ> = H|ψ> は非常に一般的な形であるから導出に際して追加の条件や仮定や前提があるはずで、
それは結局ド・ブロイ場ψ(x,t)のラグランジアンを考える普通の方法と本質的に違いはなかろうと予想するわけです
何にせよこの場でこれ以上意味のある議論は難しいと思います

なお、>>568のようにQEDから例えば物性の全てが演繹できるというのは自然界の階層性を無視した幻想であるわけですが
そもそも>>517は演繹できないということを主張するものでは全くなく
複雑で汚い現象論的なハミルトニアンに直接対応するラグランジアンが存在するかは定かでないということです
0575ご冗談でしょう?名無しさん
垢版 |
2021/08/17(火) 13:32:02.25ID:AVTgkHIq
>>573
おっしゃる「追加の条件や仮定や前提」によって
「ド・ブロイ場ψ(x,t)のラグランジアンを考える普通の方法と本質的に違いは」なくなることの根拠が分からないので議論できませんが、
少なくとも私は土・ブロイ場のラグランジアンからその運動方程式を導く仮定とは本質的に異なるだろうと考えています。
なぜなら>>553で述べました通り、理論としての次元がそもそも異なるからです。
形式上にかよった導出になることはあっても、むしろ本質的には別物になるはずです。

> 何にせよこの場でこれ以上意味のある議論は難しいと思います

私がこのスレにもとめているのは、「実際にこのように作用を構成できる」か
「作用は構成できるわけがない。なぜなら〜」といった回答をいただけることですので、
なぜ追加の条件によって結局普通の方法と本質的に同じになると考えておられるのか伺ってみたいものですが、
ひとまずはここまで長らくおつきあい下さりありがとうございました。
別の方から何かご意見を伺えるかもしれないので、この場で回答をくださる方には引き続き対応してみます。

> なお、>>568のようにQEDから例えば物性の全てが演繹できるというのは自然界の階層性を無視した幻想であるわけですが
> そもそも>>517は演繹できないということを主張するものでは全くなく
> 複雑で汚い現象論的なハミルトニアンに直接対応するラグランジアンが存在するかは定かでないということです

幻想ではありません。階層性に言及しておられますが、
たとえば凝縮系の有名なモデルであるイジング模型は場の理論のIR固定点として実現されます。
したがって、「QEDを知らなくても凝縮系の有効理論を議論できる」は正しいステートメントですが
「QEDから凝縮系の物性の全ては演繹できない」は誤っています。
(でなければ、とりもなおさず、凝縮系において新物理が見えていることになりますからね)
しかし、現象論的に構成されたハミルトニアンに必ずしもラグランジアンが対応しないのは私も同意します。
ただし、今回の質問ではそのような現象論的なハミルトニアンまで考慮にいれていません。(だからこそスピン系は棚上げしておきたい)
0576ご冗談でしょう?名無しさん
垢版 |
2021/08/17(火) 13:46:55.57ID:???
なんか物理方程式として
f:t→t,r
が導出できないか

と言いたいって質問に聞こえるけどこの理解で合ってる?
それなら答えは無理だろ常識的にになるけど
0577シュレーディンガー方程式を停留条件とする作用が欲しい人
垢版 |
2021/08/17(火) 14:08:41.16ID:AVTgkHIq
>>576
すみません、「物理方程式としてf: t --> t, rを導出する」という言葉の意味がよく分からないです。
回答になるか分かりませんんが、一応、ここまでの議論によって、
私の質問は次のように短くまとめられるようにはなったので掲げておきます:

時刻tの関数| psi(t) >を引数とする汎関数S[ | psi(t) > ]で、
その停留条件delta S = 0としてシュレーディンガー方程式i (d/dt) | psi(t) > = H(t, x, p) | psi(t) >
が実現されるような作用をつくることはできるか?
(ただしHについては、自由理論に限るや、スピン系は除外する、などの制限を適宜追加してOK)

というものです。ちなみに、

時刻t空間rの関数psi(t, r)を引数とする汎関数S[ psi(t, r) ]で、
その停留条件delta S = 0としてシュレーディンガー方程式i (∂/∂t) psi(t, r) = H(t, x, p) psi(t, r)
が実現されるような作用

は実際つくることはできます。
http://yuru2physics.blog.fc2.com/blog-entry-265.html
(NGワード回避のため一部全角にしてあります)
しかし、時刻tの関数| psi(t) >を引数とする汎関数S[ | psi(t) > ]から
シュレーディンガー方程式を導く議論は見たことがないので質問しました。
0579ご冗談でしょう?名無しさん
垢版 |
2021/08/17(火) 15:58:41.30ID:???
>>575
残念ながらQEDからはフェルミ液体論すら導出されません
非常に煩雑な計算さえすればどんな有効模型やその物理が万物の理論から得られるはずだ、というのがまさに幻想なわけです
実際には誰も成功していません
0580ご冗談でしょう?名無しさん
垢版 |
2021/08/17(火) 16:22:19.11ID:AVTgkHIq
>>579
これ以上は各々の信念の問題であり、なおかつ私の質問からも逸れていきますので深掘りはいたしませんが、
現在成功していないことをもって幻想と切り捨てることはできません。
たとえば原子核はフェルミ流体で有効的に記述できますが、それもまたQCDから第一原理計算されなければなりません。
現在QCDから第一原理計算できつつある核子数はせいぜいリチウムが関の山らしいので、
もちろん多体系の第一原理計算が非常に難しいのは事実でしょうが、少なくとも幻想などではないのです。

誤解のないよう付け加えておくと、決して私は有効理論が理論として無価値などとは思っていません。
第一原理から膨大な計算の末にIR固定点を特定する、というのを毎度毎度やるのは馬鹿げていますし、
そういう意味では階層ごとに固有の有効理論を研究することには多いに意味があります。
しかし、物理学の体系としては、よりマイクロスコピックな理論から演繹されなければならない、というだけです。
それこそまさに統計をやっておられる方々の究極の目標なのだと思いますが。
0581ご冗談でしょう?名無しさん
垢版 |
2021/08/17(火) 16:42:31.65ID:???
>>580
第一原理計算は、無限の計算資源があれば有効理論や現象論の結果を再現し得ますが、これは例えばフェルミ液体論という理論そのものを導くことができるわけではありません
0582ご冗談でしょう?名無しさん
垢版 |
2021/08/17(火) 17:01:13.34ID:AVTgkHIq
>>581
むしろ無限の計算資源などなくてもよい、というのが、まさに階層とかMore is different.とかの考え方なんじゃないですかね?
なにかマクロな系をシミュレーションするにあたり、そのサイズすべてを計算できるほどの資源は必要なく、
たかだかその系の相関長くらいのシミュレーションさえできれば十分なわけです。
0583ご冗談でしょう?名無しさん
垢版 |
2021/08/17(火) 17:45:58.38ID:cYaBgmCN
>>563
>せん断耐力について

何万トンもの力を加えても、キムヨナの太腿をせん断することはできないよ。
0584ご冗談でしょう?名無しさん
垢版 |
2021/08/17(火) 19:17:54.62ID:???
阿蘇の史(さかん)
@asonosakan
返信先:
@asonosakan
さん
物理学者のNicholas Gisinは,実数を使う代わりに「有限の情報」しか持たない数を使う,オルタナティブな古典力学の体系を提唱している.この古典力学では,方程式が決定論的であるにもかかわらず,初期値を指定できる精度に限界があるため,カオス系が真に非決定論的になる.https://archive-ouverte.unige.ch/unige:131921
午後1:27 ・ 2021年8月17日・Twitter for Android
0586ご冗談でしょう?名無しさん
垢版 |
2021/08/17(火) 22:46:24.04ID:ESyeDwx+
またみんなで寄ってタカって難しいハナシしてる…🤣🤓🤡🤢(´Д`●)


しかし俺はハゲではない…🤠
0589ご冗談でしょう?名無しさん
垢版 |
2021/08/18(水) 02:02:04.96ID:VXXxtMAq
>>489
>>493 見たか聞いたか?ええ解説や思うぞ。🤓🤡🤢(´ω`🌀)
俺様は初心者やさかい、url の取得方とか分からんのだ。
0591ご冗談でしょう?名無しさん
垢版 |
2021/08/18(水) 09:46:08.90ID:n57HMHC7
美少年😁🤣😍💚🧡💜❗
0593ご冗談でしょう?名無しさん
垢版 |
2021/08/18(水) 09:57:44.56ID:???
ビッグバンはあったという証拠(確度はともかく)ってもう見つかってますっけ?
それともまだインフレーションがあるとすると辻褄が合うの段階にとどまってる?
0595太上天君
垢版 |
2021/08/18(水) 14:08:52.07ID:n57HMHC7
>>593
証拠は見つかっていないが、辻褄が合うってことが証拠になるんだよwwwwww
0596ご冗談でしょう?名無しさん
垢版 |
2021/08/18(水) 19:05:45.64ID:5g0j6UJ9
>>593
ビグバンのハナシか🤓🤡🤢(´ω`🌀)
インフレーションのハナシか
日本の景気浮揚のハナシか
どっちなん?

>>590
ほーらついに賛同者があらわれた!(どらくえのテーマ)♪
0598ご冗談でしょう?名無しさん
垢版 |
2021/08/18(水) 19:42:03.42ID:/KBwUSOm
あちらこちらで聞いて調べても、回答がまちまちではっきりしなかったので教えてください

部屋の中に20度の水が入ったコップがあります
室温が25℃、30℃、35℃の時、コップの水が25℃になる時の速さに違いがあるかないかを教えてください
風とか日光の影響はなしとします

個人的には全部一緒だと思うのですが…上手いこと証明できなくて
よろしくお願いします
0599ご冗談でしょう?名無しさん
垢版 |
2021/08/18(水) 19:58:09.48ID:???
>>598
あちらこちらって具体的にどこなんだろ
そこでどういう答が返ってきてID:/KBwUSOmがそれにどういう対応をしたかは重要だと思う
ID:/KBwUSOmは、あちらこちらで返ってきた答では満足できないってことで
同様の答がこのスレで返ってきても満足できないだろうから
0600太上天君
垢版 |
2021/08/18(水) 20:03:24.64ID:ldTdAPt9
宇宙には始まりも終わりもない。
あるのは途中経過だけ。
途中経過を語るのが科学だよ。

科学で始まりや終わりを語ることはできないよwwwwwwwwwwwww
0601ご冗談でしょう?名無しさん
垢版 |
2021/08/18(水) 20:15:01.75ID:???
原子核の周りを周る電子の軌道は円軌道であるのに
電子雲はどうして球のような形をしているのですか?
0603太上天君
垢版 |
2021/08/18(水) 20:19:46.02ID:ldTdAPt9
>>601
宇宙には最大も最小もない。
あるのは中途半端な大きさだけ。
中途半端な大きさを語るのが科学だよ。

科学で最大や最小を語ることはできないよwwwwwwwwwwwww
0604ご冗談でしょう?名無しさん
垢版 |
2021/08/18(水) 20:53:02.15ID:???
>>601
球の電子雲は s 軌道だけ
s 軌道は軌道角運動量が 0 だから、そもそも回ってない
電子が等方的に存在してるから球にしかならん
0608ご冗談でしょう?名無しさん
垢版 |
2021/08/18(水) 21:57:10.71ID:???
>>604
なるほど
「電子の軌道は円軌道」っていうのがそもそも違うってことで合ってますか?
ラザフォードが実験してボーアが原子模型を作って…という時系列を学んだばかりだったので
その時の理解とごっちゃになっていたかもしれません
0609ご冗談でしょう?名無しさん
垢版 |
2021/08/18(水) 22:05:39.85ID:???
>>604
横だけど
ボーアモデルのs 軌道の角運動量はℏになるが
量子力学のs 軌道の角運動量0と対応しないようにみえる。
矛盾が無いのか教えて
0618ご冗談でしょう?名無しさん
垢版 |
2021/08/19(木) 17:34:33.45ID:tVxGR35V
>>585
旦さん、ども長いことお待たせしましたどすなぁ🤣🙄🤡🤢(´ω`🌀)

松浦壮「量子とはなんだろう」講談社BluBacks
p.197 嫁😹
0619ご冗談でしょう?名無しさん
垢版 |
2021/08/19(木) 17:35:25.76ID:???
>>612
>ボーア模型における軌道の概念は間違っていた

水素原子の基底エネルギーがボーアモデルと量子力学で殆ど同じになる
電子の軌道角運動量がℏと0で全く違うのに同じエネルギーになる理由は?
0620ご冗談でしょう?名無しさん
垢版 |
2021/08/19(木) 17:39:03.57ID:Zay1/92R
>>609
軸がランダムな方向を向いてたら、平均的にはどうなる?
0626ご冗談でしょう?名無しさん
垢版 |
2021/08/19(木) 21:10:52.79ID:???
原子に古典軌道がなくても、角運動量がゼロでなければ外部磁場等により
エネルギーの違いで分裂する。
0631ご冗談でしょう?名無しさん
垢版 |
2021/08/20(金) 00:10:22.08ID:???
縮退してる状態を適当に抜き出してそれがもとの対称性を持たないと主張することに物理的意味はない
0632ご冗談でしょう?名無しさん
垢版 |
2021/08/20(金) 00:39:38.39ID:inW33Pt2
だいたいさぁ、謎のフタコト、ミコトのやり取りなんかで🙄🤡🤢(´🍙`🔴)
語れる物理学がホントにあるんならぁ、お目にかかりてぇ
もんだ!🤣
0635ご冗談でしょう?名無しさん
垢版 |
2021/08/20(金) 01:04:21.38ID:inW33Pt2
>>633
オマエさぁ、もしかして日常的にキモチワルイ
言われてんとチガウ?ヾ(´・ω・`)
いやさ、実は俺テレビに向かって、ダイアン津田
とか相手にキモチワルイ言うのがどうやら癖に
なってるようで、いったいどこでそんな呪い受けてきた
のか?…とつらつら考えるに、オマエしか思いあたらひんのや。
すると、オマエ→俺→と、呪いの連鎖が…恐ろしい、恐ろしいぞぉ。🙆
0637poem
垢版 |
2021/08/20(金) 04:44:38.35ID:???
唐突なキチガイ

人間の体〜脳とか〜

左脳、右脳、小脳、間脳延髄脊髄、神経網、RNA機構とDNA核、細胞
って別けられて

左脳(6次元か5次元)、右脳(6次元か5次元)、小脳(4次元)、間脳延髄脊髄(3次元)、神経網(2次元)、RNA機構とDNA核(1次元)、細胞(0次元)

0次元…X^0=1(実数X)or0(非数X)…ªº←とかな計算記号で、ªX=1、ºX=0(←実数にしたら)って感じどうかな

0次元はªXの1項の中の数値は自由(ただし実数にしたら1)、そして最大項数は上の1次元…+1、つまり1項のみしか許容しない。これは私(一人称)を表す…細胞

1次元は+X−X、1項の中の数値は自由(これが普通の数。数直線の中の)、そして最大項数は2次元の×2、つまり2項を許容する…これは貴方と私の関係(二人称)、2項なら貴方と私、1項なら私(周りに対する)…RNA機構とDNA核(DNAは有機コンピューターであり、生理的な(細胞の(原始的な))思考体、苦しいこと辛いことに晒されるとDNAは処理される)

つづく
0638poem
垢版 |
2021/08/20(金) 04:57:17.42ID:???
2次元は×X÷X、これは空間
かけ算層が空間
(べき算層が空間というのは間違い)
空間は3次元空間、^3だが、空間があるのは×X÷Xこれは距離、空間的拡がりが倍数的(拡大縮小、縮尺)(体積を持つ物の個数で距離が測れる)で、自由な距離×Xの最大項数が3項だから^3…3次元空間。これは三人称、彼、貴方、私を表す。神経網は体に枝を伸ばす空間的な細胞組織。

それと物理量が乗関数、かけ算わり算なのは空間変化を記述したい、時間的な変化は記述してない(言ったことは間違ってた)
0639poem
垢版 |
2021/08/20(金) 05:20:31.95ID:???
3次元は^X、(X)√
これは4人称
方向を自由に取れる(3次元方向でも4次元方向でもn次元(注意:^的n次元であり、今メインで語ってる計算階と最大項数のn次元とは異なる))
つまりこの見える3次元空間の外、見えない次元を自由に取り、それとの方向的な物理相互作用
最大項数は4。⌒4。○^^^^。
これはkgかなって思ったりするけど間違ってる可能性高し。わからない
間脳延髄脊髄は方向を思考し設定し、神経網(空間的)に伝えて空間的動作を行わせるんじゃないかな
3次元^√が質量と考えるわけ

m:sの比が速度系、1:1なら速度、2:2、n:nでも
1:2が加速度2:4でも1/2:1でも
2:1が減速度1:1/2でも4:2でも

m/s、(m/s)^2、(m/s)^x…速度
m^2/s^4、m/s^2、m^(1/2)/s…加速度
m^4/s^2、m^2/s、m/s^(1/2)…減速度

このm:sの比によって、速度の効果、時間変化率が変わるが
ではこの^数は時間なのかというと、時間変化「率」であって、時間操作(時間の伸び縮み)ではない。あくまで時間の「比」でしかない
^√は時間ではない
けど、質量は?
質量比…(もとい通常の物理で、質量は数直線の原点から片側しか許容しないから─(4kgと1kgの鉄球を衝突させて、4kgは1kgの速度が質量に分散するが、1kgは4kgの速度を受けて4倍の速度にはならない)─から質量という狭義ではなく)…ではなく質量と反質量を含めた運動係数みたいなものを、「時間変化率」と同一視できないかなと考えたのだが

間違ってる可能性高し
0640poem
垢版 |
2021/08/20(金) 05:40:23.08ID:???
4次元は⌒、?
これが時間じゃないかなって
小脳(自我)

質量が3次元の^√
空間が2次元の×÷、時間が4次元の⌒?
だと
電気と磁気は時間と空間の物理量を交換したものだ
電気(kg^x)(s^y)と磁気(kg^x)(m^y)
って対称になるから、こうなれば綺麗だなって

最大項数は5。○⌒⌒⌒⌒⌒。

あと、
時間が1次元に進む(パッシブ1次元な)訳whith質量

^√の範囲は
∞(^∞)←x(^x・(x)√=x)→1((∞)√)
⌒?の範囲は
∞(⌒∞)←?(⌒x・(X)?)→x((∞)?)

3次元(√)の最小が1
これは計算階の0次元、X^0=1(実数X)or0(非数X)にあたる
3次元(質量?)は底((∞)√)が0次元で、進行はない「位置」…つまり質量は安置される
4次元(⌒?)は最小がx、つまり4次元(たぶん時間)の底((∞)?)が1次元(+−)に相当するので、時間はパッシブに進行する
のだと思っている

空間は底が0だから0次元以下
空間は失われる
0641poem
垢版 |
2021/08/20(金) 05:51:49.67ID:???
5次元
6次元
わからん
けど
パッシブ2次元
パッシブ3次元
歪み(変形進行)

荒らしピンポンダッシュ完了
さるるるるる
0642poem
垢版 |
2021/08/20(金) 06:33:03.00ID:???
彡さっ
エネルギーはべき関数3次元^√
エネルギーは質量について記述したい
すっミ
0647poem
垢版 |
2021/08/20(金) 16:44:41.26ID:???
>>646
量間…確率で出現・ランダムエンカウント
相間…装置との距離で出現・イベントエンカウント
メコスジ…大人になったら出遭う・ストーリーエンカウント
0648poem
垢版 |
2021/08/20(金) 16:53:16.62ID:???
荒らし…そこら辺を徘徊・シンボルエンカウント(アクティブ・ノンアクティブ)

ノンアクティブ…特定の流れに反応
アクティブ…スレ開いたら書き込む
レイド…思い付いたとき脈絡なく爆
0649poem
垢版 |
2021/08/20(金) 16:57:02.27ID:???
647はみすった
648と書き方合わない
0651ご冗談でしょう?名無しさん
垢版 |
2021/08/20(金) 20:00:48.22ID:1ldxP6S4
大きい動物の動作はゆっくり、小動物は素早く動くのは重力のせいだ。

つまり大きい動物は自分の体重が重すぎて速く動けないわけだ。

だから月でなら、アフリカ象もうさぎのようにすばしっこく走り回ることができるはずだwww
0652ご冗談でしょう?名無しさん
垢版 |
2021/08/20(金) 20:12:20.07ID:???
粒子・量子
未来・過去

粒子の過去改変機構─量子の確率的存在・不確定性

量子の未来改変機構→粒子の物質的振る舞い・量子現象の物質性

質問
確率現象が過去改変機構だと想定すると量子論の確率論は具体的にどんなことが起こってるだろうか?

顕微鏡でミレナイ
0653ご冗談でしょう?名無しさん
垢版 |
2021/08/20(金) 20:22:44.73ID:???
現在・過去・未来〜♪
そうか!まず現在から過去を振り返り、それから未来を予測することが大切なんだ!
これで吉野さんが電池の研究をすることにしたのだとか
0654ご冗談でしょう?名無しさん
垢版 |
2021/08/20(金) 21:47:11.61ID:sAzd5vXp
そして3ヶ月後、>>644 の姿は…🙄🤡🤢(´ω`🌀)
0656ご冗談でしょう?名無しさん
垢版 |
2021/08/21(土) 13:55:22.38ID:1gqsA/Eg
銀河って重力の引力と電気の斥力を使って計算すると回転問題がありえないし崩壊してしまうんでしょ?
空間的な引力斥力だけじゃなくて時間的な遠隔作用とか他の遠隔作用とかが銀河の保持に関わってるとして含めて計算すると崩壊しないしダークマターもいらないのでは?
陽子が集まると電気的な斥?力
電子が集まると電気的な斥力
物質が集まると重力的な引力
中性子が集まると謎的な謎力
磁粒子が集まると磁的な謎力
素粒子が集まると微力な無力
銀河は糊付けされて回転してるんだよ
0657ご冗談でしょう?名無しさん
垢版 |
2021/08/21(土) 13:57:25.82ID:1gqsA/Eg
ダークマターって必要?
0659ご冗談でしょう?名無しさん
垢版 |
2021/08/21(土) 16:26:57.32ID:???
>>657
遠方にある他の銀河系の運動、回転運動をその銀河からの”電磁波で観測”すると
観測された恒星、ガスの質量で重力や一般相対論からの計算値と一致しない。

”電磁波で観測できない質量物質=ダークマター”の分布を仮定すれば計算と合う。
電磁波を殆ど放射しない質量物質の存在を仮定しても現代物理学に矛盾が無い。
謎の引力、謎の遠隔作用とかは太陽系内で現在も発見されてないから法則もない
ダークマターが無い証拠がそろえば、検討するかもしれない。
現代物理学で説明出来ない証拠がそろえば検討する、超常現象の類と同じ扱い。
なぜかトンデモはその類の説に最初から飛びつく。
0660ご冗談でしょう?名無しさん
垢版 |
2021/08/21(土) 17:04:31.85ID:???
スレタイ(すれたい)を読(よ)める人(ひと)だけ
このスレ(すれ)に書(か)き込(こ)んでください。
0661ご冗談でしょう?名無しさん
垢版 |
2021/08/21(土) 17:06:14.03ID:WMMOwXN+
ところで初歩的な質問なのでが、
抵抗の単位Ohm-cmのcmというものはどのように解釈したらいいのでしょうか。
1pごとに1OhmならばOhm/cmですよね。
Ohm×pとはどんな意味ですか。
0662ご冗談でしょう?名無しさん
垢版 |
2021/08/21(土) 17:24:44.45ID:???
>>661
>Ohm-cm
それは 電気抵抗率ρ の単位
ρ = R・A/L A: 断面積 L:長さ
例:導線の抵抗率1、断面積1、長さ1ならば、(その方向の)導線の抵抗は1Ω
0663ご冗談でしょう?名無しさん
垢版 |
2021/08/21(土) 17:39:33.46ID:???
>>662
市販の導線の規格表には、電気抵抗率ρ断面積Aが載ってる
使う用途で長さLが決まれば導線長の抵抗が求まる、測定する必要が無い。
便利だろ
0664ご冗談でしょう?名無しさん
垢版 |
2021/08/21(土) 19:01:05.12ID:???
>>659
空間的な位置取り…引力斥力
時間的な位置取りは?…物理変化(時間変化)速度の変化。…入力された物理への異常レスポンス(同じ質量がある引力で片方だけ速く片方だけ遅く引き合う、とか…留まる(時間反応性の変化))

中性子は本当に中性で、周りに遠隔力を何も及ぼさない、相互作用しない、置物のような、中性なのか
陽子・電子のように引力斥力を生じる=空間的な遠隔力があるものと異なる、
空間ではない(時間などの)遠隔相互作用を何一つ持たず中性なのか
時間的遠隔力を持つ場合、中性子は引き合わないし斥けあわない。引力斥力は持たない。空間的な位置取りはしない。
引力斥力では観測できない

現代科学は空間的な相互作用しか観測できないんじゃないか

中性子は中性というのは現代科学で相互作用を観測できないからでないか
0665ご冗談でしょう?名無しさん
垢版 |
2021/08/21(土) 19:02:12.09ID:???
時間的な位置取り、物理レスポンスの変化、留まる留まらない
他の空間・時間以外の物理機能、遠隔力も存在するのではないか
空間・時間以外なら、質量の遠隔力も空間・時間の遠隔力と同じように存在するはず。観測できない。その質量の遠隔作用には引力斥力も発生しない。異なる機能

銀河の回転問題(ダークマターを必要とする問題)にそういった未知の空間(引力斥力)以外の遠隔力は存在しないのか
空間的な位置取り以外にこの宇宙には現代科学で観測できない、他の物理機能の位置取りは実在せずトンデモで、ありえない、オカルトなのか
ダークマターもまたオカルトだとは思わないのか。見えないものが目の前に漂ってるとか幽霊ですか?
幽霊とダークマターの違いを教えて、ダークマターがオカルトじゃない理由を説明して?
中性子の中性ではない遠隔力、陽子・中性子が共に持つ、質量を持つ物の(引力斥力でない)遠隔力なども、銀河を橋渡ししていて、色々な物理的機能、遠隔力で糊付けされてて、今の科学では中性子の相互作用を観測できないのと同様に銀河の糊付けを観測できず
ダークマターのような幽霊を信じちゃってるのが
現代の科学者でダークマター予想ではないのか?
0666ご冗談でしょう?名無しさん
垢版 |
2021/08/21(土) 19:03:37.83ID:???
うっわ乱文過ぎる。少し離れてると乱文になるな

で、>>659>>658 この可能性はない?幽霊信じるの?
今の科学で発見できてない相互作用はなく、相互作用は空間的な相互作用しかなく、空間的な位置取りで全て引力斥力で観測できて、他の物理機能は実在しなく、他の物理機能も引力斥力で、引力斥力を全て観測できるから、他の物理機能は実在しなく、現代科学はダークマター以外全て観測できてて、現代科学は全て正しく、キミも正しいの?
0668ご冗談でしょう?名無しさん
垢版 |
2021/08/21(土) 20:34:51.47ID:???
>>665
黒い背景で黒い球は見えないだろ、そんなことも判らん馬鹿か
遥か遠方にある物質が電磁波(光)を放射、反射しなければ見えない。

遠方の恒星を回る惑星は直接見えないが、恒星からの光の変化を詳しく観測することで
惑星の質量や軌道が計算できる。

銀河の見えない質量物質=ダークマターも同様、おかしな推定は何もない。
現在でも判らないのは、ダークマターの主な構成物質が何か?ということ。
0670ご冗談でしょう?名無しさん
垢版 |
2021/08/21(土) 21:00:33.09ID:???
単純な疑問だけど物理学で階層性が何故成り立つのかを解説した文章ってある?
論文でも教科書でもブルーバックスでもなんでもいいけどあるなら知りたい

ソフトウェアじゃないから原子を論じるときにクオークの存在は考えなくてもいいと出来るのは都合が良すぎると感じるんだよね
0672ご冗談でしょう?名無しさん
垢版 |
2021/08/21(土) 21:23:09.69ID:???
>>671
文字通り桁が違うから問題にならないって考え方にはあまり賛同できないなあ
せめてそのスケールでは測定できないほど小さいから階層性が成立するとか測定理論に基づく厳密な整備とかないの
0673ご冗談でしょう?名無しさん
垢版 |
2021/08/21(土) 21:25:54.49ID:???
>>670
>原子を論じるときにクオークの存在は考えなくてもいい

クオーク同士(または核子同士)に作用する強い力の到達距離は10^-15m以下
原子核より小さい範囲。 電子(レプトン)には作用しない。
素粒子同士の重力も無視でき、原子構造は原子核と電子の電磁気力だけで説明できる。
0674ご冗談でしょう?名無しさん
垢版 |
2021/08/21(土) 21:28:31.67ID:???
>>673
何で自然がそんなに都合良く出来てるのっていう疑問には答えていないと思いますが…
ただこれを物理の疑問じゃないと言われたら反論はできない

例えば量子重力理論ができたとしてもそれが日常のスケールで問題になることはないっぽいということですよね
0675ご冗談でしょう?名無しさん
垢版 |
2021/08/21(土) 21:28:35.98ID:???
>>672
それぞれの理論のエネルギースケールはあるわけで
どれだけの桁開いていればいいかは知らんけど全然違うスケール見るなら関係ないよね
0676ご冗談でしょう?名無しさん
垢版 |
2021/08/21(土) 21:29:56.49ID:???
>>674
エネルギースケールが違う物理だったらなぜ関係ないかでなく

なんでエネルギースケールが違うのかの話?
そんなん今のメイントピックやん
0677ご冗談でしょう?名無しさん
垢版 |
2021/08/21(土) 21:39:07.11ID:???
>>674
>何で自然がそんなに都合良く出来てるの

人間原理で納得するしかないな

物理学は始めに有る自然現象を数学的に説明する学問だからね。
0682ご冗談でしょう?名無しさん
垢版 |
2021/08/22(日) 00:29:10.60ID:???
>>668
ダークマターって地球上の地上付近、目の前にもあるっていうよね
黒い背景で黒い玉は見えない?
電磁波反射せずに黒い玉があるなら衝突するよね?目の前に黒い塊あるよね?でもないよね?幽霊じゃんそれ?顕微鏡でも見えるはずだよね?電磁波放射しない電磁的相互作用しない未知の粒子の玉か素粒子の粒が?少なくとも銀河と相互作用するなら見える粒子と相互作用するはずだから顕微鏡でわかるよね?なにかあるって?目の前にあるんだよね?
目の前にも普通に漂って銀河には普遍的にあって銀河の引力を助けるんだよね?
で幽霊は見つけられるのかな?それとも銀河と同じサイズの丸い玉で、小さい人間は通り抜けて、銀河は通り抜けない、ダークマターは素粒子じゃなく、銀河と同じサイズの超絶巨大なサイズの単素粒子(ミクロで網目の広いザルのように希薄)ですかね?

超巨大な幽霊探してるの?
それとも目の前にも黒いミクロの粒があるって?
ミクロの粒はこんな科学が発達した顕微鏡で観測できない時点で希望薄だよね?それとも幽霊と友達になりたいの?マンキンの巫術でも使いたいの?
超巨大な特大単素粒子なら幽霊じゃなくて実体の希望ありだけど、望遠鏡で銀河見て、いくら電磁波反射しないからといっても、銀河がブラウン運動してなければやっぱり希望薄じゃないかな

今の科学者は幽霊学者名乗る?
0683ご冗談でしょう?名無しさん
垢版 |
2021/08/22(日) 03:27:56.66ID:???
観測できるためには測定器と相互作用してくれなくてはならない。
相互作用が弱ければ目の前にあっても見づらい。ニュートリノなんてその典型。
カミオカンデ並みの物量の測定器でようやく検出可能な程度に相互作用してくれて見えるようになる。
ダークマターはさらに相互作用が弱い。銀河並みの物量でなくては検出可能なほどには相互作用しない。
それだけのこと

仮に相互作用が弱いものは何でもかんでも幽霊と名付けるなら、確かにダークマターは幽霊だな。
で、それが何か問題でも?
実際ニュートリノを幽霊粒子と比喩的に呼ぶこともあるしな。
0686ご冗談でしょう?名無しさん
垢版 |
2021/08/22(日) 09:34:54.43ID:???
銀河から来る物質の光による観測だけに頼る銀河の質量不足だけの話ならば、
観測できない黒い塵などの物質が銀河に分布してると仮定しても事足りる。
ダークマター=普通の物質 で一般人(古典論者)は何も困らない。

ダークマターを仮定する必要があるのはビッグバン仮説との整合性のためだ
ビッグバン仮説によれば、黒い普通の物質(バリオン)が大量あると理論上
問題。
ダークマター=普通の物質 だとビッグバン仮説を主張する物理学者が困る。
そんだけ
0687ご冗談でしょう?名無しさん
垢版 |
2021/08/22(日) 10:11:59.42ID:???
>>686
ダークマターの銀河分布が恒星がある渦巻には無くてもそれほど問題にならない。
銀河の渦巻より何十倍も遠い(体積なら1000倍以上)領域の発光ガスの運動観測で
重力不足と計算される。
見える渦巻銀河の1000倍以上の体積領域に(黒い)普通物質が漂ってもおかしくない.


宇宙物理学者の殆どがビッグバン仮説を主張してるから、謎のダークマターが必要になる。
0689ご冗談でしょう?名無しさん
垢版 |
2021/08/22(日) 11:39:13.51ID:???
階層性の話は違う話が混じってるな
なぜ階層があるかってのは人間の都合だし
階層が違うと無視して良いてのは結果論だ
微小でも無視できない事はあるから
無視しても結果が合うことだけ無視してるだけだし
なぜ階層をつけて無視するかと言えば
手間を省くためだ
0690ご冗談でしょう?名無しさん
垢版 |
2021/08/22(日) 12:51:58.29ID:???
>>683
で?相互作用が希薄なダークマターが?それとニュートリノも?銀河をつなぎ止める糊として充分力を持つとでも?

(ダークマター・ニュートリノ)微かな力も塵も積もれば何とやらですか?
物質の方が塵も積もったとき力が大きくなって、その力の差の数値は開くと思うんですけどね
0692ご冗談でしょう?名無しさん
垢版 |
2021/08/22(日) 13:32:56.53ID:???
>>686
>>667
銀河の質量不充分は光を発しない惑星、冷たい塵、で大方詰められて、あとは未知の惑星、塵が地球から遠いところの銀河の内か外にあれば、質量の計算うまくいくかも知れないのか。
へぇー
質量不充分はむしろ問題でなく、ビッグバン理論との整合性でダークマターが提唱されてるのか
そっちはわかんないから、むり

ただ、銀河質量不充分が惑星・塵を含めれば足りるかもって言うのも
もちろん惑星・塵の総量は今モデリングされてるより多いだろうけど、
もしそれで質量不充分でも、また今のモデリングであっても
中性子の相互作用が微か過ぎて観測できないから中性とか言わず、中性子は引力斥力を生じない=空間的な相互作用しない
中性子がするのは時間的な相互作用…とかかも?
とか見方を変えて
観測方法を空間的に作用反作用を及ぼす力ではなく、時間的に物理の性質が変わるのを、物理を入力して応答の予想と実測とのズレにより算出するとか、
今の科学では観測できないことがある、知らないことがある、相互作用は作用反作用だけでなく物理応答や他の物理機能も存在し、今の観測機でそれを測れない、中性子は微かな力で中性なんかじゃなく観測方法がわからない大きな力を持つと認め、
重力・電磁気力・強い力・弱い力が正しいという神話は捨て
空間の力・時間の力・質量の力・何の力・他の力・また他の力
という分類に改革し
0693ご冗談でしょう?名無しさん
垢版 |
2021/08/22(日) 13:34:01.12ID:???
改革し例えば
・重力=空間的
・電磁気力=空間的(ただし電気力と磁気力は分けるべきで、磁気が空間力なら、電気が磁気力。電磁波は空間と時間の力両方持つから合成)
→重力と磁気力は習合する
・弱い力と電磁気力は同じっていうよね?
・核力は陽子と中性子の力だから、空間と時間の力
→電気力・磁気力と核力の習合

重力、電磁気力、核力、弱い力は殆ど融合する

・クォークの力とか
・素粒子の力とか
→新たな力だし(微かな力でない)
・質量自体の力も別にある(重力とは異なる。重力は引力を発生する時点で空間の力)

例えはまあいいけど
銀河の糊付けにそんな観測方法がわからない空間以外の物理機能の遠隔力を想定すれば、
今の観測してる質量不充分でも充分つなぎ止めてられる

っていう風に銀河の質量不充分問題とビッグバン理論問題
ビッグバン理論は知らない
質量不充分問題は観測方法のわからない力を含めれば今の質量で問題ない。銀河の糊付けの主なファクターは引力(重力の)だけでなく、未知の力で、今観測されてる質量に重ねて未知の力を組み込めば、計算合う。ダークマターは必要ない
0694ご冗談でしょう?名無しさん
垢版 |
2021/08/22(日) 13:38:13.94ID:???
>ただし電気力と磁気力は分けるべきで、磁気が空間力なら、電気が磁気力。

ただし電気力と磁気力は分けるべきで、磁気が空間力なら、電気が「時間力」。
0695ご冗談でしょう?名無しさん
垢版 |
2021/08/22(日) 14:31:11.75ID:???
電気と磁気はローレンツ変換によって混合しますので
分けるべきか分けるべきでないかで言えば分けるべきではありません
何言ってんのか分からないので全部読んでませんけども取り急ぎツッコミ
0696ご冗談でしょう?名無しさん
垢版 |
2021/08/22(日) 14:35:36.01ID:???
>>693
>質量に重ねて未知の力を組み込めば、計算合う。

物理学では「未知の力」とかは何でもありの意味でしかなく、誰にも相手にされずゴミ箱。

湯川秀樹博士は当時、未知の力だった原子核の力を量子論を基に中間子論をつくり
中間子の質量を計算し実験と一致したから、他の物理学者に認められたのだよ。

キミの説も認められたかったらそこまで努力するんだな。
0700ご冗談でしょう?名無しさん
垢版 |
2021/08/22(日) 17:24:15.93ID:???
原子核の崩壊で、水素原子核(陽子)とヘリウム原子核(ALPHA粒子)が出てくるのに、次のリチウム原子核が出て来ないのはなぜですか?
0702ご冗談でしょう?名無しさん
垢版 |
2021/08/22(日) 18:14:42.46ID:V0EzQ6WG
>>696
未知の力については具体的に機能を説明している。空間の力「引力斥力」がm^±n or ±1/n、時間の力「留まる、留まらない(レスポンス)」はs^±n or ±1/n、質量の力は「引力ではない(引力は作用反作用で空間)」kg^±n or ±1/n
って明言してて、「未知の力」=幽霊って言ってるわけではないんだよ。そっちのダークマターは「未知の粒子」=幽霊って言ってるけど

科学者じゃないから湯川って人ほどの努力は無理。なんならキミほどの努力も無理。このスレの中で1番努力してない、努力できない自覚はある。けど科学者の仕事だよね?キミたちの。先人や現行の鵜呑みにして、そこに足し算して雑なもの付け足していって先人や現行のものを否定したら頭悪いって思い込んで、迎合しなければならず理論に毛がモサモサ生えて複雑化しかかできないキミたちは、先人や現行を審議し精査しときに否定して引き算できる力を養わなきゃ

幽霊学者
似非科学者との違いは迎合力だけで根っこは同じ
0703ご冗談でしょう?名無しさん
垢版 |
2021/08/22(日) 18:29:47.68ID:???
>>695
ちなみに陽子と電子、電気は単極子が存在し双極子は存在しない
磁力はNとSの双極子しか存在せず、単極子が存在するわけって簡単なんだよ

双極子っていうのは方向がある。矢印がある
分離できないってのは分離できない理由がある。NとSの2点は直線を作る。
速度は切り分けて、近づくものと遠ざかるものに分離できるか?
双極子であるもの、2点で線となる性質のあるものは速度を持つ。
磁力は
磁気単極子に速度を持つから双極子しかない
そして磁気双極子は空間の力、空間の作用反作用、空間の位置取り、
引力斥力を持つ

じゃあ陽子・電子は?、電気単極子だと思われてるこの2つは?引力斥力を持つこれらは?
磁気と電気は違うなら、陽子と電子は引力斥力でなく「留まる・留まらない」など別の力を持たなければならない。引力斥力は持たない
電気と磁気が同じものなら、空間の力の単極子を電気、双極子を磁気という
つまり、陽子・電子は空間の力を持ち「引力斥力」、磁気は陽子・電子の空間的な力に速度を持った(スピンでよい)「方向(極性)を持った引力斥力」となる

これって陽子・電子は磁気単極子と同じではないか。そのものではないか。

電気と磁気と同じもの?
陽子・電子と電気と思っているものが実は磁気で
磁気単極子と磁気双極子を同じものと言ってるだけ。
ローレンツ変換?単極子の運動による双極子はローレンツ変換とか言うのじゃないの?

中性子が真の電気力、時間の力
磁子(真の電荷力)も観測できないが実在する
中性子を微かな力で中性といってる今の科学力で検出できない
0704ご冗談でしょう?名無しさん
垢版 |
2021/08/22(日) 18:31:50.83ID:V0EzQ6WG
>磁力はNとSの双極子しか存在せず、単極子が存在するわけって簡単なんだよ

磁力はNとSの双極子しか存在せず、単極子が存在しないわけって簡単なんだよ
0706poem
垢版 |
2021/08/22(日) 18:42:19.21ID:???
ちなみに重力の起源は、星の中に凝縮した陽子の塊だと思っている

陽子は原子核を作る。そこに働くのは斥力であってはならない。引力でなければならない。

電子は斥力、斥けあう位置取り
電子と陽子は互いの位置取り、つかず離れず
陽子は引力、引き合う位置取り

星の核には陽子過剰か裸の層がある

集まれば集まるだけ、強くなる
電気的に中性の普通の原子を引きつけるほどに

重力は磁気(単極子)力

kg^x・m^2以上なら相対力は1/m^1以上

1/mは距離が縮まる
引力
0708ご冗談でしょう?名無しさん
垢版 |
2021/08/22(日) 18:45:52.46ID:???
>>705
鵜呑みにするだけで頭悪いって言われない?

そこまで難しい?この文章?

方向があるものは双極子って言ってるだけなんだけど、それすらも理解できない?
0711ご冗談でしょう?名無しさん
垢版 |
2021/08/22(日) 18:49:42.28ID:V0EzQ6WG
>>709
よくわかんないけど式建ててみたら計算あった!天動説は正しいんだ!ぼくは定量的に測定量を論じられた!
に読めた
0712ご冗談でしょう?名無しさん
垢版 |
2021/08/22(日) 18:53:21.99ID:V0EzQ6WG
今の物理は天動説
複雑な都合定数・係数・式の組立、計算手法
ご都合主義
ゴミゴミした毛の生えたゴミ
足し算で継ぎ足していったなれの果て
先人や現行を引き算できない
複雑な天動説

裸の地動説の地肌は出ない
0713ご冗談でしょう?名無しさん
垢版 |
2021/08/22(日) 18:55:33.84ID:V0EzQ6WG
鵜呑みにする
このすきるは
かがくしゃに
ひっすです!
0714ご冗談でしょう?名無しさん
垢版 |
2021/08/22(日) 18:56:29.75ID:???
>>711
今の物理は
天動説と地動説がどういう機序で互い説明できるかを定量的に説明できます。
定量的に論じられない妄想は物理板ではなく哲学板かメンタルヘルス板でやるべきです。
0717ご冗談でしょう?名無しさん
垢版 |
2021/08/22(日) 19:20:32.70ID:???
ここが俺の居場所でないということだけは分かった
物理学ってのはアホが空想して悦に浸るための道具じゃねえんだわ
じゃあなゴミども
0721ご冗談でしょう?名無しさん
垢版 |
2021/08/22(日) 19:37:44.40ID:???
というか一般論として文系の抱く数学や科学は圧倒的に正しいって思想一体何?
だから正しいことが保証されている数学や科学を援用すれば文系の研究も正しさが担保されるみたいなやり方一体何
0722poem
垢版 |
2021/08/22(日) 19:51:37.93ID:???
>>714
量子論が天動説でなく地動説って言うのをキミの定量的?式で証明してみて
0723ご冗談でしょう?名無しさん
垢版 |
2021/08/22(日) 19:53:55.01ID:V0EzQ6WG
>>715
今の科学のご都合ご整合性先人や現行の妄想を鵜呑みにする妄想に現実的に否定を突きつけてるだけだが

妄想からは現実が妄想に見えるという現実
0724ご冗談でしょう?名無しさん
垢版 |
2021/08/22(日) 19:56:57.43ID:I4LLEu0h
スターグループなら全国に系列店があるから安心
0726ご冗談でしょう?名無しさん
垢版 |
2021/08/22(日) 20:08:03.82ID:Aq1hrY82
>>721
実に不思議なことに、文系さんたちの間には、🤓🤡🤢(´Д`😹)
《文科系>>>>>理科系》という謎の優越感(劣等感?)
信仰が強くあるよね。ところが実に不可解なことに、
東大→官僚(次官レースに乗ってる)てな奴が
その中に混ざったりはしていないんだよな🤣
0727ご冗談でしょう?名無しさん
垢版 |
2021/08/22(日) 20:08:41.93ID:V0EzQ6WG
>>721
それは理系も同じ
先人の理論なら、現行の理論なら研究の正しさが担保される鵜呑み
先人の理論から現行の理論からだけからしか研究しないやり方

理系も文系も人間としての根っこは同じ
鵜呑み族
鵜の中の鵜
先人主義、否定したら頭悪いから否定せず頭いいごっこ

本当はもの凄い頭いいのに勿体ない

ちなみに、頭いい人と頭悪い人って矛盾すると思う?
頭いい度と頭悪い度を両方持ったら相殺すると思う?
…しないんだよ
頭いいと頭悪いは一直線じゃなくx軸y軸

普通の人はどこに位置するか?

頭いい軸と
頭悪い軸と
両方ハイクオリティ

頭良いんだけど、頭悪いのが普通の人
キミたちだよ

キミたちは頭いいの
同時に頭悪いから
頭そこまで良くないの

天才は
頭いい軸高し
頭悪い軸低し
頭いいんだよ

自分の場合は
頭いい軸低し
頭悪い軸低し
頭悪くないけど頭良くない
に属する

頭いい…頭いい軸高、頭悪い軸低い
頭悪くない…頭いい軸低、頭悪い軸低い
─ニュートラル。↑は頭いい側↓は頭悪い側
頭良くない普通の人凡人…頭いい度高、頭悪い度高
頭悪い人…頭いい度低、頭悪い度高い

頭いい頭悪いが作る面積は

頭悪くない(自分みたいなのが)最小
頭良くない(キミたちのようなのが)最大

つまりキミたちは頭の能力的には頭いい天才を凌駕してるんだよ

でも頭悪い軸も高いからクオリティ低いだけ
0728ご冗談でしょう?名無しさん
垢版 |
2021/08/22(日) 20:10:03.03ID:V0EzQ6WG
キミたちは
ここの人たちは
普通の中堅の科学者は
凡人は

頭いいんだよ

頭悪いけど

頭の能力は天才以上
0730ご冗談でしょう?名無しさん
垢版 |
2021/08/22(日) 20:17:06.65ID:V0EzQ6WG
文系も同じ

凡人は

頭いい度高い、と同時に、頭悪い度高い

ハイクオリティ

頭の良さと悪さを両方持つ
高潔種

頭の良さと頭の悪さ両方持つから、勉強すれば、色々学問の中堅に至れる
頭の能力が高いから

自分みたいなのは
1番面積が最小、だから学問の世界に入るのは無理

だが頭悪くはない
0732ご冗談でしょう?名無しさん
垢版 |
2021/08/22(日) 20:20:21.20ID:V0EzQ6WG
そして
頭の良い軸が高く、頭悪い軸も高い
キミたちは
鵜呑みする
0733ご冗談でしょう?名無しさん
垢版 |
2021/08/22(日) 20:21:08.80ID:V0EzQ6WG
>>717
これ自分じゃない
0734ご冗談でしょう?名無しさん
垢版 |
2021/08/22(日) 20:21:41.06ID:V0EzQ6WG
頭いいキミたち
そろそろ目覚めろ
0737ご冗談でしょう?名無しさん
垢版 |
2021/08/22(日) 20:33:40.11ID:V0EzQ6WG
キミたちは
まず
天才より頭の能力は高いと
自覚するところから始めな

頭いい人たち

鵜呑みにするな
先人の理論を
現行の理論を

足し算だけで複雑化じゃなく
批判的思考=引き算思考、間伐する思考を身につけろ

複雑化した理論を裸にしろ

天動説じゃなく地動説
ご都合、ご整合主義でなく、裸にして根本を求めろ
0739ご冗談でしょう?名無しさん
垢版 |
2021/08/22(日) 20:55:02.19ID:???
菅首相の地盤である横浜市長選で自民候補が敗れる
もはや菅内閣は総辞職しかなくなった。

何故か日本では危機的状況で科学思考欠落の場当たり総理しか出ない
鳩山由紀夫、菅直人(偽理系)と大差ない。
0741ご冗談でしょう?名無しさん
垢版 |
2021/08/22(日) 22:19:04.86ID:lA877FwC
>>738
俺にはオマエこそきちがいに映るんだが?🤣🤓🤡🤢(´Д`🌀)
0743ご冗談でしょう?名無しさん
垢版 |
2021/08/22(日) 23:14:57.27ID:lA877FwC
>>742
オマエモナ〜🙄🤓🤡🤢(´Д`🔵)
0745p
垢版 |
2021/08/23(月) 01:57:26.14ID:yrHM9Bfc
>>717
他の大多数は「アホが空想して」じゃないぞ?
「アホが空想して」と「頭いい奴が迎合して」の違いもわからないなら、キミも「頭いい奴が迎合する」に迎合できない、こちら側の「アホが空想してるけど迎合してると思い込んでる」側だからな
「聞きかじったことだけ迎合しているが、聞きかじったことと違うか、キミに理解できなければ迎合しない」こちら側の人間だ
0746ご冗談でしょう?名無しさん
垢版 |
2021/08/23(月) 02:36:40.02ID:yrHM9Bfc
力学と各分野の統一を
近接力と遠隔力の統一を

物理記号【m】【s】【kg】【…】で全てを表す
物理記号を計算記号【ªº】【+−】…【^√】【⌒?】…数字の計算のみで表すように

量子論すら力学の機能(計算記号、物理記号)として表せるように
確率論を力学の機能で確定できるように

力学(物理記号)を突き詰めた数学(計算記号化)が全ての基点

科学はやがて数学に至り、数学で科学を表す、あらゆるものを解ける時代へ

数学は科学の根源である
0747p
垢版 |
2021/08/23(月) 02:50:05.87ID:yrHM9Bfc
>>698
この指摘は正解

>>697
ついでにこの指摘も正解って言ってほしいでしょ?
0748ご冗談でしょう?名無しさん
垢版 |
2021/08/23(月) 03:18:33.01ID:yrHM9Bfc
>>718
結構、頻繁に1週間2週間間隔あけてる

鳥頭は正解
0749ご冗談でしょう?名無しさん
垢版 |
2021/08/23(月) 03:27:56.24ID:yrHM9Bfc
>>719
これは不正解

上のレスで、レイドって言ったとおり
「思い付いたら爆撃」
反応が欲しいとかってわけでもないみたい
なんか「学問に挿入したい」だけかも
自分でもわからない。まだわかってない
0750ご冗談でしょう?名無しさん
垢版 |
2021/08/23(月) 03:33:16.88ID:yrHM9Bfc
>反応が欲しいとかってわけでもないみたい

って語尾が他人称だけど、他人称でなく自分称で曖昧って感覚を書き表したくて使った

自分でも自分の感覚は完全にはわからない
深い内面は
0751ご冗談でしょう?名無しさん
垢版 |
2021/08/23(月) 03:51:23.87ID:yrHM9Bfc
ちなみに余談だけど

精神年齢高い
精神年齢高くない
精神年齢低くない
精神年齢低い
の精神年齢の高い、精神年齢の低いも
X軸Y軸の面

精神年齢高くて、精神年齢低い=おっさん・おばさん
精神年齢高くて、精神年齢低くない=いい大人
精神年齢高くなくて、精神年齢低くない=いい子供
精神年齢高くなくて、精神年齢低い=ガキ

これも誹謗中傷でよく言うから書いた

あと、
精神年齢高い=物事を過剰に融合して、同一視する。関係ないことも融合させる。例:自分の感情と客観的事実を融合させる
精神年齢低い=物事を過剰に分離して、別視する。同じことも分離する。例:自分の感情や悪事などの規範や客観的事実を立場や見え方によって分離する
精神年齢高くなくて低くない=物事を正しく見れる

これは合ってるかわからない
0752ご冗談でしょう?名無しさん
垢版 |
2021/08/23(月) 03:56:59.28ID:yrHM9Bfc
>>751
間違ってるかも
わからない
0753ご冗談でしょう?名無しさん
垢版 |
2021/08/23(月) 04:11:47.47ID:yrHM9Bfc
いや、
精神年齢高い+精神年齢低い=おっさんおばさんで、過剰に融合させる人、過剰に分離する人、2種類いるか

するとz軸で二分化されて、過剰に融合される人、過剰に分離させる人がいる
つまりz軸の指標如何で過剰か過剰じゃないかになり、

正しくは
精神年齢高い=融合
精神年齢低い=分離
Z軸高い=過剰(何の指標化わからない)

いい子供は精神年齢高くなく+精神年齢低くない=融合も分離もさせないが、それが過剰で悪いかはZ軸……いやZ軸とα軸があるかも

精神年齢の誹謗中傷もよく言うから書こうと思ったけど間違ってたわ。わかってなかった。

頭良い頭悪いもz軸α軸あるとどうなるかわからないけど
とりあえず頭いい頭悪い、それと頭の能力の面積はたぶんX軸Y軸だけだと思う
0754ご冗談でしょう?名無しさん
垢版 |
2021/08/23(月) 04:14:49.35ID:yrHM9Bfc
>>753
Z軸α軸がある

z軸だけで過剰(過剰じゃない)1軸でなく
良い過剰、悪い過剰(過剰じゃない・過剰じゃない)で2軸
って思い直して、突然変更した
0755ご冗談でしょう?名無しさん
垢版 |
2021/08/23(月) 04:19:07.44ID:yrHM9Bfc
どちらにしても指標はわからない

頭いい、頭悪いの(誹謗中傷のよくあるもの)の繋がりで、精神年齢高い、精神年齢低い(誹謗中傷のよくある)を余談的にちょっとだけで書こうとしたけど、
わかってなくて無駄に多くなってしまった
これは意図してなかったごめん
0757ご冗談でしょう?名無しさん
垢版 |
2021/08/23(月) 08:27:07.65ID:???
>>756
normalizationが若干おかしい
あとthetaを[0, pi / 4)に制限するように速度と関係づける必要があるけれど
多分それって[[cosh, sinh], [shin, cosh]]と同じになるんじゃなかろうか
Eucledeanにしたいだけなら普通の回転行列でおk
0759ご冗談でしょう?名無しさん
垢版 |
2021/08/23(月) 11:53:45.39ID:+fvqUsxq
水蒸気と気温の関係ですが、空間内の水蒸気の水分子の運動が、太陽光線?などで励起され、結果、気温の上昇となる、のでしょうか?
0762ご冗談でしょう?名無しさん
垢版 |
2021/08/23(月) 17:26:24.65ID:Bmz9NINp
深谷賢治さんが

「普通は大学1年生で電磁気学を習い、2年生でベクトル解析を習うのですが、これはどうも順序が逆なんですね。大学2年生でやるベクトル解析は
1年生で習う電磁気学のところまで到達していないんです。また、ベクトル解析の本はすでに山のようにあるので、それを片っ端から眺めてみたのですが、
やはり大体は電磁気学に使えるようなところまでは書かれていない。その理由は自分が本を書いてみてようやくわかりました。ベクトル解析を電磁気学で
実際に使えるレベルまで書くのはすごくむずかしいのです。」

と書いていますが、これは本当ですか?

第1に、ベクトル解析が必要な電磁気学をベクトル解析よりも先に習うというのがおかしいですよね。
第2に、順序は逆でもそれでも習うベクトル解析のレベルが1年生で習う電磁気学に必要なレベルには達していないというのもおかしいですよね。

みなさんは、電磁気学に使えるようなレベルでベクトル解析を理解していますか?
0764ご冗談でしょう?名無しさん
垢版 |
2021/08/23(月) 18:05:02.85ID:Bmz9NINp
Michael Spivakの『Calculus on Manifolds』を読破すれば、電磁気学で必要なベクトル解析はマスターしたと言えますか?
0767ご冗談でしょう?名無しさん
垢版 |
2021/08/23(月) 21:04:50.16ID:SqeECu4s
>>765
オマエの親戚か?🤣🙄🤓🤡🤢(´Д`🌀)
0769ご冗談でしょう?名無しさん
垢版 |
2021/08/23(月) 22:12:58.64ID:Bmz9NINp
たとえば、戸田盛和さんのベクトル解析の本は電磁気学に使えるような本ではないでしょうね。
0771ご冗談でしょう?名無しさん
垢版 |
2021/08/24(火) 00:40:08.15ID:IoLnkOkC
>>769
何故そー思うの?🤣🤓🤡🤢(´Д`🌀)イヤヨンデナインダケドサー
0772ご冗談でしょう?名無しさん
垢版 |
2021/08/24(火) 00:42:32.99ID:IoLnkOkC
>>97
いやそのまさかだと思うが?🤣🤓🤡🤢(´Д`🌀)
0776ご冗談でしょう?名無しさん
垢版 |
2021/08/24(火) 10:48:31.76ID:???
ハミルトン力学の位相空間って運動量と位置をひとまとめにした空間ですよね
次元が異なる2つの量をまとめるのっていいんですか?
たとえば位相空間内での回転とか考えると次元が異なる2つの量が混ざり合ってヤバそうな気がするんですがどう理解すればいいですか?
0777ご冗談でしょう?名無しさん
垢版 |
2021/08/24(火) 11:00:28.24ID:???
>>776
例えば45度の位置にあればq,pを両方持ってるし
90度の位置にあれば片方だけ持ってるってだけ
というわけでq,pがごっちゃになるとは言えないし

これで不満ならそもそもqは位置エネルギーに対応してpは運動エネルギーに対応してるのでどっちもエネルギーだから次元は合ってる
0783ご冗談でしょう?名無しさん
垢版 |
2021/08/24(火) 21:06:03.92ID:???
韓国がノーベル賞級の発見!
ttps://news.biglobe.ne.jp/international/0823/rec_210823_1949994862.html

量子が波でもあり粒でもあるのは、観測した時に変わるというのが定説だったが
ついに韓国がそれは間違いであると実験で証明した

量子は特定の条件で変化する
2つの性質を持っているわけではない

これは、量子コンピュータの否定であり、二重スリット実験やパラレルワールドや
マルチバース理論やDブレーン宇宙など、ありとあらゆる量子力学を根底から覆すノーベル賞級の成果であると思います

皆さん的にはどういった感想をお持ちになりましたか?
0785ご冗談でしょう?名無しさん
垢版 |
2021/08/24(火) 21:35:44.84ID:7YNRcjgu
>>783
ちょっとナニ言ってるかさっぱりわからないんだけど、
もしかして合同結婚式あげたい若者たちの宗教なの?
0787ご冗談でしょう?名無しさん
垢版 |
2021/08/24(火) 23:47:01.42ID:???
>>784
その基礎科学を覆す内容の実験ですからな

>>785
俺も正直、完全に理解できるわけではない なんとなくしかわからん

>>786
今ある量子コンピュータは量子コンピュータでないってことなんだろう
よくわからないが

このニュース気になるんだよね
二重スリット実験も間違いだったってことなんだろうか
0788ご冗談でしょう?名無しさん
垢版 |
2021/08/25(水) 00:17:37.78ID:jqmcm0Vk
ブラックホールに吸い込まれていく人間はどの時点で死ぬと思いますか?
0790ご冗談でしょう?名無しさん
垢版 |
2021/08/25(水) 00:37:36.25ID:???
>>784
>基本的な知識が百年くらい前で止まってそうだな

「アインシュタインも理解できなかった秘密を韓国人科学者が解いた?」
「世界にばら撒く反日少女像の秘密」を解いたほうが面白そうだ。
0791ご冗談でしょう?名無しさん
垢版 |
2021/08/25(水) 00:56:37.02ID:jqmcm0Vk
>>789
ブラックホールに地面ってあるんですか!!!?
0793ご冗談でしょう?名無しさん
垢版 |
2021/08/25(水) 05:06:02.64ID:???
>>783
くっくっくの大勝利

彼は「質量が粒子性を、電荷が波動性を示す」と再三言っていた。

電子なら、その質量と電荷の重心は一致しておらず

電荷は質量の周りで波動的に振動していると。

量子力学は完全に虚構だと、最初から完全に見抜いていた。
0794ご冗談でしょう?名無しさん
垢版 |
2021/08/25(水) 05:08:43.27ID:???
>>786
何も分かってない。

何でも「量子」とつければ予算が付くから
名付けてるだけで、実質は超電導か光を使った新種コンピュータに過ぎない。

量子力学はまったく関係ない。
0799ご冗談でしょう?名無しさん
垢版 |
2021/08/25(水) 11:47:12.12ID:???
この手法は量子力学的な現象が量子コンピュータで正確にシミュレート出来ると示した上で決定不可能問題と対応付ければ何でも示せそう
0800ご冗談でしょう?名無しさん
垢版 |
2021/08/25(水) 14:23:20.76ID:???
アルミが反射する、これはわかる

完全遮光をうたったカーテン、傘などは、ぶつかった光エネルギーはどこにいくんですかね、爆散する?

吸光シートとの違いは

アーク溶接など、とにかくものすごい光エネルギーをぶち当てられたときにそれを消失させる、もっとも現実的でコスパのいい方法を知りたいです
0801ご冗談でしょう?名無しさん
垢版 |
2021/08/25(水) 14:34:15.32ID:ncuwV7ad
>>800
お客さん、レーザーにはレーザーですよ!物理の世界には、🤓🤡🤢(´ω`🔵)
レーザー冷却というのがあります。なんと数ミリケルビンまで
冷やっ冷やですぜ!…え、溶接…?位相を合わせてレーザー溶接じゃ!
0803ご冗談でしょう?名無しさん
垢版 |
2021/08/25(水) 14:41:33.53ID:???
よくわからんけど個人なんで
研究用設備とかないんで
アルミなら用意できて効果は実感できたのですが、ガチやばい本命にはアルミ突破されたのです
0804ご冗談でしょう?名無しさん
垢版 |
2021/08/25(水) 16:52:03.57ID:ncuwV7ad
>>802
日本人は、半島に核が落ちればいいなんて言わないし
そもそも思ってないよ。
半島人は年中日本に原爆落とせと宣伝してるね。

山本義隆なんて、川勝と一緒になってリニア妨害工作に必死だしな。
恥を知れよ人間の屑ども。
0809ご冗談でしょう?名無しさん
垢版 |
2021/08/25(水) 18:40:40.04ID:???
>>800
遮光カーテンは黒い布で吸収してるから熱になるだろ
ある程度以上に光量が多ければ集光して熱で蒸気を作って発電機回した方が高効率らしい
0811ご冗談でしょう?名無しさん
垢版 |
2021/08/25(水) 19:50:03.00ID:???
http://5ch.net/adv.html

差別・蔑視 *
 地域・人種などの差別発言は人権問題板で、蔑視的思想発言は政治思想板で、それ以外は削除対象になります。
 上記2板を含めて、差別・蔑視の意図がある地域名または苗字等の書き込みは、その真偽を問わず削除対象になります。
 それ以外の書き込みに関しては議論となることを前提に静観します。
0814ご冗談でしょう?名無しさん
垢版 |
2021/08/25(水) 20:47:42.94ID:???
結局、一般人が目も開けられないレベルの光害による嫌がらせにあったらアルミ以外に頼れるものはないのか
0815ご冗談でしょう?名無しさん
垢版 |
2021/08/25(水) 21:06:08.63ID:???
そういえば
ネトウヨも人なんだから叩いちゃだめ。彼を批判するだけだから左翼リベラルは信頼できない
と言っている凄いのを目撃したな
0818ご冗談でしょう?名無しさん
垢版 |
2021/08/26(木) 02:03:35.84ID:???
帯電ブラックホールの場合は2重の地平面の中に通常空間があるから地面がある可能性もある
2つの地平面の間は時間と空間が入れ替わってて停止できないけど、その内側ね。
数式で書けば
g_00 = -1 + 2GM/(c^2 r) - GQ^2/(c^4 r^2) … M:BH質量, Q:BH電荷
となってて r が 0 付近と無限遠で g_00 < 0 となってるのが通常空間
g_00 = 0 となる r が 2ヶ所あるのが地平面
0822ご冗談でしょう?名無しさん
垢版 |
2021/08/26(木) 13:05:43.09ID:Wh1s9Tpa
時間の一方向の流れ、って、熱力学第三法則と関係しているのですか?
0823ご冗談でしょう?名無しさん
垢版 |
2021/08/26(木) 13:28:34.56ID:???
>>822
因果律に関係。原因は必ず結果を生む。時間という原因は必ず次の時間という結果を生む。熱も伝わるという結果を生み拡散する。エネルギーもエントロピーという結果を生む。
0826ご冗談でしょう?名無しさん
垢版 |
2021/08/26(木) 13:43:56.13ID:???
>>822
一方向ではない
初期状態から未来方向でも過去方向でもエントロピーは増大する
エントロピーが時間方向に増大すると錯覚してるのは
初期状態を過去に置くのが普通だから
0828ご冗談でしょう?名無しさん
垢版 |
2021/08/26(木) 15:35:45.86ID:0VFeWLZe
何だ神田言って、
みんなQちゃんのこと大好きなんだね!🤣🤓🤡🤢(´ω`🔴)ハハハハ
0833ご冗談でしょう?名無しさん
垢版 |
2021/08/26(木) 16:23:05.49ID:0VFeWLZe
>>793
…しかしそれにしてもわからんなぁ…🤓🤡🤢(´Д`🌀)

たとえ工学屋上がりの電気バカだろうと、大学で量子力学くらい
単位とってくるだろうに…🤓…まぁ、理学部物理学科卒のはずなのに、
高校生レベルの微積の初歩まったく理解してなかったバカ知ってるが。
こともあろうに、「量子力学の成績凄く良かった」と自慢してたな🤣
0836poem
垢版 |
2021/08/26(木) 17:33:38.21ID:???
>>830
硬さは+−
空間での振る舞い(量)は×÷

V速度
×V、または÷Vというのは、元になる量VがあればV×V、V÷Vとなり、その単位は(m/s)^2、(1)と空間での振る舞いが変化してしまうから軟らかい。(×Vだと反応してしまう)

初速+Vまたは−Vだと、元になる量VがあってもV+V、V−Vで、その単位は変わらず(m/s)
だが速度Vが違うと衝突する。速度の異なる者にとってギャップは壁。+−の数値が異なると壁として振る舞う
×÷の数値が異なっても壁としては振る舞わない。融合するから。
つまり硬さを持つためには、×÷では持ち得ない。+−、または(上で言った)ªºしか物理的に、計算的に硬さを持ち得ない。
たぶん硬さは数値(強さ)を持つから+−。ªºだとXをどんな数字にしても+1にしかならないから、全ての硬さは1しか取り得ないとなる。だから硬さは+−。

つまり初速+Vなどの+として出る初期数値などは硬さ。観測初め0の時点で存在するギャップで壁

垂直抗力と硬量って質問だけど、垂直抗力はちょっとわからないけど、硬量はこれ。
垂直抗力というのは復元性みたいなもの?みたいな感じに思えるから、+−で復元性みたいな機能はない。
0838ご冗談でしょう?名無しさん
垢版 |
2021/08/26(木) 17:51:09.35ID:???
素粒子は大きさが無い点粒子らしいけどどういうこと?
大きさが無いってことは存在しないってことなのでは?
0839太上天君
垢版 |
2021/08/26(木) 17:55:41.64ID:/awJCNgK
>>836
キミは知ったかしてるけど、ノーベル賞は取ったの?

えーーーーーーーーーーーーっ!?ノーベル賞も取ってないのに知ったかしてるの???

wwwwwwwwwwwwwwwwwwwwwwwwwwwwwwwww
0840poem
垢版 |
2021/08/26(木) 18:02:14.44ID:???
>>836
復元性は純粋に質量の周辺だと思う

【ªº】X^0=100%、X=∞%〜−∞%。(これはわからん)
【+−】硬量(理由:壁)(硬量は比ではなく差)
【×÷】空間(変位)(理由:空間は拡がり(比))
【^√】実体?←NEW(理由: (∞)√ =1 (負の無限は1=0次元)最底が0次元なのでパッシブで固体(進行も消失もしない)。実体は消そうとしても非在化しない(と思う)からこれが実体(…何量?))
【⌒「 】時間(理由: (∞)「 =x (負の無限は基準の数=1次元)最底が1次元なのでパッシブで1次元進行。1次元進行だから時間)
【??】質量(パッシブで2次元進行するよう。質量の多世界連続体として他世界に質量の影響、というか、違うな。質量が肥大化し、他世界を飲み込んでいく可能性。連続体の0.000違いの世界と別世界だったこの世界がどんどん併呑していくのだろう)
【??】

【^√】は方向であり、空間の3次元以上のn方向ある。これはあらゆる方向に実体があるということ

【⌒「 】【??】時間と質量。時間は留まる、質量も慣性を持つなど復元性みたいなのを持つ
【×÷】は量、【^√】はエネルギー
エネルギーと言っても大抵は誤解しているが、量を変化させる変化自体のこと。
J=kg(m/s)^2とよく言うがこれは単なる運動系の量。×÷の量であり、エネルギーでなく力でしかない
(m/s)^2の^2がエネルギー。
エネルギーはほっとくと失う。加速度や色々な2乗量はほっとくと1乗量、定速度などになる。
ロケットのエンジン噴射で加速度を生み出しているのはkg(m/s)^2ではない。^〜〜であり、力が加速度を生みだしているのではなくエネルギーが加速度を生みだしている。量を支配する物理。変化させる物理。量を生み出す物理。ほっとくと失う物理がエネルギー。量は慣性で失わない。変化量は失うが(変化量はエネルギーの供給されてる間のみ保つ)

まあぐちゃぐちゃかいたけど
垂直抗力は計算階高い()
硬量は計算階低い(+−)
ってこと
0842poem
垢版 |
2021/08/26(木) 18:09:46.93ID:???
>>839
自分のいるタイミングでは久しぶり
似非・疑似・トンデモ諸々のと同じく個人論で日の目を見ることはないよ。科学的査読?無理無理。
なんか雑談する?
0843ご冗談でしょう?名無しさん
垢版 |
2021/08/26(木) 18:26:44.00ID:0VFeWLZe
>>834
何度でも言うが、私は断じてネトウヨなどではないよ。
思想的にはずっと左翼だ。天皇も大嫌いだし。だが、
日本に潜入しているブサヨクども(オマエのことだよw)
の破壊工作の酷さと悪質さの実態に吐き気がするわ。
そんな💩騒動で、わが眷属を汚すわけにはいかぬ。
0845太上天君
垢版 |
2021/08/26(木) 20:25:20.60ID:/awJCNgK
>>842
キミよ、知り合いにアマチュア将棋3段のやつがいるんだwww

そいつに言わせると「藤井聡太は大したことない。オレなら勝てる」だそうだw

そいつは将棋の大会に出たことはあるようだが、地方都市レベルの大会で優勝や準優勝はしたことがないらしいwww
だがそいつに言わせると、地方大会の上位者になると、プロの名人や高段者と同じレベルの将棋の達人らしいwww

まあザコというものはそういうものだと言うことwww

学問・物理の世界でも同じことだよwwwwwwwwwwwwww
0848ご冗談でしょう?名無しさん
垢版 |
2021/08/27(金) 15:49:25.06ID:oImgacuX
おまいら忘れてないか、ここは物理板だ。🤓🤡🤢(´Д`🔴)
物理を愛していない奴ら立ち去れ!
0849ご冗談でしょう?名無しさん
垢版 |
2021/08/27(金) 15:51:44.15ID:far3uX8A
>>848
お前が消えろ
0853poem
垢版 |
2021/08/27(金) 17:27:13.64ID:???
>>845
将棋(偽、学問)の大会(のような何かパプリックなイベントや勉強会程度にも)に参加したこともない

大学はFランで専門科目一つも取れなかったし、高校は通信制だったから微積分習ってない(他の数学単元も習ってないし、物理なんかも選択で中学レベルだったし)、大学の入試は高1レベルが解ければいいレベルだったし、
工学大学だったんだけど、そもそも工学だから学問の趣味と異なってた時点で進路間違いだし、工学じゃなく理学の才能もあるわけじゃないし、数学は苦手だし、中学レベルで止まってるしで、そこでキルヒホッフとか初めて習ったほどで、基礎実験科目だから習ったけど、理解できなかった(というかW(ワット)求めてエネルギー収支出すと計算合わなくない?)、実験の小論文書かされるけど書いても一カ所でも点もらえる部分(まともに書けてる所)なかった。
今はニートやって、定時制高校でも通おうかな(できるなら通いたいけど)って現状

ともかく、中学レベルで止まってるわけ。

中学レベルの学問で色々考えてるわけだから、この領域を科学の分野に当てはめると、基本的な力学と各分野の無学者が習う入門範囲と理学の文系大学の教養科目程度。
単位学と言うのがあれば力学+単位学、数学の分野に算数で習って以降やらない基礎計算学(計算記号学)ってないけど、あるなら自分の主な領域は
力学+単位学+基礎計算学(計算記号学)
の範囲。殆どこの範囲の論。読み返してみるとこの範囲で説明つく
この三単元が合わさって、数力学って分野が新たに生まれたらいいなって微かな希望

ともかく、将棋の大会に参加したことあるどころか大会には一切参加したことない、公民館で将棋打ってるレベルだから
地方大会のレベルとか知らないし、言ってるようになんだっけ?「地方大会のランカーは世界レベル?」(嬲るために参加することはあるだろうけど)
意訳:俺は世界レベルは倒せないけど、日本レベルなら倒せる、日本の一握りには敵わないが日本でほぼトップに食らいついてる?
まずは定時制で高校の勉強から始めたい
0856ご冗談でしょう?名無しさん
垢版 |
2021/08/27(金) 23:04:40.06ID:???
一般家庭用100Vのコンセントに並列に商品を差し込むと、あら不思議
電気料金が最大90%削減。
のように宣伝してる高額商品がある そんなことが物理学的に可能なのか
ぼったくり詐欺か
0858ご冗談でしょう?名無しさん
垢版 |
2021/08/27(金) 23:30:06.18ID:???
皆が装置を付けると電力会社が大損するから違法化されてしまう
ごく一部の人が使うことで得する商品。
0862ご冗談でしょう?名無しさん
垢版 |
2021/08/28(土) 00:23:39.24ID:???
電気料金とか詐欺が物理がどう関係あんだ
そんなもの商品説明見なきゃわからんから貼れと言ってる

貼れないならおまえが詐欺師
0863poem
垢版 |
2021/08/28(土) 00:53:28.17ID:???
>「地方大会のランカーは世界レベル?」(嬲るために参加することはあるだろうけど)
>意訳:俺は世界レベルは倒せないけど、日本レベルなら倒せる、日本の一握りには敵わないが日本でほぼトップに食らいついてる?

全然違ったわ


>>862
コンセントに刺す雷サージならAmazonで見た
ちょっとまってて
0864poem
垢版 |
2021/08/28(土) 00:56:43.27ID:Qoe5hAUF
貼れるかな

これ
貼れなかった
Amazon検索、「雷バスター」
0865poem
垢版 |
2021/08/28(土) 01:06:08.75ID:Qoe5hAUF
どういう原理なんだろ

この雷サージがあるなら同じ原理で他の電源口の電圧抑える機器あると思うけど

アナログな家電なら大丈夫だろうけど、デジタルな電子機器は大丈夫なのかな
0868poem
垢版 |
2021/08/28(土) 02:41:47.67ID:???
電圧を打ち消すんじゃない?
瞬間高電圧の逆位相を返して

なんかこれつけてスマホ充電しながら音楽聴くと音楽が歪むとかってレビューがあったような?

節電も多分同じ原理だと思う。交流の逆位相返すと電圧下がるんじゃない?
電圧下げれば電流減るから節電できる

節電の方は探してないけど
0869poem
垢版 |
2021/08/28(土) 02:43:16.69ID:???
>>868
瞬間高電圧の逆位相返すとか
そんなような説明書きだった虚覚え
0870poem
垢版 |
2021/08/28(土) 02:51:43.52ID:???
交流の逆位相で電圧消せるとかって交流電気の計算知ってる人とかっている?
工学の分野かな
ここ理学の人が殆どだろうから電気工学の分野までカバーしてない疑惑
0871poem
垢版 |
2021/08/28(土) 03:00:16.38ID:???
電気工学つながりでそういえばだけど
高電圧電線に鉄棒接触させてスパークとか解決してなかったじゃん

これもまた訊きたい
0872poem
垢版 |
2021/08/28(土) 03:49:34.47ID:Qoe5hAUF
ごめん
今雷バスター見返したら、ただの雷サージ(バイパス)みたい

逆位相って見たのあれだ、電源のノイズ消す奴だ。こちらもプラグだったはず
スマホ充電しながら音楽聴くと歪むってどっちのレビューだったかな
こちらはコンポ使ってる人が愛用しててどうの(ノイズが大きい地域とかなんとか?)

ノイズキャンセルは逆位相でやるって当然だけど、
よくよく考えたら電圧消すのを逆位相でやるってようはショートか

すると
ノイズキャンセル─逆位相(抵抗率振動?)
雷サージ─短絡
節電─?

節電プラグがあるとするなら
変圧器とか組み込んで、逆向き(順方向?)に流す電圧(と電流)変えたら電圧抑えられる(キルヒホッフの応用問題)とかって仕組みとかでできないかな


ごめん。雷サージの説明とノイズプラグの説明、記憶違いしてた
0873poem
垢版 |
2021/08/28(土) 04:02:25.95ID:???
>>860
>>866
>>その他
この問題、キルヒホッフの応用とか使って電圧電流トランスで変えたら、できないかな

なんかやってみて。単純に抵抗変えるとかそのまま流す遮断するの一電源方式じゃできない気がする
キルヒホッフの2電源方式なら可能かもしれないから
電力(エネルギー)収支が一致しないキルヒホッフの(通常の解き方の)問題がもう一方の電源電圧を変動させたり、抵抗に掛かる電圧が合成されてる可能性
0874poem
垢版 |
2021/08/28(土) 04:18:40.73ID:???
ノイズキャンセルと電圧消すって同一に認識してた。
ここの人達の量子論の話で逆位相で波を打ち消すとか見過ぎてたのも一因にありそう

雷サージプラグの説明書きとノイズキャンセルプラグの説明書きの記憶のすり替え
雷サージプラグ関係なかった。ごめん
0875ご冗談でしょう?名無しさん
垢版 |
2021/08/28(土) 04:43:37.19ID:???
コンセントに装置を挿すだけで実際の消費電力量より料金メータ電力量が何割も小さく出来るのか?
0876poem
垢版 |
2021/08/28(土) 05:01:52.48ID:???
電流の合成則←ある
抵抗の合成則←ある
電圧の合成則←ない
0877poen
垢版 |
2021/08/28(土) 05:32:56.57ID:???
>>875
キルヒホッフでどうなるか

電池二個の並列つなぎ
片方の電池が電源
もう片方は電源を変圧して二電源目(戻す)(電圧高くするか低くするか)

キルヒホッフ的にどうなるか誰か計算して
0878poem
垢版 |
2021/08/28(土) 06:00:08.16ID:???
電流合成則
a+a=A

抵抗合成則
並列
1/ω+1/ω=1/Ω
直列
ω+ω=Ω

電圧合成則
直列
v+v=V
並列
……?
(1/v+1/v=1/Vだと電池並列つなぎしたとき一個分の明るさにならない)

誰か教えて
0879poem
垢版 |
2021/08/28(土) 06:05:46.57ID:???
>>878
v1=v2のとき
v1=v2=V

v1≠v2の時の話
0880poem
垢版 |
2021/08/28(土) 06:30:42.44ID:???
色々弄くってこれしかあり得ないかな
電圧合成則
v1:V=V:v2
0881poem
垢版 |
2021/08/28(土) 06:44:42.83ID:???
すると電源だけの場合
機器の抵抗100Ω電源100V→1A

トランスで同電圧反射(+は+へ、−は−へ)
(XV)^2=100v1×100v2
X=100V
機器の電流1A

トランスで低電圧反射
100V→75V
V=√(7500)=86.6V
機器の電流0.866A

減った

トランスの電流消費
100V→75V
YA→4/3YA

Yを極端に小さくすれば
A=小さい

よって
0.866Aとか抑えられる?(節電プラグは実在可能?)
0882poem
垢版 |
2021/08/28(土) 06:54:40.56ID:???
極端に小さくすれば←できないか(変圧比で決まる)

86.6V
a+a=0.866A
a=0.433A

100V→75V
3/4YA→0.433A
Y=0.308

0.433+0.308=0.741A
このケースだと
机上の空論的に0.25A節電できる
0883poem
垢版 |
2021/08/28(土) 07:06:50.86ID:???
ほー…って思った
電圧の合成則判明してそれ使ったら、ほぼオームの法則とトランス比だけで解けて、中学の教養で解けるのか

自分からも大学時代から長年、なーがねーん抱いていた疑問
「キルヒホッフって完全に間違ってね?」
「この電圧合成則つかったらキルヒホッフの解どうなる?」(←面倒くさいから今度やろうって後回しモード)(計算面倒くさいから他の人にやってもらって答えだけ聞きたい)
0884poem
垢版 |
2021/08/28(土) 07:10:07.00ID:???
>>882
いやまった、
A=0.866
a=0.433←にはならなかったりするか?単純に1/2していいのか?だめじゃない?
どちらにしても節電はできるだろうけど
0885poem
垢版 |
2021/08/28(土) 07:13:26.79ID:???
あと余談だけど
キルヒホッフで電源逆向きにするの(電源同士でサーキットする向き)
電圧合成則したら虚数にならない?
0886poem
垢版 |
2021/08/28(土) 07:23:05.84ID:???
あ、V^2=(+V)(−V)だからならないか
二電源のサーキットになってて逆向きだからいいのか
すると真ん中の抵抗には直接電圧掛からないのか
0887poem
垢版 |
2021/08/28(土) 07:31:35.59ID:???
>>886
って、このサーキットキルヒホッフって並列電圧でなく直流電圧だから電圧則自体意味ないな

あとは真ん中の抵抗はホイートストンブリッジ@バラバラバージョン(たすき掛けに電源電圧のアンバランスが掛かるよね?)
0890ご冗談でしょう?名無しさん
垢版 |
2021/08/28(土) 07:56:31.02ID:saqbiFoZ
>>889
逝ってよしw
0892ご冗談でしょう?名無しさん
垢版 |
2021/08/28(土) 09:34:50.81ID:???
>>888
>力率改善器
それは企業向け設備による電力の契約料金
一般家庭100Vの電気料金には適用されない、企業向電力を家庭用に流用すると違法行為

一般家庭100Vのコンセントに付ける謎の装置が力率改善器のような物なら
実際の消費電力より電力量メータ値が何割も減らすのは物理的に可能か?
0895ご冗談でしょう?名無しさん
垢版 |
2021/08/28(土) 11:38:28.48ID:???
宇宙開闢から原作までの、全ての起こりうる状態の数が、
どのくらいの大きさになるのか示した人っていますか?

素粒子の数と空間の大きさが有限なので、とてつもなくでかい数字になるにせよ、無限にはならないと思うのですが。
0897ご冗談でしょう?名無しさん
垢版 |
2021/08/28(土) 11:47:12.06ID:???
>>893
違法以前に、コンセントに装置を挿すだけで物理学的に可能か?
だけ

>>894
電力契約の農業目的でない家庭生活に安い電力を使えば違法、町工場なども同様。
0898ご冗談でしょう?名無しさん
垢版 |
2021/08/28(土) 11:53:26.18ID:???
>>895
現在でも宇宙空間が有限かどうかは判っていない。
有限でなくともビッグバン説と矛盾しない、素粒子の数は密度と解釈したほうがいい。
0899ご冗談でしょう?名無しさん
垢版 |
2021/08/28(土) 12:03:53.34ID:???
>>897
はあ?自分で企業向電力を家庭用に流用すると違法行為とか制限付けといてどういうことだよ?
違法かどうか関係ないならできるって事だろうがよ
0900ご冗談でしょう?名無しさん
垢版 |
2021/08/28(土) 12:12:08.37ID:???
>>894
国の産業競争力政策で産業用の電力料金は一般家庭より割安に設定されている。
それでも先進国最低の韓国や台湾の電力料金の約2倍高い、自然エネ政策でさらに高くなる
電力を大量に使用する半導体の価格競争で日本国内の半導体企業はコストで負ける。
0901ご冗談でしょう?名無しさん
垢版 |
2021/08/28(土) 12:13:25.56ID:???
原子を野球場の大きさとすると
原子核が2mmの粒子で電子は髪の毛(0.1mm)よりも小さいということですが
原子核の中は真空なんでしょうか?
0902ご冗談でしょう?名無しさん
垢版 |
2021/08/28(土) 12:14:28.02ID:???
>>897
農家が進相コンデンサ付けて基本料金安く契約するってのは普通だろ。力率改善なんて客より寧ろ電力会社の都合なんだし
0903ご冗談でしょう?名無しさん
垢版 |
2021/08/28(土) 12:16:30.41ID:???
>>899
一般家庭の100V用のコンセントに付ける装置だとなんども言ってるだろ

その条件でも出来るというなら、物理学的な原理なり根拠があるのか
0904ご冗談でしょう?名無しさん
垢版 |
2021/08/28(土) 12:25:45.74ID:???
>>902
農家、町工場の動力用電源は料金メータが別に有るんだよ。家庭用料金メータとは別
違法配線・トランスを使い自宅・家庭の配線に繋いで使用すれば違法行為になる。
窃盗と同じ、女子大生がスマホ充電で捕まっただろ
0907ご冗談でしょう?名無しさん
垢版 |
2021/08/28(土) 12:31:34.16ID:???
>>903
どっちみちそういうのは法律や電力プランの話だろ、
物理学的にはコンセントに差した装置からスマートメーターに無線で割引適用指示を送るとか、
電力会社がそういう製品を作ろうと思えばどうにでもできる。
実際にあるかどうかは知らん。
0909ご冗談でしょう?名無しさん
垢版 |
2021/08/28(土) 13:01:26.32ID:???
>>907
>どっちみちそういうのは法律や電力プランの話だろ
何度も違うと言ってるだろ

コンセントに装置を挿すだけで最大90%も電気料金が安くなるなら
ぼったくり詐欺商品か、でないなら料金メータ自体を騙す電気的仕掛けがあるかだ。

進相コンデンサで力率を改善するだけなら有効電力で回る料金メータは騙せない。
0910ご冗談でしょう?名無しさん
垢版 |
2021/08/28(土) 13:25:51.21ID:???
>>909
だから違う違うばっかりでいったい何の話をしてんだ
全部の前提を否定して自分の結論に持ってくなら聞くなやボケナスが
0912ご冗談でしょう?名無しさん
垢版 |
2021/08/28(土) 13:49:14.25ID:???
>>910
有効電力を計る料金メータから繋がってるコンセントに手のひらサイズの装置
を並列に挿して他で使ってる有効電力消費が90%も減ったように料金メータ計を
操ることが電気理論的に可能か という単純な質問。

誰もそれが物理的に可能と説明したスレが無いから、詐欺商品ということで〆
0913ご冗談でしょう?名無しさん
垢版 |
2021/08/28(土) 14:03:57.90ID:???
電力会社からそういう仕組みが提供されてるならあくまで物理的には可能てレスあっただろ
仕組みが提供されてないのは物理の問題じゃないわ
何勝手な事ばっかり言ってるんだこいつは
0914太上天君
垢版 |
2021/08/28(土) 14:09:43.02ID:lRWAgGHA
キミたちに言っておく。

キミたちは何かを主張したらそれをきちんと証明することだ。
そして論文を書いてネイチャー誌に投稿したまえ。
そしてノーベル賞を取ることだよwww

wwwwwwwwwwwwwwwwwwwwwwwwwwwwwww
0915ご冗談でしょう?名無しさん
垢版 |
2021/08/28(土) 14:15:07.47ID:???
>>856
以前にも節電になるって流行りかけたな、同じものか知らんが
使い物にならなくなる電気器具が出たから廃れたんだろう
電流を部分的に止めるんだから当たり前だが
力率改善器なら違う原理だな、三相交流の大電力モーター以外には意味がないだろうが
0917ご冗談でしょう?名無しさん
垢版 |
2021/08/28(土) 14:25:12.49ID:???
>>915
電力量メータを操作する電気回路の仕掛自体は可能らしいが
その影響で他の電気器具は壊れる危険がある。ということね
ありがと。
0918ご冗談でしょう?名無しさん
垢版 |
2021/08/28(土) 15:16:32.08ID:zaiFLc4m
論法的に正しいでしょうか↓

反重力装置というのは時空の歪みを意図的に起こし
重力を作る装置です

時空の概念です

水晶の振動というのは時空の歪みの結果です
つまり水晶の振動の制御で反重力装置をできるわけです
ワープ技術は時空の概念です

洗濯機の容器の内側に振動する水晶を全周に配置
回転させると時空の歪みが生じて時空の渦ができます

渦というのは小さな力で大きな結果を出すことができます

もう一つ同じ装置を用意し今度は逆回転同じ回転数
同調させます

ゲート技術の出来上がりです
0919ご冗談でしょう?名無しさん
垢版 |
2021/08/28(土) 15:23:53.96ID:zaiFLc4m
もう一つ↓

ブラウン管テレビに使うような電子銃による
電子ビームによって出力される三色

これを新しい演算装置に使えないかと思うんですがどうでしょうか

量子コンピューターの原理見るとできそう
0920ご冗談でしょう?名無しさん
垢版 |
2021/08/28(土) 15:39:01.38ID:zaiFLc4m
もし電子ビームによる新しい演算装置ができるのであれば
50年代には開発されてなければならなかった

その当時ダイオードによる演算装置の可能性がもてはやされた時代
ほかの可能性を考えなかったと思われる

そしてそのまま時間が流れてしまった
0922ご冗談でしょう?名無しさん
垢版 |
2021/08/28(土) 15:52:10.35ID:zaiFLc4m
妄想というか論理的に電子ビームによる新しい演算装置
ができない理由を説明してください

質問です
よくわからないから妄想と断定?
0924ご冗談でしょう?名無しさん
垢版 |
2021/08/28(土) 15:57:13.39ID:zaiFLc4m
デジタルはあるかないかの論法

電子ビームによる新しい演算装置は
一つで3つの出力がなされる
色の数がよくわからないけどテレビに表示される
色数だと思う

その組み合わせで演算装置かできるのではないのかと思った

組み合わせ膨大
0926ご冗談でしょう?名無しさん
垢版 |
2021/08/28(土) 16:10:16.79ID:zaiFLc4m
論理的に否定してほしかったんですがね

わからないから無視ですか?
0927ご冗談でしょう?名無しさん
垢版 |
2021/08/28(土) 16:12:04.70ID:zaiFLc4m
わからないなら素直にわかりませんとも言えない人

これ新しい概念だから否定したいだろうけど
0928ご冗談でしょう?名無しさん
垢版 |
2021/08/28(土) 16:42:09.80ID:???
具体的内容も書かずに質問して拒否されたら具体的内容を書けって、厚かましいにも程があるだろw
0929ご冗談でしょう?名無しさん
垢版 |
2021/08/28(土) 16:48:27.40ID:zaiFLc4m
これすごく単純な論法なんですよ

ここまで説明されて理解できませんか?
0933太上天君
垢版 |
2021/08/28(土) 19:21:57.01ID:lRWAgGHA
キミたちの中にノーベル賞を取った者はいるかね?

何!?いない?

キミたちは物理を語るのは早すぎないか?

wwwwwwwwwwwwwwwwwwwwwwwwwwwww
0935ご冗談でしょう?名無しさん
垢版 |
2021/08/28(土) 20:54:42.66ID:???
>>918
中性子星が衝突してやっと超高精度の測定器で分かる重力波ができるんだ
人が作れる程度の水晶ごときで起こす重力波など誰も気付かねーよ
それとワープは単なる重力場じゃない、因果律までねじ曲げる技術が
重力制御ごときで使えるわけがない

>>919
そりゃ進行波管だろ、それともその前のビーム4極管か?
進行波管は普通の真空管より高性能で色々使われたけど
今は半導体に置き換わってる
0937ご冗談でしょう?名無しさん
垢版 |
2021/08/29(日) 01:47:54.00ID:zoIgsPOn
結局この宇宙は細部まで決定論的に動いているのでしょうか?
0939ご冗談でしょう?名無しさん
垢版 |
2021/08/29(日) 05:21:15.21ID:???
利己的遺伝子論ってほんとか? 怪しくないか?
利己的原子とか、利己的素粒子とか、利己的分子とかはないのはほぼ明らかだろうが
遺伝子までなると、自他の区別がついて、他を滅ぼすような働きを持つのか?
ハチとかアリが集団行動とるのは血縁≒遺伝子が関係してるってことかららしいが
これは結果論で、似たような行動・発想だから集団行動するだけではないのか?
0941939
垢版 |
2021/08/29(日) 05:55:36.02ID:???
この一番目の発言でそうか?と疑問におもった
人間の人生だと赤ちゃんが最も利己的って事になるわけだが


リチャード・ドーキンスは著書『利己的な遺伝子』の中で、「利他的であることは利己的であることの延長にある」と述べています。
つまりこういうことです。生物の行動は、個体として生き残って種として繁栄していくように遺伝子的に規定されています。
しかし、それには順番があり、種を繁栄させる前に、まず個体として生き残らなければなりません。赤ちゃんが利己的なのは、個体の生存戦略上、当然のことなのです。
https://diamond.jp/articles/-/278480

道家道学院
水に学べ、赤子に学べ
老子の思想は 水のようにさらさらと高いところから低いところへと流れ どんな器にも添う 柔軟で柔らかい生き方が最善であると説いています。
それは純粋無垢な赤子の生き方でもあるのです。
水や赤子のような 柔軟で弱い生き方が 本当は最も強い生き方であり 無為自然な姿だと 老子は説いています。
http://tokyo-dokan.jp/taoistroad/roushi.html

「神の国に入れる人」
イエスに触れていただくために、人々は乳飲み子までも連れて来た。弟子たちは、これを見て叱った。
しかし、イエスは乳飲み子たちを呼び寄せて言われた。
「子供たちをわたしのところに来させなさい。妨げてはならない。
神の国はこのような者たちのものである。
はっきり言っておく。子供のように神の国を受け入れる人でなければ、決してそこに入ることはできない」
乳飲み子は、物心もついていません。イエスさまのお話を、頭で理解することもできません。
ただ、お母さんなどの懐に抱かれているだけです。
その乳飲み子のようになれとは、いったいどういうことでしょうか。
自分を抱いている人、親に、全面的に信頼している。全くゆだねきっている姿がそれです。
神さまに対してこのようであれ、ということです。
http://zushikyokai.holy.jp/sermon/ser_140511.html
0943ご冗談でしょう?名無しさん
垢版 |
2021/08/29(日) 11:18:48.03ID:/Bnipgn7
(質問)
最近の事件で「警察官が威嚇の為に上空に向けて発砲」とありますが、
その弾丸はどうなるの?
落下して誰かに当たって、って事もあるんだよね?
0946ご冗談でしょう?名無しさん
垢版 |
2021/08/29(日) 12:47:23.64ID:???
>>937
決定論はとっくに否定されている
少しは世間の情報を仕入れたら?

>>938
統一理論ではない、4つの相互作用は標準理論
普通の音は電磁気力で構成された分子間の現象
中性子星の中で伝わる音なら強相互作用
0947ご冗談でしょう?名無しさん
垢版 |
2021/08/29(日) 12:48:00.42ID:???
>>939
そういう名前を付けただけ、言葉の一人歩きは妄想にしかならん
利己的分子と言えるのは水分子だな
酸素と水素で水ができる反応を触媒して自分自身を増やすんだから
充分に利己的だろ
0948ご冗談でしょう?名無しさん
垢版 |
2021/08/29(日) 12:54:54.83ID:zoIgsPOn
>>946
そうなんですね
でもスレタイ通りのちょっとしたことを聞きたかったので
0949ご冗談でしょう?名無しさん
垢版 |
2021/08/29(日) 13:02:51.23ID:???
疑問というかアレだけど
よりファンダメンタルな理論が
単にIF文全パターン書き下す以外に表現方法の内容な理論で
美しさとか方程式とかとは無縁である可能性も否定できないって理解で合ってる?
0951ご冗談でしょう?名無しさん
垢版 |
2021/08/29(日) 14:41:01.63ID:???
>>949
面白そうな質問に見えるから興味もったけど理解できなかった
IF文全パターン書き下す理論ってどういうこと?
具体例とか欲しい
0953ご冗談でしょう?名無しさん
垢版 |
2021/08/29(日) 15:06:14.07ID:???
>>952
うーーん
それってIF文を書き下す理論(=IF文の並び全体で理論になっている)というよりIF文の一つひとつが理論になってるのでは?
いまいち「IF文全パターン書き下す理論」という言葉の意味が分からない
なんか面白そうな予感はするんだけどなあ

それから「美しさとか方程式とかとは無縁である可能性も否定できない」はどういう意味?
0954ご冗談でしょう?名無しさん
垢版 |
2021/08/29(日) 15:26:47.79ID:???
>>949
少数の前提から多くの結果を出せる構造でなければ理論として構成できない
現実が関連ないバラバラの組み合わせである可能性はあるけど理論には作れないので
作られる理論は何らかの美しさや方程式を持つ
0955ご冗談でしょう?名無しさん
垢版 |
2021/08/29(日) 15:27:25.59ID:???
>>953
確かに言葉が曖昧で俺もイメージしきれていないから正しく伝えきれてないんだと思う
後者は単純にソフトウェア工学とかでは忌避されるコーディングだよねくらいの意味
0956ご冗談でしょう?名無しさん
垢版 |
2021/08/29(日) 15:29:39.81ID:???
>>954
それは自然科学の言語として数学を選択したことによる弊害では?
そりゃ数学は構造を持つから数学に依存する理論も構造を必然的に持つと言われればそりゃそうでしょう
0957ご冗談でしょう?名無しさん
垢版 |
2021/08/29(日) 15:48:03.32ID:T40cexdT
何で知性も教養も立派な学歴もありそうなのに限って、🤓🤡🤢(´Д`🌀)
この板にしばらく居ると、こっそり釣り嵐になるんですか?
0958ご冗談でしょう?名無しさん
垢版 |
2021/08/29(日) 15:52:29.79ID:???
>>955
ちょっとまずあなたの日本語を正確にしたい
「ソフトウェア工学とかでは忌避されるコーディング」=IF文だらけの理論
という意味でおk?

「よりファンダメンタルな理論が
単にIF文全パターン書き下す以外に表現方法の内容な理論で
美しさとか方程式とかとは無縁である」かもしれない
とは
「よりファンダメンタルな理論」がIFだらけのきったねえ理論かもということでおk?

あとそれから「よりファンダメンタルな理論」とは何の理論と比較して?

だんだん言いたいことは分かってきたから次のレスくらいで回答出せると思う
0959ご冗談でしょう?名無しさん
垢版 |
2021/08/29(日) 15:53:30.60ID:BjiCJ0a9
>>957
まともに議論に参加しようという気概のないやつにはこっちは用はねえんだよ
今おもしろくなりそうな話してんだからすっこんでろゴミ
0960ご冗談でしょう?名無しさん
垢版 |
2021/08/29(日) 16:03:11.91ID:???
>>958
そうそう。
だいたいそんな感じ
あまりにも曖昧すぎるからファンダメンタルな理論(標準理論の先を想定)に話を限定してるだけ
要するにソロンの先に一本の方程式を追求してるけど、実はそれ自体が幻想である可能性もあるのかなあと
0961ご冗談でしょう?名無しさん
垢版 |
2021/08/29(日) 16:16:24.48ID:???
>>960
ははあ、そういうことですか
まずあなたがどのくらい予備知識あるのか分からないけど
一応いま標準模型の話がでてきたのでその文脈に即して回答します。

標準模型の親玉であるゲージ理論にはいくつかパラメタをインプットする余地があって
これがたぶんあなたのいうIF文うんちゃらかんちゃらということなのだと思う
このパラメタにかくかく設定(あなたのいうIF文)すれば、こんな理論が出力される、
しかじか設定すれば、あんな理論が出力される、といった具合に。
そしてこのパラメタたちに特定の(現実にそうであるような)パラメタをインプットしたものが標準模型。

で、あなたの言葉に寄せていうところの「IF文の余地が少ない理論」というのは
ゲージ理論に即して言えば、理論からはアプリオリに決まらないインプットパラメタが少ない理論、
と言えると思うんだけど、まさにこれがBSM( = beyond standard model)の目指してる境地と言えると思う。
もちろんまだ実験の射程には入ってない。

そういうわけで実験で確証されてる範囲内(標準模型)だけでいえばパラメタだらけの理論、
あなたの言葉いうところのIF文いっぱいの理論ってこと。
で、もしBSMもパラメタだらけだったとしても、やっぱりその理由をさらに高エネルギーな理論に追い求める必要が
生じるから、結局のところ、BSMは究極にパラメタの数が削ぎ落とさた究極理論に行き着かなきゃいけないんだろう
その究極理論の候補の一つが超弦理論。
超弦理論にはパラメタがたった一つしかないから、実験の射程にはまだまだ入らないとは言え、多くの理論家を満足させてる。
これで回答になったかしら。

IF文うんぬんかんぬんってのがパラメタの話かと思って繰り込み思想に新しい風でも吹き込んでくれるかと一瞬期待しかけたけど勘違いかも
0962ご冗談でしょう?名無しさん
垢版 |
2021/08/29(日) 16:17:29.45ID:???
幻想の可能性など当然でしょ
そっちを追求しても無駄だから成果がでる可能性がある方を追求するだけ
0963ご冗談でしょう?名無しさん
垢版 |
2021/08/29(日) 16:24:00.08ID:???
>>961
ただ単に思いつきを書き込んでいるだけだからそこまで深くは考えていない、こっちこそごめん
現状だとその答えになるのはわかってるんだ
0964ご冗談でしょう?名無しさん
垢版 |
2021/08/29(日) 16:27:11.16ID:???
>>961
「現状だとその答えになるのはわかってる」とは
あなたに既にある程度の予備知識があって、>>961くらいのことは分かっている、という理解でおk?
ならば、なおさら、あなたが今どういうインスピレーションを持ってるのか気になるんだが
0969ご冗談でしょう?名無しさん
垢版 |
2021/08/30(月) 14:31:16.28ID:???
Universe Sandbox2ってやつでブラックホールのシミュレーションみてみたんだけど
何個も配置した天体が吸い込まれてるどころか、スイングバイ的にふっとばされてるのが殆どなんだけど
そういうものなの・・?
てっきり、鳴門の渦潮みたいにどんどん合体していくイメージだったんだが
0970ご冗談でしょう?名無しさん
垢版 |
2021/08/30(月) 14:53:29.56ID:GmJFRPc5
白色矮星
https://ja.wikipedia.org/wiki/%E7%99%BD%E8%89%B2%E7%9F%AE%E6%98%9F
>白色矮星の大部分を占める縮退した電子は熱をよく伝導する。
>そのため白色矮星の質量のほとんどは等温で、また高温である。
>表面温度が 8,000 K から 16,000 K の白色矮星は、コアの温度はおよそ 5,000,000 K から 20,000,000 K であると考えられる。


白色矮星の多くの領域は、等温になっているらしい。
そこで以下の理屈から思ったんだけど、白色矮星の光度は中心部の温度の1.5乗倍に比例しているんじゃないか?
まず、白色矮星の単位面積当たりのエネルギー輸送効率は光の圧力と総圧力の比に比例する。
縮退した電子1個あたりのエネルギーは総圧力の5分の2乗に比例するとすると、表面付近の縮退が解ける領域の総圧力は温度の2分の5乗に比例する。
そして光の圧力は温度の4乗に比例するから、光の圧力と総圧力の比は、温度の1.5乗に比例する事になり、
白色矮星の光度は、中心部の温度の1.5乗に比例する事になる。
0973ご冗談でしょう?名無しさん
垢版 |
2021/08/30(月) 16:19:49.73ID:???
ブラックホールでジェットが出るってのも
吸い込み損なったプラズマがぶっ飛んでんだ
渦で絞られた磁力線がノズルになってる事もあるがな
0975焼き肉学
垢版 |
2021/08/30(月) 18:40:10.71ID:???
数学、力学、熱力学、電磁気てどうだろう ガスとか使うし。
0976ご冗談でしょう?名無しさん
垢版 |
2021/08/30(月) 18:47:24.20ID:???
>>971
>輸送効率からして何でそうなるか分からんな

「単位面積当たりのエネルギー輸送効率は光の圧力と総圧力の比に比例する」と言うのは、
↓のPDFのP123の右側上あたりの数式を参考にして言っている。

ハヤシ・フェイズが発見された頃(P123の右側上あたり)
http://www.asj.or.jp/geppou/archive_open/1996/pdf/19960303c.pdf
「縮退した電子1個あたりのエネルギーは総圧力の5分の2乗に比例する」と言うのは、
↓のグラフを見てそう感じたから言っている。

このグラフは、白色矮星の質量ごとの理論的な半径を示している。
https://ja.wikipedia.org/wiki/%E3%83%9A%E3%82%AC%E3%82%B9%E3%82%B9%E5%BA%A7IK%E6%98%9F#/media/%E3%83%95%E3%82%A1%E3%82%A4%E3%83%AB:ChandrasekharLimitGraph.svg
0977ご冗談でしょう?名無しさん
垢版 |
2021/08/31(火) 00:45:14.41ID:???
エネルギー輸送効率てのはエネルギー流量のことか
圧力比に内側の質量が掛かってるのを無視しちゃダメだろ
0979ご冗談でしょう?名無しさん
垢版 |
2021/08/31(火) 07:37:09.37ID:???
真空に高電圧を掛けると、気体が固体化する。エーテルが存在するとすれば、真空の何かに干渉した結果となるだろう
0982ご冗談でしょう?名無しさん
垢版 |
2021/08/31(火) 10:43:35.81ID:yUCCSH06
簡単な質問で申し訳ありません

下線部の3P(triplet p)と1P(singlet p)は逆ですよね?

1重項スピン関数χ(0,0)を掛け合わせた方が1Pで、3重項スピン関数χ(1,m)を掛け合わせた方は3Pでよろしいでしょうか?

https://i.imgur.com/icZs793.jpg
0983ご冗談でしょう?名無しさん
垢版 |
2021/08/31(火) 11:21:11.00ID:nkR2h0VH
光などの電磁波は、「屈折」させると波長によって曲がり具合が異なる
だから分光ができるが、

「反射」させるとどんな波長でも必ず揃ったまま経路が変わるってんだから不思議だなあって
0985ご冗談でしょう?名無しさん
垢版 |
2021/08/31(火) 11:48:24.22ID:???
>>983
光は最小時間時間で伝搬する。 フェルマーの原理
物質の違いで光の振動数に対する速度が変る。
それでどちらの現象も説明できる。

物理の原理から説明するのが物理学だからね、別々の現象のままで感想は不要。
0986ご冗談でしょう?名無しさん
垢版 |
2021/08/31(火) 11:50:51.94ID:yUCCSH06
>>982
もう1点お願いいたします

・Ψμ(+)には1重項スピン関数χ(0,0)を掛け
・Ψμ(−)には3重項スピン関数χ(1,m)を掛け
との記述も逆な気がします

正しくはこうでしょうか?
・Ψμ(−)には1重項スピン関数χ(0,0)を掛け
・Ψμ(+)には3重項スピン関数χ(1,m)を掛け
0988ご冗談でしょう?名無しさん
垢版 |
2021/08/31(火) 12:42:00.85ID:yUCCSH06
>>987
なるほど

Ψμ(+)が対称で1重項スピン関数χ(0,0)が反対称だから掛け合わせて反対称

Ψμ(−)は反対称で3重項スピン関数χ(1,m)が対称だから掛け合わせて反対称

ということですね
0990970
垢版 |
2021/08/31(火) 13:50:55.33ID:2M2LeV7D
>>977
>圧力比に内側の質量が掛かってるのを無視しちゃダメだろ

「白色矮星の光度は中心部の温度の1.5乗倍に比例しているんじゃないか?」と言うのは、質量固定の話だよ。
>>976 で参考にした数式から白色矮星の光度は白色矮星の質量にも比例するんじゃないか?
0992ご冗談でしょう?名無しさん
垢版 |
2021/08/31(火) 21:17:14.35ID:89vtDkjy
タイムパラドックスを防ぐために変装をしました。(サングラスなどを掛けて)
それで、貴女と同一人物であることをサングラスを取らずに告げました。この場合、タイムパラドックスに陥るでしょうか?
0993ご冗談でしょう?名無しさん
垢版 |
2021/08/31(火) 21:36:02.41ID:89vtDkjy
貴女というのは、大人の方の女性です。変装をしたのは、小さな女の子です
0995ご冗談でしょう?名無しさん
垢版 |
2021/08/31(火) 22:52:51.59ID:???
電子顕微鏡ってあるじゃないですか。
物凄く細かく見えて原子まであるかないかわかって原子で文字が書いてあるの見えるようなの。
原子に近づけると電流が流れる量が凄くなるからそれでわかる、と。
トンネル効果でわかるのはいいんだけど、そんなに小さく細かく精密に短針針をどうやって動かせるもんなの?
原子の大きさ以下の緻密な操作なんでしょう?
0999ご冗談でしょう?名無しさん
垢版 |
2021/08/31(火) 23:51:32.66ID:HDoOvl7z
2001年宇宙の旅のエンディングは
結局何を言いたかったのでしょうか?
1000ご冗談でしょう?名無しさん
垢版 |
2021/08/31(火) 23:57:38.85ID:P5K1t35+
ループ量子重力ってもう死んでる?
10011001
垢版 |
Over 1000Thread
このスレッドは1000を超えました。
新しいスレッドを立ててください。
life time: 33日 15時間 0分 9秒
10021002
垢版 |
Over 1000Thread
5ちゃんねるの運営はプレミアム会員の皆さまに支えられています。
運営にご協力お願いいたします。


───────────────────
《プレミアム会員の主な特典》
★ 5ちゃんねる専用ブラウザからの広告除去
★ 5ちゃんねるの過去ログを取得
★ 書き込み規制の緩和
───────────────────

会員登録には個人情報は一切必要ありません。
月300円から匿名でご購入いただけます。

▼ プレミアム会員登録はこちら ▼
https://premium.5ch.net/

▼ 浪人ログインはこちら ▼
https://login.5ch.net/login.php
レス数が1000を超えています。これ以上書き込みはできません。

ニューススポーツなんでも実況